NPTE Questions/Facts

अब Quizwiz के साथ अपने होमवर्क और परीक्षाओं को एस करें!

What is the level of Cognitive Functioning Scale?

The Rancho Los Amigos Levels of Cognitive Functioning scale is used to determine the cognitive and behavioral recovery in individuals following a traumatic brain injury as they emerge from a coma

How is Finkelstein's test performed?

The examiner passively pulls the wrist and thumb into ulnar deviation and applies longitudinal traction

Athetosis

slow, irregular, twisting, involuntary movements

Tonic Labyrinthine Reflex (TLR)

stimulus: position of labyrinth in inner ear - reflected in head position response: -in the supine position, body and extremities are held in extension -in prone position, body and extremities are held in flexion normal age of response: birth to 6 months

Petrissage

technique which consists of kneading manipulations that press and roll the muscles under the fingers or hands. It is used to break up adhesions between the skin and underlying tissue, increase elasticity of the skin, loosen adherent fibrous tissue, and improve venous and lymphatic return.

Transverse friction massage

technique which involves the use of a thumb or index finger to exert intense pressure in a direction perpendicular to the muscle fibers usually with the tendon placed on slight stretch. The massage technique is most commonly utilized in chronic overuse problems such as rotator cuff tendonitis, patellar tendonitis, medial epicondylitis, and lateral epicondylitis.

Endotracheal suctioning

This airway clearance technique is used ONLY when regular cough technique, huffing, assisted cough and tracheal stimulation has failed to adequately remove secretions

Tympanic membrane temperature reads what?

Tympanic membrane temperature reads the infrared heat waves released by the ear's tympanic membrane. An accurate measurement requires the examiner to pull the ear backward to straighten the ear canal.

What is Balance Efficacy Scale (BES) used for?

Used to measure a patient's self-confidence when performing functional tasks -Less than 50 = Low Self-Confidence with ADL's

Hemi-arm sling

Vertical arm sling that uses a humeral cuff with figure-eight suspension to provide a vertical upward force that supports the humerus without restricting the elbow and forearm Used with patients with shoulder subluxation

What is bulbar palsy?

Weakness or paralysis of the muscles innervated by the motor nuclei of the lower brainstem; affects the muscles of the face, tongue, larynx, and pharynx

Overflow urine incontinence

When the patient leaks or dribbles urine because your bladder is too full

Hyperkalemia can produce what on ECG?

Widened PR intervals and QRS or, tall T waves.

What is tertiary intention or delayed primary intention?

Wound is allowed to heal by secondary intension then closed by primary intention as final treatment

What is purulent exudate?

Yellow or green in color with a thick, viscous consistency. Purulent exudate often signals wound infection and may be associated with a foul odor.

Silver sulfadiazine is what?

an antimicrobial drug used for the prevention and treatment of wound sepsis.

Grave's disease

an autoimmune disorder that is caused by hyperthyroidism *Heat intolerance, nervousness, weight loss, tremor, and palpations

Simple random sample

every member of the population has a known and equal chance of selection

What is xeroderma?

excessive dryness of the skin -Could indicate thyroid deficiency or diabetes

What are urticaria?

hives

Noble Compression Test

identifies if distal iliotibial (IT) band friction syndrome is present Pt is supine + knee flexed at 90 degrees + hip flexed at 45 degrees Pressure is applied to the lateral femoral epicondyle while extending the knee (+) Test = reproduce pain

Cluster sampling

The population is divided into clusters or areas and random sample of the clusters is selected -All the units in the selected sample are measured *Less costly and more efficient than simple random sampling

What does visceral pain from the gall bladder refer to?

The right mid-back and lower back

Analgesia

inability to feel pain

Ataxia is what?

lack of muscular coordination Characterized by: -Staggering and unsteadiness -Wide base of support and movements are exaggerated

Stress urinary incontinence (SUI)

loss of bladder control caused by the application of external pressure

What is serosanguinous exudate?

pink in color and can be a normal exudate in a healthy healing wound.

A Swan-Ganz Catheter measures what?

-Central venous pressure (CVP) -Pulmonary artery pressure (PA) -Pulmonary capillary wedge pressure (PCWP)

What does a negative pressure wound therapy typically indicate?

-Chronic or acute wounds --> cannot be closed by primary intention: --Dehisced surgical incisions --Full-thickness wounds --Partial thickness burns --Heavily draining granular wounds --Flaps --Grafts --Most ulcer types

What are the contraindications for compression therapy?

-Circulatory obstruction -DVT -Heart Failure -Infection -Malignancy of treated area -Unstable or acute fracture -PE

Deep partial thickness burns show the follow what features?

-Color = mixed red, waxy white, blanching with slow capillary refill -Surface = broken blisters, wet surface, insensitive to light touch OR soft pin prick but sensitive to pressure -Edema = Excess scars and slow healing

Treatment of venous insufficiency

-Compression -Exercise -Proper shoe wear -Regular skin inspection -Elevation

What is abscess?

-Containing puss and surrounded by inflamed tissue --> from localized infection --Bacterial infection -Healing = draining or incising the site

What are common symptoms of heat stroke that are not part of heat exhaustion?

-Flushed -Dry skin -Labored breathing -Stong/rapid pulse -Convulsions -Pupils that contract and dilate

Hypokalemia

-Hypokalemia refers to severe potassium depletion in the circulating blood. -The condition is commonly manifested by episodes of muscular weakness or paralysis and postural hypotension.

Hypomagnesemia results in what?

-Increased intracellular calcium levels -Vomiting convulsions -Muscle cramps -Hyperreflexia -Falls

Acromioclavicular joint is common in what population? What movement irritates it the joint the most?

-Long-term weightlifters -Individuals that perform a great deal of overhead work Across their chest motion compresses the AC joint

What do tracheal breath sound like?

-Loud -Tubular sounds

What is valgus stress test in full extension checking?

-MCL -ACL -PCL -Posterio-medial capsule -Semimembranosus -Medial Quads -Posterior oblique Ligament

A PT performs 'Valgus stress' test in 30 degree of knee flexion, which of the following structures can be accessed by this test when checking medio-lateral instability?

-MCL -PCL -Posterio-medial capsule -Posterior oblique ligament

What do bronchial sound like?

-More tubular -Hollow sounding -Louder than vesicular breath -Heard over anterior chest near the 2nd and 3rd intercostal spaces *Inspiratory phase is shorter than the expiratory phase *Slight pause between the two

Acetic acid

-Negative polarity -Reduced calcific deposits -Treatment = myositis ossificans *Iontophoresis

What are the risks of HRT?

-PE -Stroke -DVT -Gallbladder disease -Small increase in breast cancer

What are the abnormal positions associated with flatfoot deformity?

-PF on the talus -Lateral drift of the calcaneus -Lateral drift of the cuboid as it follows the calcaneus -DF of the metatarsals -Abduction of the forefoot

Zinc oxide

-Positive polarity -Antiseptic -Promotes tissue healing --> treat skin ulcers *Iontophoresis

Lidocaine

-Positive polarity -Local anesthetic used to treat soft tissue pain *Iontophoresis

Magnesium sulfate

-Positive polarity -Muscle relaxant -Treat muscle spasms *Iontophoresis

When can CPM be used?

-Post fracture immbolization -Tendon -Ligament repair *NOT in TKA

What is a common symptoms for heat exhaustion?

-Profuse sweating -Normal body temperature (may be slightly elevated) *Heat exhaustion --> Heat Stroke = the pt sweats less and skin becomes dry

Lacunar Infarcts

-Pure motor hemiparesis -Pure sensory stroke -Ataxic hemiparesis

What is the goal treatment for eschar?

-Sharp debridement -Enzymatic debridement -Autolytic debridement -Wet to dry dressing -Wound irrigation -Hydrotherapy

What do vesicular breath sound like?

-Soft -High-pitched -Breezy sounds *Inspiratory phase is longer than the expiratory phase and no pause between the two

What are the symptoms of vegetative state (VS)?

-Transient state of wakefulness without awareness characterized by cyclic sleep patterns -Spontaneous eye opening and movement -Cognitive function -Normal speech -Normal body temperature -Lacks purposeful responsiveness to stimuli *Persistent = >1 month after acute trauma *Permanent = 3 month after nontraumatic brain injury or 12 months after traumatic brain injury

Senile osteoporosis

-Typically occurs with pts. over the age of 70 -Bone formation changes (cortical and trabecular bone) occur due to increasing age = Increase of fractures (prox femur, prox humerus, prox tibia, and pubic rami)

Anterior Cerebral Artery Stroke

-Unilateral Weakness -Sensory Loss in Lower Limbs -Memory Impairment -Aphasia -Apraxia

Diaphragmatic breathing exercises are recommended for?

-Ventilation improvement -Gas exchange facilitation in lung tissues -To induce relaxation *NOT dyspnea management

What do rhonchi sound like?

-low pitched wheeze during both inspiration and expiration -Due to obstructed inflammatory secretions or liquids -can disappear with coughing so secretions may be involved

Wagner Ulcer Grade Classification Scale

0 - no open lesion, but may possess pre-ulcerative lesions; healed ulcers; presence of bony deformity 1 - superficial ulcer not involving subcutaneous tissue 2 - deep ulcer with penetration through the subcutaneous tissue; potentially exposing bone, tendon, ligament or joint capsule 3 - deep ulcer with osteitis, abscess, or osteomyelitis 4 - gangrene of digit 5 - gangrene of foot requiring disarticulation

Glasgow Coma Scale (GCS)

1-5 = Patient's verbal response 1-6 = Motor response 1-4 = Ability to open their eyes *Opens eyes spontaneous = 4 *Opens eyes with speech = 3 *Open eyes response to pain = 2 *Does not open eyes = 1

According to the ADA standards, hallways should be how many inches for one wheelchair to pass? What about two?

1. 36 inches 2. 60 inches (*this is the minimal)

The four steps for EBP?

1. A clinical problem is identified, and an answerable research question is formulated. 2. A systematic literature review is conducted in order to collect the best. evidence. 3. The research evidence is summarized and critically analyzed. (During this step, the type/design of the study is identified.) 4. The research evidence is synthesized and applied to clinical work.

Gillet's test

1. Patient is standing. Place your thumb of your hand under PSIS of limb to be tested and place your other thumb on center of sacrum at same level as thumb under PSIS. 2. Ask patient to FLEX hip and knee of limb being tested as if bringing their knee to chest. 3. Assess movement of PSIS via comparison of positions of your thumbs. Make sure your eyes are level with your thumbs. PSIS should move in an INFERIOR direction (+) TEST: no identified movement of PSIS as compared to sacrum

Trans-theoretical Model

1. Pre-contemplation 2. Contemplation 3. Preparation 4. Action 5. Maintenance

A physiotherapist is educating parents of a child about Legge Perthe Calve disease. What 'initial' sign of the disease the physiotherapist must warn the parents about?

1. Presence of a limp 2. Dragging of the leg 3. Slight atrophy of the thigh muscle 4. Later = antalgic gait + positive trendelenburg *Onset is usually insidious, course is prolonged over a period of several years

What is the standard measurement for a hemisphere-wheelchair?

17.5 inches

What is the height for a standard junior wheelchair?

18.5 inches

Typical two-point discrimination values for the hand

2-4 mm = fingertips 4-6 mm = dorsal of the fingers 8-12 mm = palm 20-30 mm = dorsum of the hand *Testing is good for children 7 years of age or older

What is the height for a standard wheelchair?

20 inches

What is the loose-packed position of the glenohumeral joint?

40-55 degrees of abduction and 30 degrees of horizontal adduction without rotation

What is a Swan-Ganz catheter?

A Swan-Ganz Catheter is a catheter inserted through vessels into the right side of the heart. It measures central venous pressure (CVP), pulmonary artery pressure (PA), and pulmonary capillary wedge pressure (PCWP). Swan-Ganz Catheters do not measure peripheral heart rate or ejection fraction. Cardiac catheterization may be used to determine ejection fraction.

C-bar splint

A hand-finger splint used to promote thumb abduction and opposition. The splint facilitates improved hand function when weakness exists Used following surgery to promote tissue healing and provide immobilization of the thumb

What is a positive Brudzinski's sign?

A positive Brudzinski's sign occurs when a patient placed in supine experiences pain with passive flexion of theneck. Passive neck flexion will also produce hip and knee flexion since the patient attempts to decrease neural tension caused by the progression of inflammation within the meninges. Brudzinski's sign is a common screening tool used when meningitis is suspected.

What is a positive Kernig sign?

A positive Kernig's sign is characterized by pain that is present when a patient is positioned in supine and cannot tolerate a combination of passive hip flexion and knee extension. The pain results from inflammation of the meninges within the spinal cord. Kernig's sign is a common screening tool used when meningitis is suspected.

Guillain-Barre syndrome

A temporary inflammation and demyelination of the myelin sheaths of peripheral nerves. Guillain-Barre syndrome results in motor weakness in a distal to proximal progression, sensory impairment, and possible respiratory paralysis. Since Guillain-Barre syndrome is a lower motor neuron disease and spasticity is not part of the clinical presentation, use of the MAS would be unlikely.

Braden Scale

A tool for predicting pressure ulcer risk 6 Risk Factors 1. Sensory perception 2. Level of mobility 3. Exposure to moisture 4. Activity level 5. Nutrition status 6. Exposure to friction and shear -All risk factors are scored on a scale from 1-4, though "exposure to friction and shear" is scored on a scale from 1-3, with a total possible score of 23 *Higher = Better

Walking is an excellent treatment option for a physical therapist to utilize for patients of all ages to improve fitness and cardiovascular health. The energy cost of walking is commonly measured as the normalized rate of oxygen consumption (mL/kg/min) or efficiency relative to distance traveled (mL/kg/m). What is the average oxygen rate for comfortable walking? A. 12 mL/kg x min B. 13 mL/kg x min C. 5.5 mL/kg x min D. 10 mL/kg × min

A. 12 mL/kg x min *The average oxygen rate for comfortable walking is 12/kg x min. The metabolic cost of walking averages 5.5 kcal/min on level surfaces; this may vary widely depending on speed of walking, body weight, gradient of surface, etc.

The medical record indicates a patient has a 15 degree extension lag. Which goniometric readings would be MOST consistent with this entry? A. 15-100 degrees active range of motion; 0-100 degrees passive range of motion B. 15-100 degrees passive range of motion; 0-100 degrees active range of motion C. 15-120 degrees passive range of motion; 15-135 degrees active range of motion D. 15-135 degrees active range of motion; 15-135 degrees passive range of motion

A. 15-100 degrees active range of motion; 0-100 degrees passive range of motion *An extension lag is characterized by passive range of motion being greater than active range of motion. The most common rationale for an extension lag would be weakness of the quadriceps or inhibition by pain.

An A1C test provides information about blood sugar levels over the last two to four months. Which of the following options is NOT a normal A1C value? A. 5.7% B. 5.6% C. 5.5% D. 2.1%

A. 5.7% A1C Test: -Normal adult = Less than 5.6% -Pre-diabetes = 5.7-6.4% -Diabetes = 6.5% or higher

Physical therapists can utilize biofeedback to treat patients who suffer from stress incontinence. In this context, which of the following options BEST describes biofeedback? A. A pelvic floor intervention that utilizes pressure recordings to support active contractions B. A pelvic floor muscle exercise that strengthens the muscles by actively shutting off the flow of urine C. A pelvic floor intervention that reeducates the muscles of patients with the inability to initiate active contractions D. A pelvic floor intervention that reeducates the muscles of patients with the ability to initiate active contractions

A. A pelvic floor intervention that utilizes pressure recordings to support active contractions

Autolytic debridement sometimes must be implemented to improve a patient's wound healing. What is autolytic debridement? A. A selective natural debridement method that uses occlusive or semi-occlusive dressings to solubilize necrotic tissue only by phagocytic cells and enzymes inherent to the tissue B. A nonselective debridement method that uses physical forces to remove contaminated tissue C. A selective natural debridement method that exclusively uses occlusive dressings to solubilize necrotic tissue only by phagocytic cells and enzymes inherent to the tissue D. A selective debridement method that uses sterile instruments to remove only necrotic wound tissue and does not require anesthesia

A. A selective natural debridement method that uses occlusive or semi-occlusive dressings to solubilize necrotic tissue only by phagocytic cells and enzymes inherent to the tissue *Autolytic debridement is a selective debridement method that uses occlusive or semi-occlusive dressings to solubilize necrotic tissue only by phagocytic cells and enzymes inherent to the tissue. It is appropriate for necrotic wounds in medically stable patients as well as individuals who cannot tolerate other forms of debridement. *Mechanical debridement is a nonselective debridement method that uses physical forces to remove contaminated tissue. Sharp debridement is a selective debridement method that uses sterile instruments to remove only necrotic wound tissue and does not require anesthesia.

Autolytic debridement sometimes must be implemented to improve a patient's wound healing. What is autolytic debridement? A. A selective natural debridement method that uses occlusive or semi-occlusive dressings to solubilize necrotic tissue only by phagocytic cells and enzymes inherent to the tissue B. A nonselective debridement method that uses physical forces to remove contaminated tissue C. A selective natural debridement method that exclusively uses occlusive dressings to solubilize necrotic tissue only by phagocytic cells and enzymes inherent to the tissue D. A selective debridement method that uses sterile instruments to remove only necrotic wound tissue and does not require anesthesia

A. A selective natural debridement method that uses occlusive or semi-occlusive dressings to solubilize necrotic tissue only by phagocytic cells and enzymes inherent to the tissue *Autolytic debridement is a selective debridement method that uses occlusive or semi-occlusive dressings to solubilize necrotic tissue only by phagocytic cells and enzymes inherent to the tissue. It is appropriate for necrotic wounds in medically stable patients as well as individuals who cannot tolerate other forms of debridement. *Mechanical debridement is a nonselective debridement method that uses physical forces to remove contaminated tissue. Sharp debridement is a selective debridement method that uses sterile instruments to remove only necrotic wound tissue and does not require anesthesia.

You are a physical therapist working with a 78-year-old man who complains of urinary incontinence. Overflow urinary incontinence occurs when the bladder continuously leaks due to urinary retention and an overdistended bladder. Which of the following correctly identifies a potential cause of overflow urinary incontinence in this patient? A. Anatomical obstruction, like from an enlarged prostate B. Laxity of pelvic floor musculature C. Detrusor muscle instability D. Impaired cognition

A. Anatomical obstruction, like from an enlarged prostate

A patient with patellofemoral pain syndrome displays a 3/5 strength of quadriceps on the R LE. Which of the following gait patterns is the therapist most likely to observe? A. Anterior bending of trunk during the right heel strike B. Posterior bending of trunk during the right heel strike C. Circumduction of the right lower extremity during left stance phase D. Circumduction of the left lower extremity during right stance phase

A. Anterior bending of trunk during the right heel strike Trunk is brought forward to move the line of gravity anterior to the axis of rotation of the knee joint. This is done to reduce the need for knee extensors *Posterior = due to weakness of hip extensors

A patient recently began taking a new medication and is now reporting constipation, abdominal pain, and abdominal tenderness. After reviewing the medication list, the patient's physical therapist knows which of the following types of medication is least likely to cause constipation? A. Antibiotic B. Calcium channel blockers C. Opiate D. Antidepressant

A. Antibiotic *Antibiotics can also cause side effects can also include stomach cramps, nausea, and vomiting.

A PT treats a patient with chronic venous insufficiency in their R LE. The patient has an ankle-brachial index of 0.88 in the affected LE. Which of the following decisions regarding compression is the therapist MOST likely to make? A. Apply short-stretch bandaging during the day time B. Apply short-stretch bandaging during the night time C. Apply long-stretch bandaging during the day time D. Avoid compression therapy at this time because it is contraindicated

A. Apply short-stretch bandaging during the day time *Short stretch = high working pressure and low resting pressure --> mimicking muscle pumping mechanism -During the day, NOT night

In order to evaluate abstract skills of a patient suffering from RLA level 5 brain injury, what would you ask? A. Ask the pt to how a pencil is different from a pen B. Ask the patient to draw a figure following your instructions C. Ask the patient to spell their name backwards D. Ask the patient their place of birth

A. Ask the pt to how a pencil is different from a pen *7 Cognitive Functions -Orientation, Attention, Abstract -Memory -Calculation -Judgement -Construction

A physical therapist identifies that an infant is unable to roll from prone to supine. Which of the following reflexes would MOST interfere with the infant's ability to roll? A. Asymmetrical tonic neck B. Moro C. Positive support D. Symmetrical tonic neck

A. Asymmetrical tonic neck

A physical therapist reads in the medical record that a patient sustained a grade 1 cystocele. Which organ is MOST associated with this condition? A. Bladder B. Uterus C. Rectum D. Small bowel

A. Bladder *Cystocele refers to a forward and downward displacement of the bladder within the pelvic cavity. Cystocele can result from delivery-related muscle weakness or injury to the bladder's supporting structures. A grade 1 cystocele occurs when the bladder drops or sags only a short way into the vagina.

A physical therapist works with a patient with an obstructive lung disease characterized by permanent widening and distortion of the medium-sized airways. The patient presents with cough, copious amounts of purulent sputum, hemoptysis, and recurrent pulmonary infections. These signs and symptoms are MOST characteristic of which of the following pulmonary conditions? A. Bronchiectasis B. Idiopathic pulmonary fibrosis C. Pulmonary edema D. Emphysema

A. Bronchiectasis *Obstructive lung diseases are diseases of the respiratory tract that produce obstruction to airflow, which can affect the mechanical function and gas exchanging capability of the lungs. Symptoms of obstructive lung disease commonly include chronic cough, mucus expectoration, wheezing, and dyspnea on exertion. *Bronchiectasis is a condition characterized by permanent, abnormal dilation and distortion of the bronchi and bronchioles that is caused by destruction of the muscular and elastic components of the bronchial walls. Common clinical presentation for this disease includes cough, copious mucopurulent sputum, fetid breath, recurrent pulmonary infections, and recurrent hemoptysis. *Emphysema is a form of chronic obstructive pulmonary disease characterized by the abnormal and permanent enlargement of the distal airways (i.e., alveoli), not the bronchioles. Symptoms of emphysema worsen with the progression of the disease and include a persistent cough, wheezing, difficulty breathing especially with expiration, and an increased respiration rate.

Which of the following is a known risk factor of lymphedema? A. Cardiac disease B. Lipidema C. Lymphangitis D. Lymphadenopathy

A. Cardiac disease

A physical therapist evaluates a patient using the Modified Ashworth Scale and documents a rating of 3. Which of the following conditions would MOST likely result in a patient exhibiting a rating of 3 on this scale? A. Cerebral palsy B. Down syndrome C. Duchenne muscular dystrophy D. Guillain-Barre syndrome

A. Cerebral palsy

A physical therapist evaluates a patient using the Modified Ashworth Scale and documents a rating of 3. Which of the following conditions would MOST likely result in a patient exhibiting a rating of 3 on this scale? A. Cerebral palsy B. Down syndrome C. Duchenne muscular dystrophy D. Guillain-Barré syndrome

A. Cerebral palsy

You are working with a patient in the hospital who has recently been admitted for severe GERD symptoms. During a walking session with the patient, the patient begins coughing up sputum with blood in it. Which test would be the BEST option to assess the patient's lungs? A. Chest X-ray (CXR) B. Compound Tomographic (CT) scan C. Ventilation-perfusion (V/Q) scan D. PET scan

A. Chest X-ray (CXR) *A Chest X-ray (CXR) is a radiographic test that uses two-dimensional radiographic film and can detect a change in the patient's pulmonary parenchyma or the presence of abnormal material such as blood. Even though this patient was admitted for GERD symptoms, their new symptoms would indicate the need for an imaging test for their lungs to determine if blood is present. *A Compound Tomographic (CT) scan is a radiographic test that uses a computer-generated image of a cross-sectional plane of the patient's body. *A ventilation-perfusion (V/Q) scan is a radiographic test that has the ability to match the lung's ventilation pattern to the perfusion pattern. *A PET scan is performed after the patient inhales or is injected with a radioactive tracer: the resulting images are often used in cancer diagnosis and treatment.

Radiographic tests are a common way to assess a patient's pulmonary system. Which of the following tests uses two-dimensional radiographic film that can expose a change in the patient's pulmonary parenchyma? A. Chest x-ray (CXR) B. Compound tomographic (CT) scan C. Ventilation-perfusion (V/Q) scan D. PET scan

A. Chest x-ray (CXR)

The normal range for urine magnesium serum is between 1.8 and 2.4 mg/dL. Hypomagnesemia will result if a patient's levels dip below 1.8 mg/dL. What is an example of one cause of hypomagnesemia? A. Chronic pancreatitis B. Addison's disease C. Hypothyroidism D. Diabetic acidosis

A. Chronic pancreatitis *Other causes associated with hypomagnesemia include hemodialysis, hepatic cirrhosis, hypoparathyroidism, severe burns, blood transfusions, and chronic renal diseases. *Signs of hypomagnesemia include confusion, hyper-irritability, and leg cramps. *Diabetic acidosis, Addison's disease, and hypothyroidism are all causes associated with hypermagnesemia

Infection control is a crucial part of a physical therapist's job. What is not a well-recognized aspect of standard precautions? A. Clean and sterilize all surfaces in direct contact with the patient B. Plastic disposable liners should be used for hydrotherapy tanks C. Emptied and clean hydrocollator tanks every two weeks D. Use only EPA-registered cleaning products

A. Clean and sterilize all surfaces in direct contact with the patient *Only some of these surfaces and instruments need to be fully sterilized.

Hyperkalemia occurs when the blood has an increased concentration of potassium ions. Which of the following options is caused by hyperkalemia? A. Decreased rate and force of contraction B. Ventricular fibrillation C. Arrhythmias D. Cardiac arrest

A. Decreased rate and force of contraction

A patient presents with peripheral nerve pathology. When they attempt to perform ankle dorsiflexion, they experience symptoms in their tibialis anterior muscle. Which of the following nerves is MOST LIKELY involved? A. Deep fibular B. Tibial C. Lateral plantar D. Medial plantar

A. Deep fibular

The medical record indicates that a patient being treated for a low back strain is taking Lasix for the management of heart failure. During ambulation activities, the patient complains of lightheadedness, fatigue, and weakness. The therapist measures the patient's vital signs and identifies an irregular pulse. What is the MOST likely explanation? A. Dehydration B. Angina C. Hypernatremia D. Renal failure

A. Dehydration *Heart failure is a progressive condition in which the heart cannot maintain a normal cardiac output, which is required to meet the body's demands for blood and oxygen. Heart failure often develops after other conditions have damaged or weakened the heart and can be acute or chronic. Lasix is a diuretic that is prescribed to decrease circulating blood volume, thereby decreasing preload

A physical therapist must be especially aware of contraindications and precautions for their new patient who is undergoing cancer treatment. What is not a known contraindication for heating modalities, such as paraffin and hot packs? A. Delayed wound healing B. Decreased pain sensitivity C. Tumor site D. Direct application to dysvascular tissue

A. Delayed wound healing *Delayed wound healing is a contraindication for the use of cryotherapy (ice packs, etc), not heating agents. Heat should not be applied directly to dysvascular tissue (tissue that has been exposed to radiation therapy) or the tumor site. It should also not be used with patients who have decreased pain sensitivity.

A physical therapist must be especially aware of contraindications and precautions for their new patient who is undergoing cancer treatment. What is not a known contraindication for heating modalities, such as paraffin and hot packs? A. Delayed wound healing B. Decreased pain sensitivity C. Tumor site D. Direct application to dysvascular tissue

A. Delayed wound healing *Delayed wound healing is a contraindication for the use of cryotherapy (ice packs, etc), not heating agents. Heat should not be applied directly to dysvascular tissue (tissue that has been exposed to radiation therapy) or the tumor site. It should also not be used with patients who have decreased pain sensitivity.

If a patient has pruritus, it means that his skin is itching. Of the following conditions, which one MOST often causes pruritis? A. Diabetes B. Venous obstruction C. Hypothyroidism D. Thyroid deficiency

A. Diabetes *Hypothyroidism signs are dry skin or hypohidrosis

A physical therapist works with a 47-year-old male diagnosed with multiple sclerosis. After approximately 20 minutes of a 30 minute therapy session focusing on mobility and activities of daily living, the patient reports feeling extremely fatigued. The therapist promptly takes the patient's vital signs and concludes they are within normal limits. What is the MOST appropriate therapist action? A. Discontinue the treatment session B. Modify the intensity of the therapy session C. Complete a formal reassessment of the patient D. Contact the referring physician to discuss the patient's condition

A. Discontinue the treatment session *The therapist should not work the patient to the point of overexertion to avoid injury and exacerbation of symptoms. If the patient is reporting extreme fatigue, the treatment session should be discontinued.

What signs and symptoms should a physical therapist not expect to see in a patient who has stage 2 lymphedema? A. Edema decreases with overnight elevation of the limb B. Scar tissue forms C. Edema is non-pitting D. Skin begins to toughen

A. Edema decreases with overnight elevation of the limb * Stage O: At risk but asymptomatic; this is also called the latent or pre-clinical stage. *Stage 1: Visible swelling; pitting edema that resolves with elevation; reversible; Stemmer's sign is negative. *Stage 2: Increase in swelling; elevation does not reduce swelling; spontaneously irreversible; Stemmer's sign is positive. *Stage 3: Elephantiasis; fibrotic deep skin folds; possible changes to skin color; changes may limit mobility.

A six-year-old patient presents with recurring episodes of seizure-like activity and the healthcare team is attempting to determine a medical diagnosis. Which of the following diagnostic procedures provides structural brain disease information regarding seizures? A. Electroencephalography B. Electromyography C. MRI D. Positron emission tomography

A. Electroencephalography * Electroencephalography (EEG) provides structural brain disease information regarding seizures. This procedure is conducted by recording the patient's ongoing electrical brain activity, which appears as waves. *Magnetic Resonance Imaging (MRI) scans allow three-dimensional imaging of the brain and are helpful in locating and monitoring tumors. *Electromyography is utilized for diagnosing primary muscle disease. *Positron Emission Tomography (PET) scans can be used for physiological mapping and brain metabolism analysis.

A physical therapist is providing therapeutic exercise to a 70-year-old male patient in a skilled nursing facility. During the session, the patient begins to complain of persistent mid-back pain that does not change with rest or changing position. The physical therapist decides to stop the session early and notify the patient's nurse because they believe the patient is experiencing visceral referred pain. Which of the following pain origins is the MOST likely in this situation? A. Esophagus B. Diaphragm and pericardium C. Pancreas D. Colon

A. Esophagus *Regarding gastrointestinal pain patterns, visceral pain from the esophagus can refer to the mid-back. This is important because many visceral conditions can present as musculoskeletal pain and may go unnoticed. The physical therapist must understand referred visceral pain for differential diagnosis. A patient referred to physical therapy for intermittent mid-back pain may not have a musculoskeletal condition at all, but instead may require treatment for a serious underlying condition. *Visceral pain from the diaphragm or pericardium can refer to the shoulder. *Visceral pain from the gallbladder, pancreas, or heart can refer to the scapular and mid- back regions. Visceral pain from the appendix, colon, or pelvic viscera can refer to the lower back, pelvis, or sacrum.

A pregnant woman is presented with the complaint of pain the lower back, and hip radiating to the groin. The patient also has hypermobility of the SIJ and is diagnosed with sacroiliac dysfunction. Which of the following exercises is the best for this patient? A. External stabilization exercises B. Kegel exercises C. Abs workout D. Single leg standing with eyes closed

A. External stabilization exercises *Pts with SIJ pain and hypermobility can benefit from external stabilization exercises like hip abduction strengthening, bridging, and the use of an SIJ support belt

When a therapist applies thermotherapy locally, this serves to decrease and increase various physiological responses across the body. How does thermotherapy affect cellular oxidation? A. For every 10 degrees Celsius of applied heat, the rate of cellular oxidation increases by 2-3 times. B. For every 5 degrees Celsius of applied heat, the rate of cellular oxidation decreases by 2-3 times. C. For every 5 degrees Celsius of applied heat, the rate of cellular oxidation increases by 2-3 times.

A. For every 10 degrees Celsius of applied heat, the rate of cellular oxidation increases by 2-3 times.

A physical therapist works with a patient on the postpartum unit of acute care hospital. The patient has a public symphysis separation. Which of the following interventions would be the MOST appropriate prior to discharge? A. Gait training with an assistive device B. Abdominal strengthening exercises C. Lumbar stabilization exercises D. Progressive resistive exercises with elastic tubing

A. Gait training with an assistive device *Pubic dysfunction results in pain during gait, weight bearing activities, and abduction fo the LE *Lumbar Stabilization = 6 to 12 week rehab *D = resistive exercises with elastic tubing may be indicated for this patient in the subacute phase

A physical therapist is grading a patient's functional balance. The physical therapist identifies that the patient can pick up items off the floor while maintaining his balance, and he is willing to accept moderate challenges. However, he cannot easily shift his weight at full range in all directions. What grade will the physical therapist MOST LIKELY give this patient? A. Good B. Normal C. Fair D. Poor

A. Good

What graft allows earlier rehabilitation? A. Hamstring grafts B. Patella tendon grafts

A. Hamstring grafts -Have fewer postoperative symptoms -Greater return to the pre-injury level of activity -More expensive than patella tendon grafts *Patella tendon grafts = faster healing time + less expensive + better at preserving graft tension postoperatively than hamstring

A physical therapist analyzes the gait of a child with spastic diplegia cerebral palsy in the school setting. Based on this diagnosis, which muscle group is MOST likely to be shortened? A. Hip adductors B. Knee extensors C. Hip abductors D. Ankle dorsiflexors

A. Hip adductors *It is classified by extremity involvement and muscle tone. Spastic diplegia describes an upper motor neuron lesion in the motor cortex of the cerebrum involving the trunk and lower extremities with the upper extremities affected to a lesser degree.

A patient is gait training with a patella tendon-bearing transtibial prosthesis. Upon inspection of the residual limb, the PT notes slight redness over the proximal patella tendon and inferior border fo the patella as well as on the distal anterior residual limb. What is the MOST appropriate therapist action? A. Increase the number of sock ply B. Decrease the number of sock ply C. Continue gait training with frequent reassessment D. Discontinue gait training until prosthetic modifications are made

A. Increase the number of sock ply

A physical therapist attempts to slow the progression of scoliosis and maintain respiratory function in a child with Duchenne muscular dystrophy by having the patient stand and ambulate with the spine in a slightly extended position. Which of the following results from maintaining the spine in slight extension? A. Increased weight bearing through the facet joints B. Increased weight bearing through the sacroiliac joint C. Increased weight bearing through the anterior vertebral body D. Increased weight bearing through the transverse processes

A. Increased weight bearing through the facet joints

Clinicians can determine a patient's total lung capacity by calculating all the air in the thorax while a patient completes their biggest inhale. What formula is used to calculate the total lung capacity? A. Inspiratory reserve volume + Tidal volume + Expiratory reserve volume + Residual volume B. Inspiratory reserve volume + Tidal volume + Expiratory reserve volume C. Inspiratory reserve volume + Tidal volume D. Tidal volume + Residual volume

A. Inspiratory reserve volume + Tidal volume + Expiratory reserve volume + Residual volume *Inspiratory reserve volume (IRV) + Tidal volume (TV) = Inspiratory capacity (IC) *Inspiratory reserve volume (IRV) + Tidal volume (TV) + Expiratory reserve volume (ERV) = Vital capacity (VC) *Expiratory reserve volume (ERV) + Residual volume (RV) = Functional residual capacity (FRC)

A seven-month-old infant is able to initiate rolling from supine to prone when playing on a level floor surface. Which scenario would MOST likely be associated with this developmental milestone? A. Integration of the tonic labyrinthine reflex B. Persistence of the asymmetrical tonic neck reflex C. Integration of the rooting reflex D. Persistence of the Galant reflex

A. Integration of the tonic labyrinthine reflex *If a reflex persists after the period of integration, there is typically an associated delay with motor skills

What information about the pia mater is incorrect? A. It forms the cerebral falx. B. It forms the tela choroidea of the ventricles. C. It is a thin, vascular membrane. D. It covers the surface of the brain.

A. It forms the cerebral falx.

What phrase regarding the Functional Gait Assessment is false? A. It has moderate evidence for patients with acute neurologic conditions. B. It has strong evidence for patients with chronic neurologic conditions. C. It has moderate evidence for chronic progressive neurologic conditions. D. It examines balance while walking.

A. It has moderate evidence for patients with acute neurologic conditions

A PT is performing gait analysis of a 7 year old boy with bare feet. The PT notices that his patient has clawed toes during gait. Which if the following statements is NOT TRUE of this gait deviation? A. It may be caused by the weak or delayed contraction of the DFs B. It occurs during the stance phase of gait C. It may be caused by spastic toe flexors D. It may be caused by a hyperactive plantar grasp reflex

A. It may be caused by the weak or delayed contraction of the DFs *Clawed toes occur during the stance phase of gait

A physical therapist is performing gait analysis of a 7-year-old boy with bare feet. The therapist notices that his patient has clawed toes during gait. Which of the following statements is TRUE of this gait deviation? A. It occurs during the stance phase of gait. B. It may be caused by the weak or delayed contraction of the dorsiflexors. C. It may be caused by weakened toe flexors. D. It may be caused by an underactive plantar grasp reflex

A. It occurs during the stance phase of gait.

A new physical therapy patient is a woman who just had a mastectomy and has well-controlled lymphedema throughout the upper extremity. In her spare time, she enjoys exploring states with tropical climates. As it pertains to lymphedema management, what is not a good precaution for this woman to follow during her travels? A. Keep the limb in dependent position during the flight due to changes in air cabin pressure B. Avoid traveling in extremely hot temperatures C. Avoid insect bites D. Wear a well-fitting compression garment during the flight

A. Keep the limb in dependent position during the flight due to changes in air cabin pressure *Precautions should be taken for individuals with lymphedema when traveling. It is not appropriate to keep the limb in dependent position during the flight. Instead, the patient should keep the limb elevated during the flight While wearing a compression garment to help control the edema into the limb. *Patients should also avoid extremely hot or cold temperature climates as well as factors that could cause infection, such as insect bites. This is especially important in tropical and subtropical climates, as filariasis (a parasitic infection of the lymphatic system) is a common cose of secondary lymphedema in these regions.

A physical therapist treats a patient admitted to an acute care hospital with congestive heart failure. Which medication would MOST likely be prescribed for this patient? A. Lanoxin (digoxin) B. Coumadin (warfarin) C. Zocor (simvastatin) D. Nitrostat (nitroglycerin)

A. Lanoxin (digoxin) *Lanoxin (digoxin) is a positive inotropic agent that increases the force and velocity of myocardial contraction, slows the heart rate, decreases conduction velocity through the atrioventricular node, and decreases the degree of activation of the sympathetic nervous system

During examination, it is observed that when light is shone on the R eye, both pupils constricted but when light is shone in the L eye pupillary contraction is not observed in both eyes. Which structure is MOST likely to be impaired? A. Left II Cranial Nerve B. Left III Cranial Nerve C. Right II Cranial Nerve D. Right III Cranial Nerve

A. Left II Cranial Nerve Light reflex goes through 2 pathways: -Sensory = CN 2 -Motor = CN 3 (pupillary constriction)

A physical therapist has been seeing a particular client for the past several months. One month ago, they had a CVA. When they returned to treatment, the therapist noticed that they had a new tendency to overestimate their own abilities. What might this change signify? A. Left hemiplegia B. Pseudobulbar palsy C. Right hemiplegia D. Lateral medullary syndrome

A. Left hemiplegia *Left hemiplegia can result in behavior changes such as increased impulsivity, poor judgment and safety, overestimation of one's own abilities, and underestimation of problems.

A physical therapist has been seeing a particular client for the past several months. One month ago, they had a CVA. When they returned to treatment, the therapist noticed that they had a new tendency to overestimate their own abilities. What might this change signify? A. Left hemiplegia B. Pseudobulbar palsy C. Right hemiplegia D. Lateral medullary syndrome

A. Left hemiplegia *Left hemiplegia occurs in patients with lesions of the right hemisphere. Left hemiplegia can result in behavior changes such as increased impulsivity, poor judgment and safety, overestimation of one's own abilities, and underestimation of problems. *Pseudobulbar palsy is a deficit of cranial nerve function, common with brainstem and vertebrobasilar strokes. Right hemiplegiaoccurs in patients with lesions of the left hemisphere and can result in behavioral changes such as increased caution, hesitancy, and insecurity. Lateral medullary syndrome (Wallenberg's syndrome) is the occlusion of vertebral, posterior inferior cerebellar, or basilar arteries.

A physical therapist is working with a 25-year-old female patient in an outpatient clinic for low back pain. During the session, the patient complains of heartburn and asks the therapist for advice on management. In addition to referral to the patient's primary care physician, which of the following is the MOST appropriate advice? A. Limit the use of back supports B. Eat more dark chocolate C. Eat more citrus fruits D. Drink more coffee

A. Limit the use of back supports *Back supports can cause abdominal pressure and may increase symptoms of heartburn. Although this may be counterintuitive because the patient is being seen for low back pain, it may also be very relevant because the odds of the patient using back support to manage symptoms are higher in this scenario. *Heartburn is often triggered by the ingestion of certain foods, including peppermint, fatty foods, citrus, chocolate, alcohol, coffee, and caffeine.

A physical therapist is conducting an evaluation on a 65-year-old female patient in a skilled nursing facility. During the evaluation, the patient reports that she has been experiencing hand pain and decreased tolerance for cold in her hands. Of the following options, which is the BEST test for the therapist to perform? A. Modified Allen test B. Bunnel-Littlertest C. Phalen's test D. Eichhoff's test

A. Modified Allen test *The modified Allen test is a special wrist and hand test that assesses vascular compromise. It is performed by first identifying the radial and ulnar arteries at the wrist, then having the patient open/close their fingers quickly several times, and then making a closed fist. Then the therapist compresses the ulnar artery and has the patient open their hand. The therapist releases compression on the artery and observes for vascular filling, and then performs the same procedure with the radial artery. *A positive finding of abnormal filling of blood within the hand indicates vascular compromise. *The Bunnel-Littler test identifies intrinsic tightness at the PIP joints. Phalen's test assesses carpal tunnel compression of the median nerve. *Eichhoff's test identifies De Quervain'stenosynovitis.

A PT treats a pt with hypoparathyroidism. Which of the following symptoms would MOST likely be associated with the pt's medical condition? A. Muscle cramps B. Weight loss C. Heat intolerance D. Moon-shaped face

A. Muscle cramps *Can commonly be observed in the hands/feet *Decreased calcium level in the blood *Increased levels of phosphorus in the blood *Lethargy, paresthesias, cognitive deficits, cardiac arrhythmias, seizures, tetany, muscle pain, and cramps

A physical therapist is using a tilt table to treat a new patient. How long should the tilted position be maintained? A. No more than 45 minutes B. At least 30 minutes C. No more than 90 minutes D. At least 60 minutes

A. No more than 45 minutes *Once the patient gets used to this treatment, they may maintain the position for 30-60 minutes

A PT interviews a pt referred to physical therapy with a diagnosis of chronic bronchitis. What finding obtained during the pt interview is MOST closely associated with the patient's medical diagnosis? A. Patient is a smoker B. Patient has a sedentary lifestyle C. Patient has numerous allergies D. Patient has severe scoliosis

A. Patient is a smoker *Smoking = -Increase hypersecretion and hypertrophy of mucus-producing cells of the larger bronchi -Decreased alveolar ventilation -Potential hypoxia and acidosis

A physical therapist administers strength testing after observing a patient exhibit a posterior trunk lean during gait. Which testing procedure would be MOST anticipated based on the observed finding? A. Patient positioned in prone; downward pressure is applied to the posterior surface of the distal thigh B. Patient positioned in sidelying; downward pressure is applied to the lateral ankle of the top leg C. Patient positioned in prone; therapist passively brings the patient's heel towards their buttocks D. Patient positioned in prone with knees flexed to 90 degrees; therapist medially rotates the hip

A. Patient positioned in prone; downward pressure is applied to the posterior surface of the distal thigh *Posterior trunk lean during gait is also known as gluteus maximus gait. This gait deviation is characterized by a posterior trunk lean between heel strike and foot flat and often occurs due to weakness of the gluteus maximus. *The described scenario is the testing procedure for Craig's test which is used to assess the degree of femoral anteversion. Specifically, the therapist medially and laterally rotates the hip with the patient in prone with the knee flexed to 90 degrees. This test does not provide information about gluteus maximus strength.

You are working with a 50-year-old male patient in the hospital when they begin complaining of burning epigastric pain that is worsened with position changes. The patient also notes that their symptoms are worse when they haven't eaten in several hours. To be safe, you refer the patient for a gastroenterology evaluation. What is the MOST likely condition that you have referred the patient for? A. Peptic ulcer disease B. Appendicitis C. Diverticular disease D. Lower back pain

A. Peptic ulcer disease *Peptic ulcer disease is a stomach disorder caused by ulcerative lesions on the upper gastrointestinal tract. Peptic ulcer disease may cause burning, gnawing, and cramping epigastric pain; this symptom is aggravated by position changes and the absence of food in the stomach. *Appendicitis may cause lower right quadrant pain. Diverticular disease may cause a patient to experience lower left quadrant pain Inflammatory bowel disease may result in lower back pain.

The examiner notices that the patient's skin contains purple hemorrhagic spots on it. Which type of skin trauma is this? A. Petechiae B. Ecchymosis C. Abrasion D. Laceration

A. Petechiae *Petechiae is a skin trauma in which the skin contains purple hemorrhagic spots. *Ecchymosis is a skin trauma in which the skin has bluish discoloration due to the extravasation of blood.

A physical therapist instructs his patient to lie supine. He then passively positions the patient's left foot lateral to the right knee. Which special hip test is he MOST LIKELY about to perform? A. Piriformis test of the left leg B. FABER test of the right leg C. Piriformis test of the right leg D. FADDIR/FADIR test of the left leg

A. Piriformis test of the left leg

When examining the patient, the examiner notices that the patient's skin is moist. Of the following options, which condition is MOST LIKELY to present with this symptom? A. Pneumonic crisis B. Late-stage diabetes mellitus C. Ichthyosis D. Hyperthyroidism

A. Pneumonic crisis *Moist skin may indicate a pneumonic crisis, fever, or recent exercise or hot drink ingestion. *Dry skin, or hypohidrosis, can be a sign of late-stage diabetes mellitus, ichthyosis, or hyperthyroidism

A physical therapist is grading a patient's functional balance. The patient is unable to move without loss of balance and requires handhold support to maintain a static position. What functional balance grade should they receive? A. Poor B. Fair C. Limited D. Absent

A. Poor

A PT works on transfer training with a patient who is currently taking a calcium channel blocker medication. Which side effect is MOST likely to occur during the session secondary to the use of this medication? A. Postural hypotension B. Tachycardia C. Sedation D. Cardiac arrhythmias

A. Postural hypotension *Calcium channel blocker agents = decrease calcium = diminished myocardial contraction, vasodilation, decreased oxygen demand of the heart. Helps with: -Hypertension -Angina pectoris -Arrhytmias -Congestive HF

A patient demonstrates difficulty ambulating due to bilateral knee pain. The patient reports the onset of pain was sudden and seemed to occur after a recent episode of the flu. Radiography reveals evidence of calcium deposits in the knee joints. Which medical condition is MOST consistent with the patient's clinical presentation? A. Pseudogout B. Gout C. Polymyalgia rheumatica D. Systematic lupus erythematosus

A. Pseudogout *Pseudogout = inflammatory condition characterized by the deposition of calcium crystals in the articular and periarticular structures *Radiographic studies and laboratory testing can aid in the differentiation of these disorders

A patient demonstrates difficulty ambulating due to bilateral knee pain. The patient reports the onset of pain was sudden and seemed to occur after a recent episode of the flu. Radiography reveals evidence of calcium deposits in the knee joints. Which medical condition is MOST consistent with the patient's clinical presentation? A. Pseudogout B. Gout C. Polymyalgia rheumatica D. Systemic lupus erythematosus

A. Pseudogout *Pseudogout is an inflammatory condition characterized by the deposition of calcium crystals in the articular and periarticular structures. This condition, which most commonly affects the knee joint, often appears insidiously, after surgical procedures or after periods of illness. Radiography can be used to confirm this diagnosis. *Gout is an inflammatory condition characterized by acute pain due to deposits of urate crystals in the joint. This condition most commonly affects the first metatarsophalangeal joint. *Polymyalgia rheumatica is a systemic inflammatory condition that is experienced primarily in older adults. Erythrocyte sedimentation rate (ESR) is high and the pain is most commonly experienced in the pelvic and shoulder girdles. *Systemic lupus erythematosus is an autoimmune condition characterized by numerous organ and joint issues. Typically, a patient with systemic lupus erythematosus exhibits a trademark butterfly rash and experiences joint pain primarily in smaller joints.

Which of the following techniques would be the MOST beneficial when treating a six year old child with Down syndrome based on the anticipated tonal abnormalities? A. Quick stretch B. Deep pressure C. Prolonged icing D. Neutral warmth

A. Quick stretch

A therapist is working in a home health setting with many patients undergoing cancer treatments. Therefore, he must be aware of the side effects of cancer treatments. Which type of cancer treatment is LEAST LIKELY to cause nausea, vomiting, and/or anorexia? A. Radiation B. Chemotherapy C. Immunotherapy D. Hormonal therapy

A. Radiation *Radiation is a site-specific treatment and common side effects include fatigue, radiation sickness, hair loss, localized burns, and fibrosis. A gastrointestinal side effect can include diarrhea.

What skin concern would a physical therapist most likely see in a patient who has vasomotor disturbances, bodily inflammation, or has begun taking a new medication? A. Rash B. Xeroderma C. Urticaria D. Edema

A. Rash

A patient is diagnosed with psoriasis. What are the most likely signs of the disease? A. Red, itchy patches with white powdery scales over the skin B. Erythema, swelling, and pain with the skin infection. C. Burns D. Itchy puss-filled sores.

A. Red, itchy patches with white powdery scales over the skin

A physical therapist determines that a patient needs a spinal orthosis to control lower back motion in order to treat stenosis. Which of the following spinal orthoses would the physical therapist MOST LIKELY recommend to the patient? A. Rigid lumbosacral orthosis B. Flexible lumbosacral orthosis C. Thoracolumbosacral orthosis D. Minerva orthosis

A. Rigid lumbosacral orthosis *Lumbosacral Orthoses (LSOs) control lower back motion or lumbosacral motion. Rigid SOs are used for postoperative protection, stenosis, low back pain, and spondylolisthesis. *A flexible LSO is indicated for low back pain, compression of abdominal incisions, or sacroiliac support during pregnancy. A thoracolumbosacral orthosis controls thoracic and lumbosacral motions, which would be too restrictive for the patient in this example A Minerva orthosis is a cervical orthosis and would be inappropriate for this patient.

A patient has a muscle spasm of the quadriceps. What is the likely end-feel for a knee joint? A. Rubbery B. Soft C. Firm D. Boggy

A. Rubbery

What condition of the skin cannot be passed from person to person through contact? A. Scleroderma B. Impetigo C. Cellulitis D. Warts

A. Scleroderma *Scleroderma is an autoimmune disorder. It is characterized by chronic, diffuse disease of connective tissues causing fibrosis of the skin, joints, blood vessels, and organs. It is an immune disorder and is not transmitted to another person.

A physical therapist inspects a burn obtained as a result of iontophoresis. The therapist describes the burn as an alkaline reaction. Which of the following factors would MOST likely be the cause of this type of adverse reaction? A. Sodium hydroxide forming under the cathode B. Hydrochloric acid forming under the anode C. he size of the cathode being larger than the anode D. An increase in the amount of space between the electrodes

A. Sodium hydroxide forming under the cathode *lontophoresis utilizes a continuous direct current which moves ions through the body's tissues. The direct current alters the normal pH of the skin and makes the patient susceptible to a chemical burn. The pH is a measure of the degree to which a solution is acidic or alkaline. Usually, a pH of 7.0 indicates a neutral solution. A pH of less than 7.0 indicates acidity and a pH of more than 7.0 indicates alkalinity. The normal pH of the skin is 4.0-5.0. The likelihood of a burn can be decreased by increasing the size of the cathode relative to the anode, decreasing the current density, and increasing the space between the electrodes.

A 32 year old lady with a distal radial fracture 8 months ago visits an outpatient facility. While examination it is noticed that her pronation and supination is limited to 60 degrees each and display a hard end feel. She has taken physical therapy before and her range of motion is the same. Which of the following would be the most appropriate intervention here? A. Strengthen pronators B. Grade 4 dorsal glide C. Grade 4 anterior glide D. Prolonged stretching of pronators

A. Strengthen pronators *No amount of mobilization or stretching can change the patient's range -Indiscriminate stretching/mobilization can cause hyper mobility of relative joints which would cause additional trauma and pain

A patient is evaluated in physical therapy three weeks following a lateral ankle reconstruction. Which of the following exercises for the involved lower extremity would be the MOST appropriate to include as part of this patient's home exercise program on the first session? A. Supine straight leg raise B. Ankle inversion/eversion with a resistance band C. Standing calf raise with upper extremity support D. Single leg stance with eyes open

A. Supine straight leg raise *Precautions following surgery include limitations on weight bearing status and restrictions on allowable range of motion (ROM)

A physical therapist working in home health treats a patient following total knee arthroplasty. Which position would be the MOST important for the patient to avoid when resting during the day? A. Supine with a pillow under the operative knee B. Supine with a towel roll along the lateral aspect of the femur C. Supine with a pillow under the calf of the operative knee D. Seated with the involved leg elevated and extended at the knee

A. Supine with a pillow under the operative knee *Positioning the patient in supine with a pillow under the knee will maintain the knee in a flexed position and increase the risk for a flexion contracture. Patients often utilize this position since it approximates the open packed position of the knee and therefore tends to be less painful following surgery.

A physical therapist examines the posture of a patient from a lateral view using a plumb line. Which structural condition would be MOST likely to cause the plumb line to fall further posterior to the hip joint? A. Swayback B. Lordosis C. Scoliosis D. Genu recurvatum

A. Swayback

Which of the following is true of lipidema? A. Symmetrical, bilateral swelling B. Postive Stemmer sign C. May be congenital or due to damage/injury D. Affects both men and women equally

A. Symmetrical, bilateral swelling

A burn wound consists of three zones. Which of the following BEST describes the zone of stasis? A. The burn causes cells to become injured; if untreated, cell death may occur. B. The burn causes irreversible cell injury in which cell death occurs. C. The burn causes minimal cell injury in which the cells should recover. D. The burn causes permanent cell mutation; the cells do not die.

A. The burn causes cells to become injured; if untreated, cell death may occur. *Zone of hyperemia: the burn exhibits minimal cell injury; cells will likely recover *Zone of stasis: cells are injured and may die within 24-48 hours unless they receive specialized treatment; these injuries are sensitive to infection and trauma *Zone of coagulation: cells are irreversibly injured; cell death occurs

The results of the jaw jerk reflex suggest that a patient may have an upper motor neuron lesion. Based on the testing procedure, which finding would BEST support the stated hypothesis? A. The mandible moves upward B. The mandible moves downward C. The mandible moves laterally D. The mandible remains in the testing position

A. The mandible moves upward *The jaw jerk reflex is a muscle stretch reflex used to assess the trigeminal nerve. The testing procedure requires the therapist to transmit force with the reflex hammer just below the lips with the mouth slightly open. Both the sensory and motor components of the reflex originate from the trigeminal nerve.

You are a physical therapist evaluating a student-athlete in the outpatient setting. The patient's medical chart notes that the patient has suffered a moderate Traumatic Brain Injury (TBI) after a collision on the field. Which of the following statements is LEAST LIKELY to be true of this patient? A. The patient experienced posttraumatic amnesia for 12 hours after the injury. B. The patient experienced a loss of consciousness for 12 hours after the injury. C. The patient experienced a loss of consciousness for 1 hour after the injury. D. Imaging of the patient's brain appears normal.

A. The patient experienced posttraumatic amnesia for 12 hours after the injury. *It is unlikely that a patient with a moderate TBI would experience posttraumatic amnesia for only 12 hours after the injury. If the patient had been diagnosed with a mild TBI, this would be expected. A moderate TBI is more likely to present with posttraumatic amnesia lasting over 1 day but less than 7 days. *Moderate TBls may result in a loss of consciousness for as little as 30 minutes or as much as 24 hours. The imaging of the patient's brain may appear normal or abnormal, despite the injury.

A patient's respiratory exchange ratio is measured as 1.1 during a graded exercise test that includes an analysis of expired gases. What does this indicate about the patient's current status? A. The patient is approaching their maximum aerobic capacity B. The patient is no longer able to meet their metabolic need for oxygen via anaerobic metabolism C. The patient is able to meet their metabolic need for oxygen via aerobic metabolism D. The patient is able to meet their metabolic need for carbon dioxide via anaerobic metabolism

A. The patient is approaching their maximum aerobic capacity *The respiratory exchange ratio (RER) is determined by dividing the volume of carbon dioxide produced by the amount of oxygen consumed, or VCO2/VO2. The ratio is approximately 0.8 at rest. *During intense exercise, the ratio can exceed 1.0 as the volume of expelled carbon dioxide increases from production by the working muscles and buffering of lactate in the blood, as well as more of the inhaled oxygen being used rather than being expelled.

A physical therapy clinic discusses methods to ensure the privacy of patients' health information. Which of the following actions would be acceptable according to the Health Insurance Portability and Accountability Act? A. The patient's identifier information is not removed from documentation sent to the insurance company. B. The therapist discusses the patient's plan of care with the receptionist. C. The therapist speaks to an employer about the patient's ability to return to work D. The therapist leaves the patient's medical chart on their desk at the end of the workday.

A. The patient's identifier information is not removed from documentation sent to the insurance company.

What statement regarding diabetes mellitus type 2 is incorrect? A. These patients are prone to ketoacidosis. B. The condition is characterized by a progressive decline in pancreatic insulin production. C. These patients are not usually insulin-dependent. D. The condition is characterized by inappropriate glucagon secretion.

A. These patients are prone to ketoacidosis.

Of the following options, which is a reason for a physical therapist to perform the Yergason test instead of Speed's test? A. To identify a tear in the patient's supraspinatus tendon B. To test for bicipital tendinosis C. To test for bicipital tendinopathy D. To test for SLAP lesions

A. To identify a tear in the patient's supraspinatus tendon *The Yergason test, and not Speed's test, is used to test the integrity of the transverse ligament. *Both Speed's test and the Yergason test are used to identify bicipital tendinosis, bicipital tendinopathy, and SLAP lesions.

What clinical indications might lead a physical therapist to administer a pinch grip test? A. To identify entrapment of the anterior interosseous nerve B. To identify the presence of cubital tunnel syndrome C. To identify the presence of carpal tunnel syndrome D. To identify the level of sensory innervation within the hand, correlating with functional ability to perform tasks involving grip

A. To identify entrapment of the anterior interosseous nerve

Why would a physical therapist perform the pinch grip test? A. To identify entrapment of the anterior interosseous nerve B. To identify the presence of cubital tunnel syndrome C. To identify the presence of carpal tunnel syndrome D. To identify the level of sensory innervation within the hand, correlating with functional ability to perform tasks involving grip

A. To identify entrapment of the anterior interosseous nerve *Elbow flexion = cubital tunnel syndrome *Tinel's Sign = median nerve

A 60-year-old woman presents for a physical therapy evaluation due to increased pelvic pressure and abnormal masses of tissue in the vagina. She is post-menopausal and has had three vaginal deliveries. What is the name of a pelvic floor disorder that causes tissue masses in the vagina that develop due to a bulging uterus? A. Uterine prolapse B. Enterocele C. Pelvic inflammatory disease D. Cystocele

A. Uterine prolapse *Cystocele is a pelvic floor disorder associated with the bladder pushing into the vagina *Enterocele is a pelvic floor disorder in which the peritoneal sac containing part of the small bowel pushes into the vaginal wall *Pelvic inflammatory disease (PID) is an inflammation of the upper reproductive tract involving the uterus, fallopian tubes, or ovaries

A pt post Colles fracture has moderate edema in their fingers and the dorsal of their hand and reports pain during AROM. Which of the following measurements is the MOST appropriate to accurately quantify the pt's edema? A. Volumetric B. Circumferential C. Girth D. Anthropometric

A. Volumetric *Volumetric measurements are often used to quantify the presence of edema in the wrist and hand *Circumferential = Girth *Anthropmetric = height, weight, BMI, waist-to-hip-ratio, and percentage of body fat --> compared to reference standards

A physical therapist treats a patient with a right transfemoral prosthesis. During gait training the therapist notes a medial heel whip on the right. What is the MOST likely cause of this deviation? A. Weak musculature that allows the prosthesis to rotate around the femur B. Improperly donning the socket into an externally rotated position C. An excessively firm heel cushion D. Medial orientation of the prosthetic foot

A. Weak musculature that allows the prosthesis to rotate around the femur

Prosthetic training requires a thorough understanding of the biomechanics of gait in addition to strong observation skills to discern the contributing factors associated with common gait deviations. Deviations may be secondary to intrinsic (patient) or extrinsic (prosthetic) factors. A. When a residual limb is weak, there is a strong propensity for the prosthesis to move more freely around the femur. This results in rotation of the prosthesis as the patient goes through pre-swing and early swing phase on the prosthetic side and may result in a medial heel whip. B. If a patient dons the socket improperly into an externally rotated position, this can result in a lateral heel whip on the prosthetic side during pre-swing through the early swing phase. C. When the heel cushion is excessively firm, it can cause the prosthetic foot to laterally rotate during heel strike. D. When the prosthetic foot is oriented medially, it can cause a lateral heel whip during the pre-swing through early swing phase on the prosthetic side.

A. When a residual limb is weak, there is a strong propensity for the prosthesis to move more freely around the femur. This results in rotation of the prosthesis as the patient goes through pre-swing and early swing phase on the prosthetic side and may result in a medial heel whip.

A physical therapist observes a patient ambulating with a transfemoral prosthesis with a quadrilateral socket. What type of gait deviation is MOST commonly observed with this type of socket? A. Wide base of support during ambulation B. Excessive knee flexion during stance phase on the prosthetic side C. Knee instability at heel strike on the prosthetic side D. Vaulting on the non-prosthetic side

A. Wide base of support during ambulation *The total contact design of the quadrilateral socket allows for primary weight bearing through the flat posterior shelf. The anterior-posterior dimension of the socket tends to be more narrow than the medial-lateral dimension *Typically, the quadrilateral socket produces a wide base of support that results in a Trendelenburg gait on the prosthetic side. The quadrilateral socket aligns the femur into abduction as opposed to the adduction angle that exists normally. The wide medial-lateral dimension allows the socket to displace laterally during midstance and tends to decrease the efficiency of the gluteus medius muscle.

A patient recently experienced a contusion resulting from a traumatic brain injury (TBI). What classification of brain damage is this considered and what can the contusion lead to? A. primary; a hematoma B. secondary; a hematoma C. primary; a brain bruise/bleed D. secondary; hypoxia

A. primary; a hematoma

Motions in the frontal plane occur around ____________ axis

A/P axis

De Quervain's tenosynovitis involves what muscles? What test do you perform to assess it?

Abductor pollicis longus + Extensor pollicis brevis Finkelstein Test -Put thumb in palm -Ulnar Deviate

Bronchiectasis

Abnormal dilation of the bronchi with accumulation of mucus, destruction of the muscular and elastic components of the bronchial walls

Guillain-Barre Syndrome (GBS)

Acute polyneuropathy -Temporary inflammation and demyelination on the peripheral nerves Motor weakness in a distal to proximal progression, sensory impairment, and possible respiratory paralysis Disability usually peaks within two or four weeks after onset *PT = PROM, positioning, and light exercise (limit overexertion and fatigue)

What motions are most likely affected with an upper motor neuron syndrome exhibited in the shoulder spasticity?

Adduction Internal Rotation Depression

Tapotement

Also known as percussion; movements consisting of short, quick tapping, slapping, and hacking movements. technique which consists of a series of brisk blows, administered with relaxed hands following each other in rapid alternating movements. It is used to increase circulation or stimulate peripheral nerve endings.

Sacroiliac Dysfunction

Altered structural or positional relationship of the sacrum on a normally positioned ilium -Common in pregnancy or postural changes and ligamentous laxity Treatment = use of external stabilization support belt to support the SI joint

Erb's Palsy

Arm weakness or paralysis -Caused by damage to the brachial plexus Signs: -Shoulder IR -Adduction -Finger flexion

Hypokalemia, hypermagnesemia, and hypomagnesemia can cause what?

Arrhythmias

What are long stretch bandages for?

Assist the calf muscles to pump blood to the heart

While conducting a submaximal exercise tolerance test, the patient's ECG monitor displays a PR interval of 0.3 seconds followed by 0.4 second QT interval. Sinus rhythm appears normal. What is the diagnosis and how do you proceed? A. Normal ECG, continue the exercise B. 1* Heart block, continue the exercise C. 2* Heart block, call 911 D. 1* Heart block, contact physician after stopping exercise

B. 1* Heart block, continue the exercise Normal PR interval is 0.20 sec = in question AV node is prolonged

A patient is admitted in an in-patient facility with a reported brain injury due to a car accident. The patient's GCS score is as follows Eye opening - 3/4, Best Motor response 4/6, Verbal response - 4/5 This therapist is likely to suspect a post traumatic amnesia of what timeframe- ? [GCS Glasgow Coma scale] A. One day B. 3 to 4 days C. More than 7 days D. One month

B. 3 to 4 days *PTA 0 - 1 days = mild brain (13-15 on GSC) *PTA >7 days = severe (less than 9 on GSC)

A physical therapist assesses a patient with suspected unilateral hearing loss using the Rinne test. With the tuning fork on the patient's skull, the patient reports that the sound disappears after 15 seconds. Assuming the patient's sense of hearing is normal, how long should the patient hear the sound when the tuning fork is later placed next to the ear? A. 15 seconds B. 30 seconds C. 45 seconds D. 60 seconds

B. 30 seconds *When performing the Rinne test, the examiner holds the vibrating tuning fork against the mastoid process until sound is no longer heard by the patient. The examiner then moves the tuning fork 1-2 centimeters away from the auditory canal until the sound is no longer heard. The duration of time is recorded in each position. The normal ratio of air conduction to bone conduction is 2:1.

When a doctor prescribes an A1C test, they are looking to gather data on a patient's blood sugar levels from the last two to four months. Depending on their A1C range, a patient may be diagnosed with prediabetes or diabetes. What A1C figure would lead a patient to be diagnosed with pre-diabetes? A. 6.5% B. 5.9% C. 7.0% D. 5.6%

B. 5.9% *An A1C test should be 5.6% or less for a normal adult. *An A1C of 5.7-6.4% would qualify as pre-diabetes. *An A1C of 6.5% or higher indicates diabetes.

What position should a patient be in to put the humeroulnar joint in the loose-packed position? A. 90-degree flexion with 5-degree supination B. 70-degree flexion with 10-degree supination C. 70-degree flexion with 5-degree supination D. Full extension and supination

B. 70-degree flexion with 10-degree supination *The close-packed position of the humeroradial joint is 90-degree flexion with 5-degree supination.

PT can teach patients who suffer from stress incontinence how to use various therapeutic interventions to alleviate their symptoms. In this context, which of the following is the BEST description of functional electrical stimulation? A. A pelvic floor muscle exercise that strengthens the muscles, and should be performed multiple times per day. B. A pelvic floor intervention that reeducates the muscles of the patients who have the inability to initiate active contractions C. A pelvic floor muscle intervention that utilizes pressure recordings to support active contractions D. A pelvic floor muscle intervention that reeducates the muscles of patients who have the ability to initiate active contractions

B. A pelvic floor intervention that reeducates the muscles of the patients who have the inability to initiate active contractions

Intermittent mechanical compression uses a pneumatic device to apply external pressure to an extremity through an inflatable appliance. Which of the following conditions is a contraindication for intermittent mechanical compression? A. Traumatic edema B. Acute inflammation C. Postmastectomy lymphedema D. Stasis ulcer

B. Acute inflammation *Acute inflammation is a contraindication for intermittent mechanical compression. Other contraindications include acute DVT, obstructed lymph or venous return, and arterial disease

A physical therapist wants to use intermittent mechanical compression on a patient after the treatment session. The therapist screens the patient for contraindications for this device prior to use. Which of the following conditions is a contraindication for intermittent mechanical compression? A. Traumatic edema B. Acute inflammation C. Postmastectomy lymphedema D. Stasis ulcer

B. Acute inflammation *Acute inflammation is a contraindication for intermittent mechanical compression. Other contraindications include acute Deep Venous Thrombosis (DVT), obstructed lymph or venous return, and arterial disease. *Intermittent mechanical compression may be used to treat subacute injuries with traumatic edema, postmastectomy lymphedema, stasis ulcers, amputations, and venous insufficiency.

Therapists use a pneumatic device to provide intermittent mechanical compression to patients who need added pressure on an extremity. What health condition would not require the use of this treatment method? A. Traumatic edema B. Acute inflammation C. Stasis ulcer D. Postmastectomy lymphedema

B. Acute inflammation *Acute inflammation is a contraindication for intermittent mechanical compression. Other contraindications include acute deep venous thrombosis (DVT), obstructed lymph or venous return, and arterial disease. *Intermittent mechanical compression may be used to treat subacute injuries with traumatic edema, postmastectomy lymphedema, stasis ulcers, amputations, and venous insufficiency.

A PT examines a patient with an acute burn to the left shoulder axilla area secondary to a spill of boiling water. What is the MOST appropriate splint for this patient? A. Shoulder sling B. Airplane splint C. C-bar splint D. Heme-arm sling

B. Airplane splint *Shoulder axilla area will tend to develop contractures toward shoulder adduction and IR *C-bar = hand-finger splint used to promote thumb abduction and opposition --> improved hand function when weakness exists (following surgery to promote tissue healing and immobilization of the thumb *Hemi-arm = provides a vertical upward force to support humerus without restricting the elbow and forearm (pt = shoulder subluxation)

A Basketball player suffered a complete achillies tendon rupture after landing abruptly on the right foot underwent a successful tendon repair surgery. Patient is in his 6th week of rehabilitation. Which of the following exercise is inappropriate here? A. Grade Ill Mobilization at the ankle B. Ankle hops in pool C. Bilateral toe raises in standing D. Unilateral stance on foam pad

B. Ankle hops in pool

While performing Dix-Hallpike test on the left side, the therapist notices a presence of sudden and persistent symptoms of BPPV. Closer examination reveals a presence of a sudden down-beating nystagmus during the test. What could be the diagnosis? A. Posterior canal, canalithiasis B. Anterior canal, cupulolithiasis C. Horizontal canal, canalithiasis D. Inferior canal, cupolithiasis

B. Anterior canal, cupulolithiasis

In order for the sternoclavicular joint to be in the close-packed position, how must the patient's arm be positioned? A. Maximum abduction and external rotation B. Arm maximally elevated C. Arm resting by side D. Arm abducted 90 degrees

B. Arm maximally elevated *The close-packed position of the acromioclavicular joint occurs with the arm abducted 90 degrees. *Both the sternoclavicular and acromioclavicular joints are in loose-packed positions when the arm is resting by the side. *Maximum abduction and external rotation of the arm result in a close. packed position of the glenohumeral joint.

A physical therapist works with a four-month-old infant. During mat activities the infant suddenly becomes unconscious. Which of the following arteries is the MOST appropriate for the therapist to palpate to assess the infant's pulse? A. Radial B. Brachial C. Popliteal D. Carotid

B. Brachial

A physical therapist discusses the process of pressure relief on a wheelchair with a patient rehabilitating from a spinal cord injury. What is the HIGHEST spinal cord level where this activity would be an 'independent' functional outcome considering the injury is complete? A. CA B. C6 C. C8 D. T1

B. C6 *Intact extensor carpi radialis, infraspinatus, latissimus dorsi, pectoralis minor, serratus anterior, pronator teres, and teres minor

A physical therapist discusses the process of pressure relief on a wheelchair with a patient rehabilitating from a spinal cord injury. What is the HIGHEST spinal cord level where this activity would be an 'independent' functional outcome considering the injury is complete? A. C4 B. C6 C C8 D. T1

B. C6 An injury at C6 will have: -Extensor Carpi Radialis -Infraspinatus -Latissimus Dorsi -Pectoralis Minor -Serratus Anterior -Pronator Teres -Teres Minor

Identify the incorrect statement about 'Complex Regional Pain Syndrome (CPRS)' A. Symptoms which are more marked distally B. CPRS symptoms progress in intensity but will not spread proximally C. Symptoms can be completely disproportionate with respect to the causative event D. CPRS can be of non-neural origin

B. CPRS symptoms progress in intensity but will not spread proximally

Hypermagnesemia occurs when the blood has an increased concentration of magnesium ions. Which of the following options is caused by hypermagnesemia? A. Ventricular arrhythmias B. Cardiac arrest C. Coronary artery vasospasm D. Sudden death

B. Cardiac arrest

Hypermagnesemia occurs when the blood has an increased concentration of magnesium ions. Which of the following CAN be caused by hypermagnesemia? A. Coronary artery vasospasm B. Cardiac arrest C. Decreased force of contraction D. Ventricular fibrillation

B. Cardiac arrest *Increased magnesium works as a calcium blocker. *Hypomagnesemia can cause coronary artery vasospasm. *Hyperkalemia can cause a decreased force of contractions. *Hypokalemia can cause ventricular fibrillation.

You are evaluating a 30-year-old female patient with complaints of tingling in their first three digits. During the physical exam, you observe muscle wasting over the thenar eminence. What condition would you MOST likely suspect, and what special test can be used to further assess the condition? A. Vascular compromise; Modified Allen test B. Carpal tunnel syndrome; Phalen's test C. de Quervain's tenosynovitis; Finkelstein's test D. FCC pathology; Ulnomeniscotriquetral dorsal glide test

B. Carpal tunnel syndrome; Phalen's test *Phalen's test is a special wrist and hand test that is conducted by having the patient flex both wrists while holding them together for one minute. Phalen's test attempts to reproduce tingling in one's hand after the distribution of the median nerve. This test indicates compression of the median nerve in the carpal tunnel. Because this patient presents with symptoms of carpal tunnel syndrome, this is the most appropriate option provided. *The modified Allen test is used to identify vascular compromise. Finkelstein's test is used to identify de Quervain's tenosynovitis. *The ulnomenisco triquetral dorsal glide test is used to identify the pathology of the TFCC.

Choose the research design that is an ex-post-facto study. A. Descriptive research design B. Causal-comparative research design C. Correlational research design D. Experimental research design

B. Causal-comparative research design *Ex-post-facto refers to the study done on facts that have already been established before the start of the research study, and that the cause or effect can not be manipulated by the researcher's involvement. *A causal-comparative study is also called an ex-post-facto study because the researcher determines the cause or effect of something that is already happened or a condition that has already occurred to the population is taken into consideration.

A patient rehabilitating from multi-trauma injuries sustained in a motor vehicle accident is not a candidate for patient-controlled analgesia. Which finding may have been MOST relevant when making this determination? A. Surgical contraindications B. Cognitive impairments C. Functional limitations D. Gastrointestinal distress

B. Cognitive impairments

A physical therapist screens a patient for contraindications prior to applying pneumatic intermittent compression to the right lower leg. Which of the following findings would be considered an absolute contraindication? A. Post-surgical edema B. Congestive heart failure C. Pitting edema D. Venous stasis ulcers

B. Congestive heart failure *Physical therapists must be knowledgeable of indications and contraindications when screening patients and considering appropriate interventions. Failure to recognize that a patient's medical history includes a contraindication to a specific intervention would be negligent and could place the patient at risk for serious harm. *The use of intermittent compression is indicated for patients who have post-surgical edema in an extremity. Edema reduction and management have been shown to improve tissue healing. *The use of intermittent compression on the extremities of patients with congestive heart failure is contraindicated since it can increase the fluid load on the failing heart. This further increases peripheral resistance and may cause a potentially life threatening situation. *The use of intermittent compression is indicated for patients who have pitting edema in an extremity. Edema reduction from intermittent compression is temporary and requires additional intervention, such as the use of compression garments, to maintain the reduction. *The use of intermittent compression is indicated for patients with venous stasis ulcers since they are typically associated with chronic venous stasis edema. Edema reduction and ongoing management have been shown to improve healing of venous stasis ulcers.

A flexor (withdrawal) reflex is a spinal level reflex that has an involuntary response to which of the following? A. Muscle stretches B. Cutaneous sensory stimuli C. Muscle contractions D. Noxious stimuli and reciprocal action of antagonists

B. Cutaneous sensory stimuli *Flexor (withdrawal) reflexes are spinal level reflexes that have an involuntary response to cutaneous sensory stimuli. These reflexes provide a protective withdrawal mechanism to remove a body part from hazardous stimuli. *Stretch (myotatic) reflexes are spinal level reflexes that have an involuntary response to muscle stretches. Inverse stretch (myotatic) reflexes are spinal level reflexes that have an involuntary response to muscle contractions. *Crossed extension reflexes are spinal level reflexes that have an involuntary response to noxious stimuli and reciprocal action of antagonists.

You are reviewing the medication list for a new patient admitted to the cardiovascular intensive care unit. As a precaution, the hospitalist has prescribed the patient. aspirin. What is the MOST likely reason for prescribing this medication? A. Decrease blood pressure B. Decrease platelet aggregation C. Control arrhythmias D. Increase blood pressure

B. Decrease platelet aggregation *Aspirin is used to prevent myocardial infarction and to decrease platelet aggregation. Aspirin works by decreasing inflammation associated with heart disease and reducing the risk of blood clot formation. *ACE inhibitors are used to reduce sodium retention in order to decrease blood pressure. Angiotensin I receptor blockers are used to block the binder of angiotensin Il at the tissue/smooth muscle level in order to decrease blood pressure. *Beta-adrenergic blocking agents are used to reduce myocardial demand, reduce blood pressure, and control arrhythmias. None of these options are used to increase blood pressure.

Hypomagnesemia cause all of the following problems except? A. Increased intracellular calcium levels B. Decreased intracellular calcium levels C. Convulsions D. Muscle cramps

B. Decreased intracellular calcium levels *Vomiting convulsions, muscle cramps, hyperreflexia, and falls

A female patient with osteopenia is placed on hormone replacement therapy by her physician. Hormone replacement therapy places the patient at GREATEST risk for which of the following conditions? A. Aneurysm B. Deep vein thrombosis C. Vertebral compression fracture D. Colon cancer

B. Deep vein thrombosis

Mr. Tom visits an in-patient facility after being diagnosed with Erb's palsy after a fall on his shoulder. Which of the following problems is Mr. Tom likely to have following Erb's Palsy? A. Difficulty in combing hair, anesthesia over shoulder, horner syndrome B. Difficulty to bring food to the mouth, weak ER, winging of lateral border of scapula C. Difficulty in unbuttoning shirt, weak abduction, flattening over humeral head D. Claw head, weak supination, elbow flexion contracture

B. Difficulty to bring food to the mouth, weak ER, winging of lateral border of scapula

A physical therapist works with a patient with a transfemoral amputation on controlled fall training. The patient ambulates with a single axillary crutch. What controlled falling technique is the MOST appropriate to teach the patient in an anterior loss of balance? A. Pull the crutch close to the body to absorb the impact B. Discard the crutch and land on slightly flexed arms C. Turn the body to absorb the fall on the back of the trunk and legs D. Land on the elbows and prosthesis to absorb the impact

B. Discard the crutch and land on slightly flexed arms *Controlled falling is a desirable training technique for patients following amputation. If they are ambulating using an assistive device, the first step is to discard it and prepare for the fall. Discarding the assistive device whenever possible allows the patient to absorb the fall on slightly flexed arms, decreasing the impact to any one specific segment of the body.

A patient with a long history of PD on L-dopa is more likely to experience which of the following at the "peak" of its dose? A. Bradykinesia B. Dyskinesia C. Dystonia D. Akathisia

B. Dyskinesia Dynamic uncontrolled or involuntary movements -Facial grimacing with twitching of the lips and tongue protrusion -Prevalent, vigorous, extensive movements in the limbs, trunk, and neck

You are working with a 76-year-old male patient in a skilled nursing facility on functional mobility training. During the session, the patient reports that they have been feeling somewhat confused and sleepy. Upon returning to the patient's room, the nurse provides recent test results indicating a sodium serum level of 120 mEq/L. Of the following, what is the MOST likely explanation for these symptoms? A. Decreased water intake B. Excessive water intake C. Hypoparathyroidism D. Vitamin A intoxication

B. Excessive water intake *The normal sodium serum level for urinary regulation is 135 to 146 mEq/L. *Other causes associated with hyponatremia include tumors and endocrine disorders. *Hyponatremia can lead to confusion, convulsions, sleepiness, and poor motor coordination.

A three-year-old child throws frequent temper tantrums, usually contrived to gain attention. The physical therapist, recognizing the child's objective, refuses to acknowledge the action. What term BEST describes this type of behavior therapy? A. Aversive conditioning B. Extinction C. Operant conditioning D. Rational emotive imagery

B. Extinction *Aversive conditioning = reduces the appeal of a behavior by associating the behavior with physical or psychological discomfort. The individual is exposed to an unpleasant stimulus while engaging in the targeted behavior. The goal would be to create an aversion to the targeted behavior. *The goal of extinction is a reduction or a loss in the strength of a conditioned response when the unconditioned stimulus or reinforcement is withheld *Rational emotive imagery is a form of intense mental practice for learning new emotional and physical habits. The behavioral technique focuses on uncovering irrational beliefs which may lead to unhealthy negative emotions and replacing them with more productive rational alternatives.

A physical therapist assesses a patient with a traumatic brain injury using the Glasgow Coma Scale. If the therapist documents the eye opening score as a three (E3), which of the following responses was observed during this portion of the assessment? A. Spontaneous eye opening B. Eyes open in response to speech C. Eyes open in response to pain D. Eyes do not open (nil response)

B. Eyes open in response to speech *The eye opening subscale is scored on a range from 1-4. A patient who can open their eyes spontaneously would be given a score of 4 (E4). *A patient who can open their eyes in response to speech would be given a score of 3 (E3). This is typically assessed by speaking to the patient using their name. *A patient who opens their eyes only in response to pain would be given a score of 2 (E2). This is typically assessed by applying pressure to either one of the patient's fingertips or supraorbital notch, or by squeezing one of the patient's trapezius muscles. *A patient who does not open their eyes to a painful stimulus (nil response) would be given a score of 1 (E1).

It is important for physical therapists and other healthcare providers to be prepared for emergencies at their places of work. Which of the following options is a disaster with an internal source? A. Flood B. Fire C. Hate crime D. Civil disturbance

B. Fire *An internal disaster is caused by an event contained within the facility itself. Fire is an internal disaster, while a wildfire is not. Other internal disasters include power failure, equipment failure, internal environmental conditions, and a bomb scare. *External disasters include terrorism, weather-related issues like hurricanes and floods, civil disturbances, hate crimes, and large community casualty incidents.

Which technique of massage is performed by stroking the skin and soft tissues of the pt in a circular, and longitudinal fashion by using the therapist's palm to apply deep pressure? A. Muscle Kneading B. Friction massage technique C. Stroking massage D. Vibratory technique

B. Friction massage technique *Used when a deep effect is needed

An 86-year-old male has moved into a large assisted living facility due to decreasing ability to ambulate and loss of independence with self-care. He has experienced a sudden onset of urinary incontinence, which he did not experience when he lived by himself in his small apartment. There have been no medication changes and his urinalysis has come back negative for urinary tract infection. What type of intervention is MOST appropriate for this patient? A. Pelvic floor strengthening exercises B. Functional mobility training and modification of the environment C. Drug therapy D. Biofeedback training

B. Functional mobility training and modification of the environment *Other answers, there has not been any indications of anatomical dysfunction

Which of the following is the spinal reflex that has an involuntary response to muscle contraction? A. Withdrawal reflex B. Golgi tendon reflex C. Stretch reflex D. Crossed extension reflexes

B. Golgi tendon reflex *Also known as reverse stretch reflex = opposite of the stretch reflex

Hypercalcemia occurs when a person has an increased concentration of calcium in the blood. Which of the following can be caused by hypercalcemia? A. Depressed heart actions B. Increased heart actions C. Cardiac arrest D. Ventricular arrhythmias

B. Increased heart actions *Hypercalcemia is known to also cause hypertension. *Hypocalcemia, or decreased calcium concentrations, causes depressed heart actions *Hypermagnesemia, or increased magnesium concentrations, causes cardiac arrest *Hypomagnesemia, or decreased magnesium concentrations, causes ventricular arrhythmias

A physical therapist treats a patient rehabilitating from a lower extremity injury with a cryotherapeutic agent. Which finding is MOST accurate when using this type of intervention? A. Increased spasticity B. Increased pain threshold C. Increased nerve conduction velocity D. Increased metabolic rate

B. Increased pain threshold *Cryotherapeutic = cold whirlpool, ice packs, ice massage, cold sprays, contrast baths

The parents of a ten-year-old patient diagnosed with Down syndrome have been advised to discourage the child's recent interest in attempting forward rolls. What is the MOST likely rationale for the avoidance of this movement in a patient with Down syndrome? A. Increases the risk for hydrocephalus B. Increases the risk for spinal cord injury C. Increases the risk for traumatic brain injury D. Increases the risk for developmental delay

B. Increases the risk for spinal cord injury *Patients diagnosed with Down syndrome are typically hypotonic and often present with vertebral instability, especially at the atlantoaxial joint (C1-C2). The described activity would significantly increase the risk for atlantoaxial joint subluxation or dislocation and should be avoided in these patients.

A patient with a transtibial prosthesis using a shuttle lock suspension complains of pain in the residual limb during gait. Upon inspection, it is noted that the patient has a small blister at the distal end of the residual limb. What factor is MOST likely contributing to this finding? A. Shuttle lock is failing to engage the pin B. Inferior translation of the liner on the residual limb C. Prosthetic socket is fitting too tightly on the limb D. Ankle component is set into excessive dorsiflexion

B. Inferior translation of the liner on the residual limb *The small blister is likely caused by the vacuum effect that is created between the silicone loner and the socket -Excessive perspiration -Poorly fitting silicone liner -Inability to tolerate the shuttle lock suspension

Insulin is secreted from which of the following? A. Prostate gland B. Islet beta cells C. Adrenal gland D. Sweat glands

B. Islet beta cells

A common way of performing Manual Muscle Testing (MMT) is via a "break-test" which involves the therapist to overpower the patient muscle contraction and thus evaluate strength of the muscle. Which of the following DOES NOT hold true for a "Break-Test"? A. Break test should be performed at the end range for one- joint muscles B. It is essential to avoid eccentric muscle contraction of the test muscle during a break test C. The grading system for a break test is same as generic manual muscle testing D. Break Test cannot judge concentric contraction of muscles against maximal resistance

B. It is essential to avoid eccentric muscle contraction of the test muscle during a break test

A physical therapist is performing the vertebral artery test on a patient. What is TRUE of this test? A. The patient is positioned prone. B. It results in decreased blood flow of the intracranial vertebral artery (VA) of the contralateral side. C. It increases the lumen of the vertebral artery (VA). D. The physical therapist should mobilize the cervical region to ensure adequate ROM before ever performing this test.

B. It results in decreased blood flow of the intracranial vertebral artery (VA) of the contralateral side. *the patient will need to be in a supine position-not prone-with their head supported on the exam table. It is inappropriate to perform mobilization of the cervical region without first performing this test

A patient without long nerve root functioning at or below L1 has what type of spinal cord injury? A. Hemisection of spinal cord B. LMN Lesion C. Complete Cord Lesion D. UMN Lesion

B. LMN Lesion *When a patient has lost the function of long nerve roots at L1 or below, he/she has sustained a lower motor neuron (LMN) lesion, also called a Cauda Equina injury.

You are conducting a physical therapy evaluation for a patient recently admitted to the hospital for injuries resulting from an MVA. During the physical exam, you observe a torn, jagged skin wound on the patient's left forearm. What is the BEST way to document this finding? A. Ecchymosis B. Laceration C. Abrasion D. Contusion

B. Laceration

A physical therapist treats a patient with cardiac pathology. The patient explains that there are times that they wake up during the night with the feeling of suffocation which is only relieved by sitting upright or standing for approximately 20-30 minutes. What is the MOST likely cardiac condition associated with this occurrence? A. Myocardial infarction B. Left-sided heart failure C. Right-sided heart failure D. Tetralogy of Fallot

B. Left-sided heart failure

A PT treats a patient with cardiac pathology. The patient explains that there are times that they walk up during the night with the feeling of suffocation, which is only relieved by sitting upright or standing for approximately 20-30 minutes. This scenario is MOST likely associated with which of the following conditions? A. Myocardial infection B. Left-sided heart failure C. Right-sided heart failure D. Tetralogy of Fallot

B. Left-sided heart failure *Chest discomfort with pressure, squeezing or pain, nausea, vomiting, dizziness, sweating, and SOB are common *Left side = no longer maintaining normal cardiac output *Right-sided HF = Cor Pulmonale -Dyspnea, fatigue, palpations, atypical chest pain, swelling of the LE, dizziness, and syncope

A physical therapist treats a patient with cardiac pathology. The patient explains that there are times that they wake up during the night with the feeling of suffocation, which is only relieved by sitting upright or standing for approximately 20-30 minutes. This scenario is MOST likely associated with which of the following conditions? A. Myocardial infarction B. Left-sided heart failure C. Right-sided heart failure D. Tetralogy of Fallot

B. Left-sided heart failure *Dyspnea refers to shortness of breath or the feeling of breathlessness that may be of cardiac origin. The severity of dyspnea is often measured by the frequency of the dyspnea. The patient description of the dyspnea is consistent with paroxysmal nocturnal dyspnea. *Cor pulmonale, also known as right-sided heart failure, refers to hypertrophy of the right ventricle caused by altered function of the lungs. Signs and symptoms include dyspnea with exertion, fatigue, palpitations, atypical chest pain, swelling of the lower extremities, dizziness, and syncope. Cor pulmonale is not associated with paroxysmal nocturnal dysphea.

A patient with a history of traumatic injury presents to therapy with a recent onset of neck stiffness and pain. The physical therapist believes this patient might have a dysfunctional spinal cord. What is the most appropriate cervical test the therapist can use to determine if dysfunction is present? A. Transverse ligament test B. Lhermitte's sign C. Vertebral artery test D. Modified Sharp-Purser test

B. Lhermitte's sign *Identify UMN lesions. *The vertebral artery test is used to assess vertebrobasilar integrity.

An examiner noticed that the skin color on a patient's palms has changed from its normal appearance to a cherry red color. What might this skin color change indicate? A. Congenital heart disease B. Liver issues C. Internal hemorrhage D. Increased carotene intake

B. Liver issues *Cyanosis (bluish, slate-colored discoloration) of the skin could indicate that the patient has congenital heart disease. *Pallor or lack of color of skin could indicate that the patient has an internal hemorrhage. *An orange-yellow skin discoloration could indicate that the patient has an increased carotene intake

You are examining a patient with complaints of Sl joint pain. Which special sacroiliac joint test is utilized by physical therapists to identify sacroiliac joint dysfunction that may be caused by a functional leg discrepancy? A. Gillet's test B. Long sitting test C. Gaenslen's test D. Goldthwait's test

B. Long sitting test *The long sitting test is deemed positive when limb lengths are reversed between sitting and supine positions. *Gillet's test is performed in order to assess the posterior movement of a patient's ilium in relation to the sacrum. *Gaenslen's test is performed to identify SIJ dysfunction. *Goldthwait's test is performed in order to differentiate between sacroiliac joint dysfunction and lumbar spine dysfunction.

The subcutaneous tissue, or hypodermis, is the layer of skin located underneath the dermis. Which of the following tissues are part of the hypodermis? A. Collagen and elastin fibrous connective tissues B. Loose connective tissues and fat tissues C. Loose connective tissues and elastin fibrous connective tissues D. Fat tissues and fascia

B. Loose connective tissues and fat tissues *Collagen and elastin fibrous connective tissues are found in the dermis, not the hypodermis *Fascia is deep to subcutaneous tissue, and not part of the hypodermis.

A physical therapist uses a manual wheelchair during a training session with a patient who has C5 tetraplegia. Which of the following wheelchairs would be the MOST appropriate based on the patient's level of injury? A. Manual wheelchair with sip and puff controls B. Manual wheelchair with handrim projections C. Manual wheelchair with friction surface handrims D. Manual wheelchair with standard handrims

B. Manual wheelchair with handrim projections *Sip and puff controls are used only on power wheelchairs. These types of controls are most often used on patients with C4 tetraplegia. Innervation at the C4 level includes the diaphragm, trapezius, face, and neck muscles. *A manual wheelchair with handrim projections would be appropriate to use during a training session for a patient with C5 quadriplegia, however, it may not be used as the primary mode of mobility due to the limited UE muscle innervation and the associated endurance issues. *A manual wheelchair with friction surface handrims would be more appropriate for a patient with C6-C7 tetraplegia secondary to the motor innervation at the C6 and C7 levels. *Standard handrims are used for patients with injury at the C8 level and below since they possess full upper extremity innervation.

Which area would be the LEAST LIKELY to develop a pressure ulcer in an elderly patient with poor mobility? A. Ischial tuberosity B. Medial malleolus C. Calcaneus D. Lateral malleolus

B. Medial malleolus

Choose the option that is FALSE about Medicare? A. People aged 65 and above, terminally ill patients, and disabled individuals can benefit from Medicare. B. Medicare is a joint state and federal health insurance policy. C. Part A of the Medicare program covers IPD care. D. Part B of the Medicare covers OPD care, lab works, and radiographic work.

B. Medicare is a joint state and federal health insurance policy.

A patient is undergoing trials to facilitate weaning from mechanical ventilation. Which of the following is a criteria to wean the patient off the ventilator? A. SaO2 above 88% B. Minute ventilation less than 15L/min C. FiO2 of 55% D. Respiratory rate of 40 breaths/min

B. Minute ventilation less than 15L/min Specific Weaning respiratory parameters include the following: -Adequate gas exchange with FiO2 less than 50% and SaO2 greater than 90% -Negative inspiratory force of 20-30 cm H20 -The ratio of respiratory rate to tidal volume (RR/VT) less than 105, with respiratory rate less than 35 breaths per minute -Minute ventilation (VE = respiratory rate x tidal volume) less than 15 L/min

When assessing for potential vascular compromise in the hand, a physical therapist would ideally perform what test? A. Bunnel-Littler test B. Modified Allen test C. Eichhoff's test D. Phalen's test

B. Modified Allen test *The modified Allen test is a special wrist and hand test that assesses vascular compromise. It is performed by first identifying the radial and ulnar arteries at the wrist, then having the patient open/close their fingers quickly several times, and then make a closed fist. Then the therapist compresses the ulnar artery and has the patient open their hand. The therapist releases compression on the artery and observes for vascular filling, and then performs the same procedure with the radial artery. A positive finding of abnormal filling of blood within the hand indicates vascular compromise. *The Bunnel-Littler test identifies intrinsic tightness at the PIP joints. Phalen's test assesses carpal tunnel compression of the median nerve. Eichhoff's test identifies De Quervain'stenosynovitis.

A patient presents to therapy with what the therapist believes is a partially-torn ulnar collateral ligament. What special test should the therapist administer to determine the official diagnosis? A. Elbow extension test B. Moving valgus stress test C. Varus stress test D. Valgus stress test

B. Moving valgus stress test *The varus/valgus stress test is a special elbow test that assesses the laxity of the ligaments *The elbow extension test rules out fractures or joint injury

One of your patients presents to therapy with decreased muscle strength and incoordination, which fluctuate but have generally been getting worse. What are these symptoms likely attributed to? A. Myasthenia gravis B. Multiple sclerosis C. Parkinson's disease D. Cerebrovascular accident

B. Multiple sclerosis

A PT should always care for signs of hypoglycemia while treating a diabetic patient. All of the following are early signs of hypoglycemia except? A. A strong desire for food B. Mumbling while talking C. Sweating and dizziness D. Trembling and weakness

B. Mumbling while talking

While reading a 6-second ECG strip of a patient, you notice that the V6 displays a PR interval of 200 ms and ST segment depression followed by T wave inversion. This is highly suggestive of? A. Zone of injury B. Myocardial Ischaemia C. Zone of infarction D. 1* Heart block and myocardial ischaemia

B. Myocardial Ischaemia *ST segment depression *T inversion *PR interval at 200 ms = normal

There is a range of neurons in the brain, all of which have various functions related to cognition, movement, speech, and more. What neuron does not send signals and is essential for the production of myelin sheaths and other neurons? A. Axon bundles B. Neuroglia C. Interneurons D. Projection neurons

B. Neuroglia *Neuroglia are support cells that do not transmit signals. They are important for myelin and neuron production as well as maintaining K+ levels. They are also responsible for the reuptake of neurotransmitters after transmission at synapses. Projection neurons are the neurons that carry impulses to the central nervous system's other parts. Interneurons are the central nervous system's short relay neurons. Axon bundles refer to the fasciculi or tracts.

You are working in a hospital setting, treating a patient with a chronic foot ulcer that has tunneling and large amounts of exudate. You utilize alginates for wound treatment. Alginates are dressings used for wounds that require packing and absorption. Which of the following BEST describes alginates? A. Adhesive wafers that form a gelatinous mass over the wound B. Nonwoven fluffy dressings that stem from seaweed C. Gels with varying absorptive capacity that are insoluble in water D. Hydrophilic semipermeable membranes with varying adhesive and absorptive properties

B. Nonwoven fluffy dressings that stem from seaweed

The thoracic springing test is a special thoracic test that evaluates the mobility of the intervertebral joint in a patient's thoracic spine. Which of the following options is NOT a positive finding of this test? A. Pain B. Numbness C. Excessive movement D. Restricted movement

B. Numbness

A physical therapist decides to use the Yergason test to evaluate a patient for bicipital tendinosis. Which of the following options would MOST LIKELY indicate a positive result of this test? A. Pain in the short head of the biceps tendon B. Pain in the long head of the biceps tendon C. Pain in the anterior shoulder D. A clicking or catching sensation

B. Pain in the long head of the biceps tendon *The Yergason test is used to test the integrity of the transverse ligament, and to identify bicipital tendinosis/tendinopathy and SLAP lesions. A positive result occurs when the tendon of the biceps long head "pops" out of the groove. This test may also reproduce pain in the long head of the biceps tendon. Pain in the short head of the biceps tendon does not constitute a positive result of the Yergason test. Pain in the anterior shoulder is a positive result of Speed's test. A clicking or catching sensation is a positive result of the compression-rotation test.

A physical therapist is employed in an acute care facility. Which scenario would provide the MOST appropriate justification for using restraints on a patient? A. Patient is on a medication that causes hallucinations B. Patient is too agitated to allow for the completion of an essential surgery C. Patient is verbally abusive to the hospital staff D. Patient is placed on bedrest orders following a major surgery

B. Patient is too agitated to allow for the completion of an essential surgery *Justifiable use of restraints = pt self-harm or harm to others or pt who is so agitated that essential medical care cannot be completed

A therapist is about to conduct ambulation for a patient diagnosed with bacterial pneumonia inside their room. Patient coughs often and postural drainage has not been carried out on that day. Considering the patient's vitals are stable and has a normal balance, which of the following precautions must be followed before starting ambulation? A. Provide patient with surgical mask, therapist wears N95 mask B. Patient wears surgical mask, therapist wears surgical mask and face shield C. Therapist wears N95 mask and face shield D. Therapist wears surgical mask and face shield

B. Patient wears surgical mask, therapist wears surgical mask and face shield *N95 mask = airborne transmission

A physical therapist is evaluating a 25-year-old runner presenting for left hip pain. During the physical exam, the therapist observes restricted hip internal rotation, referred pain to the posterior thigh, and weakness of hip external rotation. The therapist has ruled out the involvement of the lumbar spine and sacroiliac joint. Which of the following is the MOST likely condition the patient is experiencing? A. Femoroacetabular impingement B. Piriformis syndrome C. Trochanteric bursitis D. lliotibial band friction disorder

B. Piriformis syndrome

A physical therapist is treating a patient during rehabilitation following a mastectomy procedure in a hospital outpatient clinic. The patient is currently in phase Il management of stage I lymphedema. Of the following, which treatment will be MOST appropriate for this patient? A. Pneumatic compression pump with high pressure B. Pneumatic compression pump with low pressure C. Ballistic exercise D. Electrotherapeutic modalities at 40 Hz

B. Pneumatic compression pump with low pressure *Pneumatic compression pumps are recommended for patients in phase II management of lymphedema with stage I lymphedema. Compression pumps should be used with low pressure, as high pressure may damage the lymph nodes. *Ballistic exercises, jogging, and rotational motions are contraindicated as they can exacerbate lymphedema. *Electrotherapeutic modalities may be recommended in some cases, but are contraindicated at frequencies above 30 Hz.

A physical therapist measures a patient's shoulder complex medial rotation in the standard position. The therapist records the patient's shoulder medial rotation as 0-70 degrees and classifies the end-feel as firm. Which portion of the joint capsule is PRIMARILY responsible for the firm end-feel? A. Anterior B. Posterior C. Inferior D. Superior

B. Posterior *A firm end-feel caused by the anterior joint capsule would most often be associated with lateral rotation *A firm end-feel caused by the inferior joint capsule would most often be associated with flexion and abduction *A firm end-feel caused by the superior joint capsule would most often be associated with extension and adduction

A female patient in her 30th week of pregnancy is presented to the emergency with a severe headache. On vital examination her blood pressure was high. What is the possible diagnosis for this patient? A. Gestational diabetes B. Preeclampsia C. Hyperthyroidism D. Hypothyroidism

B. Preeclampsia

A female patient in her 30th week of pregnancy is presented to the emergency with a severe headache. On vital examination her blood pressure was high. What is the possible diagnosis for this patient? A. Gestational diabetes B. Preeclampsia C. Hyperthyroidism D. Hypothyroidism

B. Preeclampsia *It often occurs after the 24th week of pregnancy and can cause a severe headache with other symptoms.

Assuming a normal progression of Duchenne muscular dystrophy, which of the following impairments would have MOST likely have been present at the initial onset of the patient's condition? A. Distal muscle weakness B. Proximal muscle weakness C. Impaired respiratory function D. Diminished sensation to light touch

B. Proximal muscle weakness *Duchenne muscular dystrophy is an inherited disorder, characterized by rapidly worsening muscle weakness that starts in the proximal muscles of the lower extremities and pelvis and later affects all voluntary muscles.

Proper infection control is essential to protecting patients' well-being. This means therapists must take necessary precautions to avoid contagion between patients. What skin disorder is not spread through direct contact? A. Herpes 1 B. Psoriasis C. Impetigo D. Tinea pedis

B. Psoriasis *Psoriasis = chronic autoimmune disorder of the skin (dry + scaly) -In ears, scalp, knees, elbows, and genitalia -Hereditary *Impetigo = bacterial infection, small pus-filled vesicles, and itching -Elderly and pediatric populations *Tinea pedis = Athlete's Foot -Fungal infection -Direct contact

You are seeing a woman for a physical therapy evaluation. She has a history of three vaginal childbirth deliveries, the most recent of which occurred six months ago. Now, she complains of difficulty with bowel movements and pressure in the vagina. Which of the following conditions is she MOST LIKELY to have? A. Urethrocele B. Rectocele C. Cystocele D. Uterine prolapse

B. Rectocele *Urethrocele is a pelvic floor disorder associated with the urethra pressing into the vaginal wall. *Cystocele is a pelvic floor disorder associated with the bladder pushing into the vagina. *Uterine prolapse is a pelvic floor disorder associated with the uterus bulging into the vagina.

While a physical therapist is working with a 50-year-old patient who has been admitted to the hospital for MRSA infection, the patient begins to complain of pain in their shoulder that does not improve with rest or a change of position. Of the following, what should the physical therapist be MOST concerned about? A. Referred visceral pain from the spleen and pericardium B. Referred visceral pain from the diaphragm and pericardium C. Referred visceral pain from the gallbladder D. Referred visceral pain from the bladder trigone

B. Referred visceral pain from the diaphragm and pericardium *Referred pain in the shoulder may be caused by the diaphragm, the diaphragmatic pericardium, or the heart. *The spleen can cause localized pain on the left side of the abdomen. *The gallbladder can cause pain on the right side of the mid and lower back. The bladder trigone can cause pain on the medial side of the left leg.

The normal urine magnesium serum level is 1.8 to 2.4 mg/dL. If an individual's magnesium serum level is greater than 2.4 mg/dL, hypermagnesemia occurs. Which of the following is a cause associated with hypermagnesemia? A. Hemodialysis B. Renal failure C. Hepatic cirrhosis D. Severe burns

B. Renal failure *Other causes associated with hypermagnesemia include diabetic acidosis, Addison's disease, and hypothyroidism *Signs of hypermagnesemia include weakness, drowsiness, confusion, and hypotension *Hemodialysis, hepatic cirrhosis, and severe burns are causes associated with hypomagnesemia

If you are dealing with a 78 years old patient diagnosed with myasthenia gravis, you must be aware of the four types of this disorder. Which of the following is most likely to occur in a myasthenic crisis? A. Begins after few weeks of the disease onset. B. Respiratory failure occurs. C. Only the extraocular muscles are affected D. It is treatable in OPD.

B. Respiratory failure occurs.

A patient with right groin pain is being evaluated. It is found that the patient is having right leg anterior as compared to the left in long sitting while supine lying shows left leg anterior to the right. Which of the following will LEAST LIKELY be present in this patient? A. Right side posterior innominate rotation B. Right side retroversion C. Right side cox vara D. Right subtalar pronation

B. Right side retroversion

A physical therapist is seeing a woman in a hospital outpatient facility for lymphedema treatment. The patient is currently post-right-breast mastectomy due to breast cancer. Based only on this information, which type of lymphedema is this patient MOST LIKELY to have? A. Primary lymphedema B. Secondary lymphedema C. Stage 2 lymphedema D. Stage 1 lymphedema

B. Secondary lymphedema

A physical therapist in an acute care hospital performs stair training with a patient who is non-weight bearing following triple arthrodesis surgery of the ankle. After the session, the therapist observes that the dressing over the patient's incision is becoming increasingly pink and damp. Which term BEST characterizes the description of this exudate? A. Sanguineous B. Serosanguineous C. Serous D. Seropurulent

B. Serosanguineous *Sanguineous exudate is described as red in color with a thin, watery consistency. It typically indicates a disruption in the growth of new blood vessels or fragile granular tissue. *Serosanguineous exudate is described as light red to pink in color with a thin, watery consistency. It is normally present during the inflammatory and proliferative phases of healing. Serosanguineous exudate is consistent with both the patient's phase of healing and the observed dressing discoloration. *Serous exudate is described as clear and light in color with a thin, watery consistency. It is normally present during the inflammatory and proliferative phases of healing. *Seropurulent drainage is described as yellow to tan in color with a thin, cloudy appearance. Seropurulent exudate is often an early sign of infection.

You are examining a patient's cognitive functions. To check the pt's ability of immediate recall, which of the following would you perform? A. Ask the pt to walk and count backward at the same time B. Show the pt 5 objects and then ask the patient their names after 5 minutes C. Ask the pt what he had for breakfast D. Ask the pt his office address

B. Show the pt 5 objects and then ask the patient their names after 5 minutes

A patient is able to stand on the floor without any sway in a well lit room but experiences frequent falls when he moves to a dark room. There is no nystagmus present. This patient is most likely to have an impairment of A. Visual system B. Somatosensory system C. Vestibular system D. Basal ganglia

B. Somatosensory system

A PT would like to strengthen a patient's right gluteus medius muscle in a manner that emphasizes its role in the gait cycle. Which of the following exercises BEST accomplishes this goal? A. Stand on the left leg, allow the right side of the pelvis to lower, then bring the pelvis back to a level position B. Stand on the right leg, allow the left side of the pelvis to lower, then bring the pelvis back to a level position C. Stand on the left leg, keep the pelvis level, then abduct the right hip against resistance D. Stand on the right leg, keep the pelvis level, then perform a mini-squat

B. Stand on the right leg, allow the left side of the pelvis to lower, then bring the pelvis back to a level position

A physical therapist works with a patient with hemiparesis who uses a hemiplegic chair for mobility. Which activity would become more challenging for the patient based on this specific type of wheelchair? A. Reaching for objects outside the base of support B. Standing up from the seat of the chair C. Performing independent pressure relief D. Elevating the legs for edema management

B. Standing up from the seat of the chair *A hemiplegic chair incorporates a seat that is 2 inches lower than a standard chair (20 inches)

Each skin type has various layers. What part of the epidermis can be found on the soles of the feet and palms of the hands? A. Stratum corneum B. Stratum lucidum C. Stratum spinosum D. Stratum granulosum

B. Stratum lucidum The epidermis is a skin layer that itself contains five layers. The five layers of the epidermis from outermost to innermost are as follows: -Stratum corneum: shingle-like dead cells filledwith keratin. -Stratum lucidum: formed from dead cells; only occurs in thick portions of the palms and soles of the feet. -Stratum granulosum: contains live keratinocytes and Langerhans cells. -Stratum spinosum: spiny layer which also contains keratinocytes and Langerhans cells. -Stratum basale: contains epidermal cells, melanocytes, and Merkel cells.

A physical therapist prepares to measure the amount of chest excursion during inspiration of a patient with a neurologic injury. Which patient position would be the MOST appropriate? A. Sitting B. Supine C. Prone D. Sidelying

B. Supine *The supine position creates support and resistance to the diaphragm. There is a direct correlation between the amount of chest expansion and intercostal strength.

When reviewing a patient's medical history, a physical therapist working in home health learns the patient has orthopnea. What is the least ideal position to treat this patient in? A. Standing B. Supine in bed C. Sitting in standard chair D. Lying on their side

B. Supine in bed *In a supine position, the patient will be lying on his back. Orthopnea describes an inability to breathe when the patient is lying or reclined. Typically, this condition is a shortness of breath when lying flat, causing patients to sleep propped over several pillows or even upright in a recliner chair. It would be appropriate to treat the client in any other position, including sitting in a standard chair, standing, or lying on their side.

A physical therapist is performing an evaluation on a patient with low back pain in an outpatient facility. During the physical exam, the therapist has the patient lie supine and passively raises the patient's hip into flexion, knee into extension, ankle into dorsiflexion, and foot into inversion. Of the following, what is MOST likely being tested? A. Sciatic and tibial nerves B. Sural nerve C. Common fibular nerve D. Spinal nerve root

B. Sural nerve *The third version of the SLR (SLR3) stresses hip flexion, knee extension, ankle dorsiflexion, and foot inversion. It is used to test the sural nerve. There are five versions of the straight leg raise test. *The SLR test is used to test for sciatic and tibial nerves. The SLR4 test is used to test the common fibular nerve. *The SLR5 (Well Leg) test is used to test the spinal nerve root.

The lumbar plexus, which can be found at the root of the lower limbs, arises from what nerve roots? A. T12 through L2 B. T12 through L4 C. L4 through S3 D. L1 through L3

B. T12 through L4 *The cervical plexus arises from nerve roots C1 through C4. *The brachial plexus arises from nerve roots C5 through T1. *The lumbar plexus arises from nerve roots T12-L4. *The sacral plexus arises from nerve roots L4-S3.

While educating a PT student, about flat-foot, the supervisor asks about abnormal positioning of the articulations. A Flat-foot deformity will present with the following abnormal positions at the foot EXCEPT A. Forefoot abduction B. Talar Dorsiflexion C. Lateral drift of calcaneus D. Dorsiflexed metatarsals

B. Talar Dorsiflexion

Regarding hemodynamics, what is atrial filling pressure? A. The difference between the left and right ventricular pressures B. The difference between venous and atrial pressures C. The difference between stroke volume and left ventricular pressure D. The difference between left ventricular end diastolic volume and cardiac index

B. The difference between venous and atrial pressures *Atrial filling pressure is the difference between venous pressure and atrial pressure. Hemodynamics is the term used to describe the study of blood flow or the study of blood circulation. Right atrial filling pressure is affected by ventricular contractions, changes in intrathoracic pressure, inspiration, coughing, and forced expiration.

The sounds of the heart provide valuable information about the cardiac cycle, and may indicate various pathologies. What occurs during the S2 ("dub") sound? A. The beginning of systole B. The end of systole C. Ventricular filling and atrial contraction D. The normal closure of the mitral and tricuspid valves

B. The end of systole * The S1 ("lub") sound of the heart is caused by the normal closure of the mitral and tricuspid valves. This sound marks the beginning of systole. *The S4 sound is associated with ventricular filling and atrial contraction and may indicate pathology.

While reviewing medical information for an patient you are going to evaluate in a hospital setting, you read that the inner layer of the patient's pericardium has been compromised. What structure is being indicated? A. The myocardium B. The epicardium C. The endocardium D. The atrioventricular valves

B. The epicardium

A physical therapist is assessing a patient's functional balance. They grade the patient's balance as being poor. What does this mean? A. The patient is unable to maintain balance B. The patient is unable to move without loss of balance C. The patient accepts only minimal challenge D. The patient accepts only moderate challenge

B. The patient is unable to move without loss of balance

Which of the following is characteristic of stage 2 lymphedema? A. Elevation reduces swelling B. The skin begins to toughen C. Possible changes in skin color D. Stemmer's sign is negative

B. The skin begins to toughen

A physical therapist is assessing a new patient's hand and wrist. Based on initial observations, the therapist performs the Bunnel-Littler test. What is most likely the reason the therapist chose to administer this test? A. The tightness around the proximal interphalangeal joint B. The tightness of the structures that surround the MCP joints C. The laxity of the ligaments D. The level of sensory innervation within the hand that correlates with grasping

B. The tightness of the structures that surround the MCP joints *The Bunnel-Littler test is a special wrist and hand test that is performed to identify the tightness of the structures that surround the MC joints. After performing the Bunnel-Littler test, the therapist will know if the capsule is tight or if the intrinsic muscles are tight. *The tight retinacular test is a special wrist and hand test that is performed to identify tightness around the proximal interphalangeal joint. The interphalangeal joint varus/valgus stress tests are special wrist tests that are performed to identify the laxity of the ligaments. The two-point discrimination test is a special wrist and hand test that is performed to identify the level of sensory innervation within the hand that correlates with grasping.

Which of the following is true of BOTH palliative and hospice care? A. They are specialized types of medical care for people living with serious long-term illness. B. They include the provision of multidisciplinary and supportive services. C. They are specialized types of medical care for people in the final stage of a terminal illness. D. They focus on comfort and quality of life.

B. They include the provision of multidisciplinary and supportive services. *Palliative care is a specialized type of care for people living with serious long-term illness; its focus is on relief from symptoms and the stress of illness. *Hospice care is a specialized type of care for people in the final stage of a terminal illness; its focus is on comfort and quality of life.

Which type of splint would MOST likely be prescribed for a patient with de Quervain's tenosynovitis? A. Ulnar gutter splint B. Thumb spica splint C. Radial gutter splint D. Dorsal forearm splint

B. Thumb spica splint

Fractures are graded as Type I-V. Which type of fracture is generally found after the fact and immediate intervention is rarely provided? A. Type I B. Type V C. Type Il D. Type IV

B. Type V *Type V fractures are usually associated with compression injury of the epiphyseal plate. This type of fracture is usually caused by a crush or compression injury *Type I and type Il fractures often require relocation and immobilization with a cast *Type IV fractures often require surgery in order to restore alignment.

An examiner noticed that a patient's skin color has changed from its normal appearance to a bluish, slate-colored discoloration (cyanosis). Which of the following conditions may be indicated by this skin color change? A. Liver Disease B. Anemia C. Venous obstruction D. Internal hemorrhage

B. Venous obstruction Can also mean: -Congestive/Congenital heart failure -Advanced lung disease *RED Skin = Possibly liver disease

People with the bleeding disorder have a defective coagulation balance and need extreme precautionary care when performing manual therapy techniques. Which of the following is a coagulation defect? A. Bone marrow disease B. Von Willebrand disease C. Dysfunctional platelets D. Low platelets

B. Von Willebrand disease

People with the bleeding disorder have a defective coagulation balance and need extreme precautionary care when performing manual therapy techniques. Which of the following is a coagulation defect? A. Bone marrow disease B. Von Willebrand disease C. Dysfunctional platelets D. Low platelets

B. Von Willebrand disease

What is barognosis?

Barognosis is tested by placing similar objects with varying weights in a patient's hand and the patient identifies the differences in weights.

Decubitus ulcers

Bed sores, pressure sores

A patient presents with abnormal posturing of the right UE. There is flexion of the right PIP with extension at MCP and DIP. Such deformity can be as a result of what?

Boutonniere Deformity Damage to the common extensor tendon (central slip) at the middle phalanx. This damage causes the lateral bands of the ED to displace anteriorly near PIP so -1. Instead of extending the PIP, extensor tendon now flexes it instead causing PIP flexion -2. While still hyper extending DIP -3. MCP hyperextension causes loss of influence of lumbricals therefore not flexing the MCP

Fahrenheit to Celsius

C = (F-32)/1.8

As a physical therapist, it is important to assess basic vitals during an evaluation, including heart rate, blood pressure and respiratory rate. Which of the following is the normal respiratory rate for an adult? A. 20-25 breaths per minute B. 10-20 breaths per minute C. 12-20 breaths per minute D. 15-20 breaths per minute

C. 12-20 breaths per minute *The normal respiratory rate for a newborn is 30-40 breaths per minute. *The normal respiratory rate for a child is 20-30 breaths per minute.

Before beginning to train for a wheelchair racing event, a 40-year-old patient with complete C7 tetraplegia undergoes a graded exercise test using an upper body ergometer. In the absence of cardiac pathology, which value represents the MOST likely maximum heart rate? A. 60 beats per minute B. 80 beats per minute C. 120 beats per minute D. 190 beats per minute

C. 120 beats per minute *T3 or higher complete spinal cord injuries typically have difficulty reaching age-adjusted maximum and target training heart rate zones. *Heart rate responses are more variable with injuries between T4-T6 *Patients with injuries below T7 are typically able to reach age-adjusted ranges.

A patient with diabetes is being treated for a wound on the plantar surface of the foot. The wound is documented as a deep ulcer with abscess. Which of the following numeric grades would the physical therapist use when grading this ulcer based on the Wagner Ulcer Grade Classification System? A. 1 B. 2 C. 3 D. 5

C. 3

A pt with diabetes is being treated for a wound on the plantar surface of their foot. The would has penetrated to the bone and resulted in a bone infection. Using the Wagner Ulcer Grade Classification system, which score is MOST consistent with the described wound? A. 1 B. 2 C. 3 D. 4

C. 3

A physical therapist works with a nine-year-old child with cystic fibrosis. What age BEST approximates the median age of survival for a patient with cystic fibrosis? A. 15 years old B. 25 years old C. 35 years old D. 50 years old

C. 35 years old *More than 70% of patients with cystic fibrosis are diagnosed by two years of age. *More than 45% of patients with cystic fibrosis are age 18 years or older. *The median age of survival for a patient with cystic fibrosis is currently in the mid-thirties. This age of survival has steadily increased each decade due to advances in treatment which has resulted in many individuals with cystic fibrosis living well into adulthood.

An A1C test provides information about blood sugar levels over the last two to four months. Which value would indicate an A1C value of pre-diabetes? A. 7.0% B. 5.6% C. 5.9% D. 6.5%

C. 5.9% *An A1C test should be 5.6% or less for a normal adult. An A1C of 5.7-6.4% would qualify as pre-diabetes. An A1C of 6.5% or higher indicates diabetes.

A PT checks the water temperature of the hot pack machine after several patients report the heat being very strong. Which of the following temperatures would be acceptable? A. 83 degrees C (181 degrees F) B. 64 degrees C (147 degrees F) C. 71 degrees C (160 degrees F) D. 94 degrees C (201 degrees F)

C. 71 degrees C (160 degrees F) *A hot pack must be stored in hot water between 158 to 167 degrees F (70-75 degrees C) *6-8 towel layers should be applied between the hot pack and the treatment surface

It is part of a physical therapist's role to educate patients on management strategies for stress incontinence. What is the best definition of functional electrical stimulation (FES) as it pertains to the treatment of stress incontinence? A. A pelvic floor muscle intervention that utilizes pressure recordings to support active contractions B. A pelvic floor muscle intervention that reeducates the muscles of patients who have the ability to initiate active contractions C. A pelvic floor intervention that reeducates the muscles of patients who have the inability to initiate active contractions D. A pelvic floor muscle exercise that strengthens the muscles, and should be performed multiple times per day

C. A pelvic floor intervention that reeducates the muscles of patients who have the inability to initiate active contractions *Functional electrical stimulation is a pelvic floor intervention that reeducates the muscles of patients who have the inability to initiate active contractions. This treatment is not designed for patients who are able to initiate active contractions. Kegel's exercises strengthen the pelvic muscles by shutting off the flow of urine and should be performed multiple times per day. Biofeedback is a pelvic floor intervention that utilizes pressure recordings to support active contractions.

What muscles are MOST commonly associated with de Quervain's tenosynovitis? A. Abductor pollicis longus and extensor pollicis longus B. Abductor pollicis brevis and extensor pollicis brevis C. Abductor pollicis longus and extensor pollicis brevis D. Abductor pollicis brevis and extensor pollicis longus

C. Abductor pollicis longus and extensor pollicis brevis

Physical therapists should teach patients who suffer from stress incontinence how to perform pelvic floor muscle exercises. What are Kegel exercises? A. Heavy resistance exercises B. Muscle reeducation for patients unable to initiate active contractions C. Active strengthening exercises D. The use of weighted vaginal cones for home exercises

C. Active strengthening exercises *FES is a pelvic floor treatment that reeducates pelvic floor muscles for patient who cannot initiate active contractions *Progressive strengthening is a pelvic floor muscle exercise = weighted vaginal cones

A physical therapist plans to begin exercises with a patient diagnosed with Guillain-Barre syndrome ten days ago. The patient currently has Poor Plus (2+/5) strength in the lower extremities. Which of the following types of lower extremity strengthening exercises would be the MOST appropriate in this early stage of treatment for Guillain-Barre syndrome? A. Gluteal and quad setting exercises and active range of motion B. Aquatic resistance exercises with foam weights C. Active-assisted range of motion and gravity-minimized exercises D. Passive range of motion, bridging, and long arc quad exercise

C. Active-assisted range of motion and gravity-minimized exercises

While examining a patient, the PT learns that the pt has acute pain in a band-like shape around their abdomen, radiating to their back. The pain is worse when the patient lies in the supine position. Based on this information, the therapist has reason to believe the pt has a gastrointestinal condition and refers the pt to their physician. Which of the following gastrointestinal conditions is MOST likely to cause this type of pain? A. Acute gastritis B. Early-stage pancreatic cancer C. Acute pancreatitis D. Hepatitis

C. Acute pancreatitis

During an initial examination, a physical therapist hears a patient report acute pain wrapping around their abdomen and radiating to the back. The pain increases in supine. The therapist suspects this pain is due to a digestive condition, so they tell the patient to consult their doctor. What gastrointestinal condition is most likely the reason for this pain? A. Hepatitis B. Early-stage pancreatic cancer C. Acute pancreatitis D. Acute gastritis

C. Acute pancreatitis *Interventions for acute pancreatitis include IV fluids, pain medication, and sometimes surgery

You are working with a 78-year old man who complains of urinary incontinence. Overflow urinary incontinence occurs when the bladder continuously leaks due to urinary retention and an overdistended bladder. Which of the following correctly identifies a cause of overflow urinary incontinence? A. Detrusor muscle instability B. Laxity of pelvic floor musculature C. Anatomical obstruction, like from an enlarged prostate D. Impaired cognition

C. Anatomical obstruction, like from an enlarged prostate

A physical therapist is working with a 50-year-old female patient in an outpatient clinic following total hip replacement surgery. During the session, the patient complains of abdominal pain, tenderness, and constipation. In reviewing the patient's current medication list, which type of medication would be LEAST likely to contribute to these symptoms? A. Opiates C. Antibiotics D. Antidepressant D. Calcium channel blockers

C. Antibiotics *Some side effects of common medications can trigger gastrointestinal changes. Antibiotics can have the side effect of diarrhea (not constipation). Side effects can also include stomach cramps, nausea, and vomiting. Because this patient recently had surgery, it is also very likely that they have been prescribed antibiotics to prevent infection atter surgery.

A patient presents with a small, round, partial-thickness wound on the lateral malleolus with distinct wound edges. Upon examination, the wound bed is a grey color without evidence of granulation tissue and appears to be dry. The wound is extremely tender to palpation and the patient reports calf pain during walking. These findings are MOST associated with which of the following types of ulcers? A. Venous stasis B. Neuropathic C. Arterial D. Decubitus

C. Arterial

What imaging technique would have MOST likely been used to confirm the diagnosis of adhesive capsulitis? A. Fluoroscopy B. Myelography C. Arthrography D. Computed tomography

C. Arthrography

Which of the following risk factors is associated with diabetic ulcers? A. Arteriosclerosis B. Atheroembolism C. Atherosclerotic peripheral artery disease D. Chronic arterial insufficiency

C. Atherosclerotic peripheral artery disease

As part of a physical therapy evaluation, a patient is asked to shrug her shoulders. What spinal cord segment and spinal nerves are involved in this action? A. C1-C4; posterior rami B. C4-C6; suprascapular C. C1-C4; spinal accessory D. C5-C7; long thoracic

C. C1-C4; spinal accessory *Neck flexion, extension, rotation, and lateral bending involve spinal cord segment C1-C4, and the C3-4 posterior rami in addition to the spinal accessory nerves. *Shoulder protraction and scapular upward rotation involve spinal segment C5-C7 and the long thoracic nerves. *Shoulder abduction and lateral rotation involve spinal cord segment C4-C6 and the suprascapular nerves.

Hemodynamics are important statistics that relate to the function of the heart and its efficiency in pumping blood. Which of the following terms refers to the volume of blood that is pumped by the heart per minute? A. Contractility B. Stroke volume C. Cardiac output D. Cardiac index

C. Cardiac output

Which of the following physiological responses increases as a result of general cold application? A. Pulse rate B. Metabolic rate C. Cardiac output D. Venous blood pressure

C. Cardiac output

Which of the following options is characteristic of a central cord lesion? A. Lateral corticospinal tract loss B. Spastic paralysis that occurs below the lesion C. Cavitation of the central cord in the cervical section D. Potential for nerve regeneration, which is often incomplete

C. Cavitation of the central cord in the cervical section

When working with an intermittent mechanical compression unit (in order to provide a patient's extremity with external pressure), a physical therapist will complete what step first? A. Set the inflation and deflation ratio B. Elevate the patient's limb 45 degrees C. Check the patient's blood pressure D. Place the unit over the patient's extremity

C. Check the patient's blood pressure

A 58 year old male patient suffering from diabetes since 5 years and a history of chest pain on the left side since 1 year is undergoing cardiac rehabilitation. Patient's chest pain is usually resolved after consuming 1 tablet of 5mg Nitroglycerin. What would NOT be the most appropriate scenario to refer this patient to a physician or emergency? A. Chest pain radiating to the left shoulder and jaw B. Increase in chest pain after stopping exercises C. Chest pain now taking 2 tablets of Nitroglycerin to resolve. D. Upper back feeling abnormally cool and sweaty on touch than usual

C. Chest pain now taking 2 tablets of Nitroglycerin to resolve *For anginal pain or discomfort that is not relieved by rest or relieved by up to 3 nitro doses in 10 to 15 minutes --> call physician *Pain moving to local to radiating = call physician *Increased chest pain = call 911 or physician *Cold sweats, clammy areas = call 911 or physician

You are assessing a patient admitted in the ICU. The patient presents with no eye movement and an absent sleep-wake cycle. Flashing light in the eye shows bilateral pupillary constriction. What state of consciousness is the patient in? A. Brain dread B. Vegetative state C. Coma D. Minimally conscious

C. Coma *Lack of eye opening and sleep/wake cycle with intact brain stem reflex responses (pupil response)

The body has 31 pairs of spinal nerves. These nerves are split up into groups. What is not the name of a spinal nerve group? A. Lumbar B. Sacral C. Cranial D. Thoracic

C. Cranial *Cranial nerves are different from spinal nerves. There are 12 pairs of cranial nerves. *The 31 pairs of spinal nerves are divided into 4 groups, as follows: -Cervical (8 pairs of nerves) -Thoracic (12 pairs of nerves) -Lumbar (5 pairs of nerves) -Sacral (5 pairs of nerves) -Coccygeal (1 pair of nerves)

Which of the following is "NOT" the appropriate assistive walking aid for a patient who has an ankle sprain and had a history of the left and right side radiation therapy for breast cancer. The patient also has controlled lymphedema. A. Cane B. Roll Walker C. Crutches D. Pick-up walker aid

C. Crutches

A physical therapist sets the parameters of an iontophoresis unit in preparation for treatment on a patient diagnosed with lateral epicondylitis. Which of the following treatment parameters would represent the highest iontophoresis dose? A. Current amplitude of 1.0 milliamps; duration of 20 minutes B. Current amplitude of 2.0 milliamps; duration of 10 minutes C. Current amplitude of 3.0 milliamps; duration of 20 minutes D. Current amplitude of 4.0 milliamps; duration of 10 minutes

C. Current amplitude of 3.0 milliamps; duration of 20 minutes *The amount of electricity used when performing iontophoresis is measured in milliamp-minutes (mA-min). This value is often termed "dosage" and is determined by multiplying current amplitude and time. A typical dosage for each treatment is 40-80 mA-min. * A current amplitude of 3.0 milliamps delivered for 20 minutes would result in a dosage of 60 mA-min.

Which impairment would best explain why a child with Down syndrome prefers to "W" sit instead of assuming a traditional sitting position when playing with toys? A. Hypotonicity B. Hypertonicity C. Decreased core strength D. Fatigue

C. Decreased core strength

A patient suffering from facial burns is admitted in an inpatient facility presents with a waxy-white skin appearance and broken blisters over the forehead. Examination reveals an insensitivity to light touch and soft pin prick. The burns can be classified under A. Superficial partial thickness burns B. Full thickness burns C. Deep partial thickness burns D. Subdermal burns

C. Deep partial thickness burns

A physical therapist is treating a patient with a nerve injury in the outpatient setting. The patient sustained an injury at work, damaging many crucial nerve fibers in her arm. During your assessment, you learn that she has difficulty sensing pain, touch, and temperature. Of the three nerve fiber types (A, B, and C), which A fiber is MOST LIKELY affected? A. Beta B. Gamma C. Delta D. Alpha

C. Delta *Afibers are larger-diameter, myelinated, and fast conducting. There are four types of A fibers, including: -Alpha fibers, which conduct information about proprioception and somatic motor control -Beta fibers, which conduct information about touch and pressure -Gamma fibers, which send motor signals to muscle spindles -Delta fibers, which conduct information about fast/sharp/ localized pain, temperature, and crude touch *B fibers are small, myelinated, preganglionic autonomic fibers. They conduct less quickly than A fibers. *C fibers are the smallest and slowest conducting nerve fibers. They are unmyelinated, polymodal fibers that respond to mechanical and thermal stimuli. There are two classifications of C fibers, as follows: -Dorsal root C fibers conduct information about pain, temperature, and reflex responses -Sympathetic C fibers are postganglionic sympathetic nerve fibers

Pruritis is a co-morbid condition with which of the following? A. Arthritis B. Venous thrombosis C. Diabetes mellites D. Low level of thyroid hormone

C. Diabetes mellites *People with diabetes mellitus may also have pruritus on the scalp, feet, ankle trunk, and ganitalia -Pruritis = condition that makes you dry --> person usually itchy and scratchy

You are referred to educate your patient in non-weight bearing gait training secondary to a chronic, non-healing diabetic ulcer on their foot. Diabetic ulcers are associated with peripheral neuropathy and arterial disease, and they are caused by recurring damage to insensitive skin. Which of the following statements is TRUE of diabetic ulcers? A. Diabetic ulcers are painful, but only upon palpation. B. Diabetic ulcers cause severe pain that may increase with limb elevation. C. Diabetic ulcers are not typically painful because sensory loss is generally present. D. Diabetic ulcers can be painful depending upon their stage of severity.

C. Diabetic ulcers are not typically painful because sensory loss is generally present.

You are referred to educate your patient in non-weight bearing gait training secondary to a chronic, non- healing diabetic ulcer on their foot. Diabetic ulcers are associated with peripheral neuropathy and arterial disease, and they are caused by recurring damage to insensitive skin. Which of the following statements is TRUE of diabetic ulcers? A. Diabetic ulcers are painful, but only upon palpation. B. Diabetic ulcers cause severe pain that may increase with limb elevation. C. Diabetic ulcers are not typically painful because sensory loss is generally present. D. Diabetic ulcers can be painful depending upon their stage of severity.

C. Diabetic ulcers are not typically painful because sensory loss is generally present. *Diabetic ulcers are not typically painful even when undergoing palpation --> sensory loss is generally present. *Common for sepsis to be present in a diabetic ulcers and gangrene *Arterial ulcers = severe pain increase with elevated limbs -Small or round -Lack granulation -Distal limb and/or foot especially the toes *Venous ulcers = aching pain

Which of the following options is a common symptom of acute flaccid myelitis (AFM)? A. Paresthesia B. Post-exertional malaise (PEM) C. Difficulty swallowing D. Limb weakness progressing to arm or leg paralysis

C. Difficulty swallowing *Acute flaccid myelitis (AF) is a rare and polio-like condition that affects the motor neurons in the gray matter of the spinal cord. Symptoms include the sudden onset of arm or leg weakness but not the progression to paralysis. Some patients may also have difficulty with swallowing or speech. *Post-exertional malaise (PEM) is a symptom of myalgic encephalomyelitis/chronic fatigue syndrome (ME/CFS). *Paresthesia (abnormal sensations) and limb weakness progressing to paralysis are possible symptoms of transverse myelitis (TM).

In order to determine the percentage of blood emptied from the ventricle during the systole phase, a therapist must calculate the ejection fraction. What formula will be used to identify this value? A. Multiply the heart rate by the stroke volume B. Divide the cardiac output by the body surface area C. Divide the stroke volume by the left ventricular end-diastolic volume D. Divide the stroke volume by the right ventricular end-diastolic volume

C. Divide the stroke volume by the left ventricular end-diastolic volume *A normalejection fraction is greater then 55% *To calculate the cardiac index, you would divide the cardiac output by the body surface area. To calculate cardiac output, multiply the heart rate by stroke volume.

Which of the following statement is NOT true about Glenn-humeral rhythm? A. When arm is abducted above 70* actively sterno and acromioclavicular elevation is brought about by serratus anterior and trapezius B. Extreme abduction of both upper extremities is a resultant of cervico- thoracic spine extension. C. Elevation till 60* is accompanied by flexion, abduction and internal rotation of glenohumeral joint D. Final degrees of elevation is contributed mainly contributed by costo-spinal extension

C. Elevation till 60* is accompanied by flexion, abduction and internal rotation of glenohumeral joint

A physical therapist inspects the progress of a partial-thickness wound on a patient's anterior forearm. The therapist notes evidence of resurfacing of the wound with notable changes in the edges of the wound. This observation is MOST consistent with which of the following conditions? A. Maceration B. Granulation C. Epithelialization D. Infection

C. Epithelialization

A gastrointestinal condition when the lower esophageal sphincter remains constricted, and food gets trapped inside the esophagus is called what? A. Gastroenteritis B. Constipation C. Esophageal achalasia D. Pyloric stenosis

C. Esophageal achalasia

What statement is incorrect regarding spinal nerves? A. Efferent (motor) fibers appear in the ventral root. B. Ventral nerve fibers include preganglionic ANS fibers. C. Every pair of spinal nerves has both a ventral and a dorsal root. D. There are 31 pairs of spinal nerves.

C. Every pair of spinal nerves has both a ventral and a dorsal root. *C1 does not have a dorsal root -Most spinal nerves have both ventral and dorsal roots -31 spinal nerves

A 52 year old male patient with 8 year history of diabetes and controlled hypertension is undergoing an exercise program at an outpatient facility. He is on atenolol since last 6 years. What advice would be the most appropriate for this patient? A. Raise intensity of exercise if pre-exercise blood sugar is above 300 mg/dl B. Take atenolol before exercise C. Exercise in the morning instead of evening D. Refrain from drinking water before exercise

C. Exercise in the morning instead of evening *It is not advisable to exercise if the blood sugar is above 300 mg/dl (Safe = 100-250 / Caution zone = 250-300) *Avoid exercise altogether above 250 mg/dl is ketosis is present *For diabetics 16 oz fluid is important before exercise

Which of the following tests is an example of a functional gait test? A. Four Square Step Test B. MiniBest Test C. Five Times Sit-to-Stand D. Romberg

C. Five Times Sit-to-Stand

A patient who sustained a CVA four weeks ago is beginning to show the ability to produce movement patterns outside of limb synergies. According to Brunnstrom, this patient is in which stage of recovery? A. Two B. Three C. Four D. Six

C. Four

A patient with Duchenne muscular dystrophy exhibits exaggerated lumbar lordosis and genu recurvatum. What other finding is the physical therapist MOST likely to identify? A. Impaired hearing B. Dystonia C. Gowers' sign D. Lower extremity muscle contractures

C. Gowers' sign *Creatine kinase (CK-MM) is a cytoplasmic enzyme of muscle that is released with breakdown of muscle tissue in Duchenne muscular dystrophy. One of the first observable signs is muscle weakness. Children attempt to compensate for lower extremity weakness by using Gowers' sign to rise from the floor.

A patient exhibits limited and pain-free active shoulder external rotation, however, passive external rotation is normal. Resistive testing of shoulder external rotation is weak and pain-free. Which of the following conditions is MOST consistent with this patient's clinical presentation? A. Adhesion of the anterior joint capsule B. Fear-related psychological response C. Grade Ill strain of the infraspinatus muscle D. Grade I strain of the shoulder external rotators

C. Grade Ill strain of the infraspinatus muscle *Fear-related psychological response results in a voluntary limitation of active motion and resistive testing. The voluntary limitation of active motion and the lack of effort with resistive testing are often associated with the fear of pain. However, fear will also typically result in a limitation of passive range of motion secondary to muscle guarding. *A strain is an injury involving the musculotendinous unit that involves a muscle, tendon or their attachments to bone. Strains are commonly described by the degree of damage to the musculotendinous unit (grades I-Ill). A grade Ill strain represents complete rupture of the muscle with a palpable defect. Although pain is quite severe when the injury initially occurs, a lack of pain often exists following the acute phase of the injury. The infraspinatus muscle is a primary external rotator of the shoulder. A grade Ill strain of the infraspinatus would most likely present as described in this scenario.

You are treating a soccer athlete who just underwent AC reconstruction after an ACL tear. Hamstring grafts and patella tendon grafts are two options for an anterior cruciate ligament (ACL) reconstruction. Which of the following is TRUE of ACL reconstruction grafts? A. Hamstring grafts generally have more postoperative symptoms B. Hamstring grafts are typically less expensive than patella tendon grafts C. Hamstring grafts generally allow for earlier rehabilitation than patella tendon grafts D. Hamstring grafts are better at preserving postoperative graft tension than patella tendon grafts

C. Hamstring grafts generally allow for earlier rehabilitation than patella tendon grafts *Hamstring grafts generally allow for earlier rehabilitation than patella tendon grafts. Hamstring grafts generally have fewer postoperative symptoms, and allow for a greater return to the pre-injury level of activity. They are generally more expensive than patella tendon grafts. *Patella tendon grafts generally have a faster healing time than hamstring grafts and are also less expensive. Patella tendon grafts are better at preserving graft tension postoperatively than hamstring grafts.

What is the name of the federal law that ensures confidentiality and privacy of health information? A. Health Insurance Privacy and Accountability Action B. Health Information Privacy and Accuracy Act C. Health Insurance Portability and Accountability Act D. Health Information Protection and Assessment Act

C. Health Insurance Portability and Accountability Act

While performing a gait analysis of a patient, the examiner observes a backward bending of trunk during the late stance phase of the right lower extremity. The physiotherapist is most likely to initiate strengthening of? Why? A. Hip Extensors B. Quads C. Hip Flexors D. Hamstrings

C. Hip Flexors (L2-L3 nerve palsy) The compensation is done to passively flex the hip by bending backwards -You will also see this in mid swing

A patient status post CVA exhibits significant weakness in the affected lower extremity. When strengthening the affected abductor muscle group using irradiation, which muscle group on the unaffected lower extremity would the physical therapist utilize to resist a maximal isometric contraction? A. Hip flexor muscles B. Hip extensor muscles C. Hip abductor muscles D. Hip lateral rotator muscles

C. Hip abductor muscles *Following a CVA, irradiation (overflow) techniques may be utilized to increase activity in weakened musculature. This is typically facilitated by providing maximal resistance to the contralateral muscle group to recruit active movement of the weaker musculature. *Resisting the unaffected hip abductor muscles will typically increase muscle tension in the contralateral hip abductors.

A physical therapist participates in a research study that examines body composition as a function of aerobic exercise and diet. Which method of data collection would provide the therapist with the MOST valid measurement of body composition? A. Anthropometric measurements B. Bioelectrical impedance C. Hydrostatic weighing D. Skinfold measurements

C. Hydrostatic weighing *Hydrostatic weighing calculates the density of the body by immersing a person in water and measuring the amount of water that becomes displaced. This measurement technique is considered the criterion or gold standard for determining body composition.

An important part of neurological assessment is examination of muscle tone. Which of the following statements is INCORRECT about spasticity? A. The quicker you stretch the spastic muscle, more resistance is encountered in that muscle. B. Spasticity can arise due to loss of control of spinal reflexes C. Increased muscle tone of grade 4 as per modified ashworth scale (MAS) is termed as rigidity D. Spasticity occurs as a result of pyramidal tract injury

C. Increased muscle tone of grade 4 as per modified ashworth scale (MAS) is termed as rigidity *Rigidity occurs due to extrapyramidal lesions like basal ganglia

A patient uses a self-administered assessment tool as a method to record daily progress. What type of reliability would be the MOST essential using this tool? A. Reliability of parallel forms B. Internal consistency C. Intratester D. Intertester

C. Intratester *Reliability of parallel forms refers to the consistency between results of two tests constructed in the same way from the same content domain. Parallel forms reliability is not an issue in this example because there is only one form of the assessment tool. *Internal consistency reliability refers to the consistency of results across items within a test. The reliability of the instrument is evaluated by estimating how well the different items within the test, which are supposed to reflect the same construct, do yield similar results. This is not of concern in the example. *Intratester reliability refers to the extent to which scores on the tool obtained by the same tester are consistent. The tool should have intratester reliability so that any changes recorded can be attributed to progress in therapy and not to unreliable measurement. *Intertester reliability refers to the extent of agreement of the scores recorded by two or more individuals. Interrater reliability addresses the consistency of the implementation of a rating system.

It is critical that all hospital staff follow safety precautions to minimize infection occurrence and transmission. What statement regarding safety precautions is incorrect? A. Sterilization is an absolute method for killing all microorganisms. B. Disinfection is the process of reducing or eliminating the microorganisms on surfaces, including hands. C. Isopropyl or ethyl alcohol should never be combined with quaternary ammonium compounds. D. Formaldehyde is one example of a disinfectant

C. Isopropyl or ethyl alcohol should never be combined with quaternary ammonium compounds.

A physical therapist performs a clinical examination on a patient who reports pain and paresthesia in the bilateral extremities and both sides of the back. The therapist completes several tests and determines the patient likely has spinal stenosis. What statement is false regarding spinal stenosis? A. Pain increases with walking B. It is appropriate for the physical therapist to perform manual/mechanical traction on this patient C. It is caused by traumatic injury D. The client may benefit from trigger point injections

C. It is caused by traumatic injury *The patient is born with a narrow spinal canal or intervertebral foramen, coupled with hypertrophy of the spinal lamina and facets, as the result of age-related degeneration or disease *The client may benefit from taking NSAIDs, taking muscle relaxants, or receiving trigger point injections. *It is appropriate for the physical therapist to perform manual/mechanical traction on this patient

A patient suffering from severe low-back pain underwent L3-L4 laminectomy 5 days ago. The surgery went without any complication and the patient is to be discharged the following week. Which of the following would be allowed for the patient to do during this period? A. Prone press ups B. Showering C. Log rolling D. Lifting heavy weights

C. Log rolling *Log rolling can be done and taught to avoid rotational forces over the repaired site and can be permitted -Rolling with neutral spine --> which the pt needs to perform the drawing in maneuver and then roll the trunk as a unit -Always follow the surgeon's instructions first for any surgery

A patient presents to physical therapy with reports of pain in the SI joint. What special test can the physical therapist use to identify the functional leg discrepancy that is causing sacroiliac joint dysfunction? A. Gillet's test B. Goldthwait's test C. Long sitting test D. Gaenslen's test

C. Long sitting test *The long sitting test is utilized by physical therapists to identify sacroiliac joint dysfunction that may be caused by a functional leg discrepancy. The long sitting test is deemed positive when limb lengths are reversed between sitting and supine positions. *Gillet's test is performed in order to assess the posterior movement of a patient's ilium in relation to the sacrum. *Gaenslen's test is performed to identify SlJ dysfunction. *Goldthwait's test is performed in order to differentiate between sacroiliac joint dysfunction and lumbar spine dysfunction.

Snowball sampling

recruitment of participants based on word of mouth or referrals from other participants

Which of the following diagnostic procedures measures intracranial pressures and spinal fluid dynamics? A. Electroencephalography B. Doppler ultrasound C. Lumbar puncture D. Positron emission tomography

C. Lumbar puncture *A lumbar puncture is utilized for the measurement of intracranial pressures and spinal fluid dynamics. This procedure is conducted by inserting a spinal needle below L1-L2. It can also be used to inject therapeutic agents and a contrast medium for radiologic examination. *A carotid/vertebral artery doppler ultrasound is useful for imaging the lumen of the carotid or vertebral arteries to assess patients at risk of stroke. *Positron emission tomography (PET) can be used for physiological mapping and brain metabolism analysis. *Electroencephalograph provides structural brain disease information, which is useful in assessing and locating seizures.

A physical therapist performs both volumetric and circumferential measurements on a patient's right upper extremity. Which medical diagnosis would MOST warrant the use of the described measures? A. Adhesive capsulitis B. Biceps rupture C. Lymphedema D. Shoulder dislocation

C. Lymphedema

What is sanguinous exudate?

red in color and indicates a bloody discharge which may be indicative of either new blood vessel growth (normal healing tissue) or a disruption of blood vessels (abnormal).

Which of the following is a state and federal health care program that serves the poor, elderly, and disabled, regardless of age? A. Workers' compensation B. Medicare C. Medicaid D. Health savings account

C. Medicaid *Medicaid is a health care program that is administered by both the state and federal governments. The Medicaid health care services program was mandated by Title XIX of the Social Security Act. *Medicare is a payer for health care services that are administered by the federal government and is for individuals 65 years and older. Workers' compensation is a third party payer for health care services that is administered by self-insured employers, private insurers, or various state agencies and meant for individuals injured on the job. A Health Savings Account (HSA) is a tax-free savings account that can be used to pay for health-related expenses. In order to have a health savings account, a person must have a high-deductible health plan.

A physical therapist reviews lab values in a patient's medical chart and notes that the patient has a blood pH of 7.25 and HCO3- of 20 mEq/L. The patient has had severe diarrhea for seven days and is currently presenting with tachypnea and lethargy. Which of the following diagnoses is MOST consistent with the described clinical presentation? A. Respiratory acidosis B. Respiratory alkalosis C. Metabolic acidosis D. Metabolic alkalosis

C. Metabolic acidosis *The normal pH of the blood is between 7.35 and 7.45. Acidosis is characterized by a blood pH of less than 7.35. Alkalosis is characterized by a blood pH of greater than 7.45. Acidosis and alkalosis can have either metabolic or respiratory causes.

A physical therapist performs circumferential measurements on a four-year-old child and notes edema in both lower extremities. Based on the clinical presentation, which diagnosis would be MOST likely? A. Lymphadenitis B. Deep vein thrombosis C. Milroy's disease D. Wilson's disease

C. Milroy's disease *Lymphadenitis is characterized by infection and inflammation of a lymph node that is usually caused by a bacterial or viral infection elsewhere in the body. This condition would not result in significant changes in the girth of a limb. *Milroy's disease is an inherited type of primary lymphedema that typically presents in infancy. Bilateral lower extremity edema is the most common symptom of this disease, and it may worsen over time. *Wilson's disease in an autosomal recessive inherited trait that produces a defect in the body's ability to metabolize copper. Symptoms typically appear between the ages of four and six and include Kayser-Fleischer rings surrounding the iris of the eye, degenerative changes in the brain, hepatitis, cirrhosis of the liver, athetoid movements, and ataxic gait patterns. This condition would not result in edema in the extremities.

A physical therapist performs a pain assessment on a patient recently referred to physical therapy. The patient reports sharp, localized pain that is consistently reproduced with mechanical testing of the painful area. Which pain classification is MOST consistent with the patient's clinical presentation? A. Nociceptive: deep somatic pain B. Nociceptive: visceral pain C. Nociceptive: superficial pain D. Central neurogenic pain

C. Nociceptive: superficial pain *Deep somatic pain is found with damage to bone, muscle, connective tissues, and blood vessels. cramping, pressing, and aching. *Superficial or cutaneous pain is found with damage to the skin and subcutaneous tissues. stabbing, sharp, or sometimes a constant dull ache.

Which of the following does NOT accurately characterize stage 1 of lymphedema? A. Decrease in edema with elevation of the lumb overnight B. Accumulation of protein-rich fluid in the limb or area of the trunk with pitting edema present C. Non-pitting edema into the limb D. Increase in edema with activity, heat, or humidity

C. Non-pitting edema into the limb

The lymphatic system helps filter blood going to many places in the body. There are two ducts in the lymphatic system. Where does the left lymphatic duct help drain fluid? A. Pleural cavity B. Left arm C. None of these D. Face

C. None of these *None of these areas are drained by the left lymphatic duct because there is no left lymphatic duct. The right lymphatic duct drains the right upper extremity, the right sides of the upper torso, neck, and face. The rest of the body is drained by the thoracic duct.

The lymphatic system helps filter blood going to many places in the body. There are two ducts in the lymphatic system. Where does the left lymphatic duct help drain fluid? A. Pleural cavity B. Left arm C. None of these D. Face

C. None of these *The right lymphatic duct drains the right upper extremity, the right sides of the upper torso, neck, and face. The rest of the body is drained by the thoracic duct.

Part A of the Medicare services provided by the federal government, do not cover which of the following? A. Hospice care for life-limiting illness. B. IPD care C. OPD visit to PCP D. Nursing facility care for elders

C. OPD visit to PCP *Part A of Medicare is considered hospital insurance that does not cover the primary care physician's visits. *Part A includes IPD services, hospice care, and nursing care facility expanse. *The PCP visits are covered in Part B which is called health insurance and includes OPD services.

Part A of the Medicare services provided by the federal government, does not cover which of the following? A. Hospice care for life-limiting illness B. IPD care C. OPD visit to PCP D. Nursing facility care for elders

C. OPD visit to PCP *The PCP visits are covered in Part B which is called health insurance and includes OPD services

A physical therapist reviews the current pain management protocol of a patient three days post total hip arthroplasty. Which of the following medications would the patient MOST likely have been prescribed for breakthrough pain as needed? A. Acetaminophen (1,000 milligrams) B. Meloxicam (15 milligrams) C. Oxycodone (5 milligrams) D. Tramadol (100 milligrams)

C. Oxycodone (5 milligrams) *Breakthrough pain is a sudden and brief flare-up of pain that becomes severe enough to "break through" the management of pain from the current regular dosing regimen of pain medications. Only lasts a short period of time. *Meloxicam is a nonsteroidal anti-inflammatory drug (NSAID) that is used to treat pain and inflammation often caused by arthritis. Typical doses of this NSAID are 7.5 to 15 mg per day. *Tramadol is a narcotic-like medication used to treat moderate to severe pain. A typical dosage of tramadol for post-operative pain management would be 100 mg every four to six hours with a maximum dosage of usually 400 mg per day.

A patient explains to a physical therapist that she was instructed to bear up to ten pounds of weight on her involved extremity. Which description is MOST consistent with the patient's weight bearing status? A. Non-weight bearing B. Toe touch weight bearing C. Partial weight bearing D. Weight bearing as tolerated

C. Partial weight bearing *Toe touch weight bearing occurs when a patient is unable to place any weight through the involved extremity, however, may place the toes on the ground to assist with balance. *Partial weight bearing occurs when a patient is allowed to put a particular amount of weight through the involved extremity. The amount of weight bearing is expressed as allowable pounds of pressure or as a percentage of total weight. Partial weight bearing requires an assistive device.

A physical therapist is responsible for maintaining accurate patient medical records. Regarding medical record documentation, which of the following statements is TRUE? A. Mistakes should be erased completely. B. Computer signatures are unacceptable, because they can be easily hacked. C. Patients are referred to by unique identifiers and by name. D. By federal mandate, records must be kept safely and securely for 7 years.

C. Patients are referred to by unique identifiers and by name.

A physical therapist collects data as part of a preseason athletic screening program designed to identify individuals susceptible to heat illness. Which of the following measures would be the MOST valuable to collect? A. Height B. Weight C. Percent body fat D. Vital capacity

C. Percent body fat

A physical therapist performs a certain knee assessment on a patient. They demonstrate a positive finding, which is characterized by ligament laxity as seen by tibia relocation. What knee test was just described? A. Thessaly test B. Posterior sag test C. Pivot shift test D. 1-plane anterior instability test

C. Pivot shift test * The positive finding of the pivot shift test is ligament laxity, as demonstrated by the patient's tibia relocating during the test. The pivot shift test is used to indicate anterolateral rotary stability. *The positive finding of the posterior sag test is sag of the tibia that is associated with the femur. The positive finding of the Thessaly test is the reproduction of a click and/or pain. *The positive finding of the 1-plane anterior instability test is excessive posterior translation of the tibia compared to the uninvolved limb.

A patient following total hip arthroplasty is placed on Heparin. Which of the following laboratory values would be the MOST affected based on this medication? A. Hematocrit B. Hemoglobin C. Prothrombin time D. White blood cell count

C. Prothrombin time

Upon examining a patient's wound drainage, you detect that the drainage contains green-colored pus. How would you describe this drainage in your documentation? A. Sanguineous B. Serous C. Purulent D. Purulous

C. Purulent *Purulent wound drainage is a milky drainage, usually thick, that can be gray, green, or yellow *Serous drainage is a thin, clear, and watery serum *Sanguineous drainage contains blood and is prevalent among deeper wounds. *"Purulous drainage" is not a commonly used term.

A physical therapist examines a patient following a traumatic crush injury to the forearm. The patient displays a wrist drop deformity and is unable to strongly grasp objects with their hand. Which nerve was MOST likely affected? A. Median nerve B. Ulnar nerve C. Radial nerve D. Musculocutaneous nerve

C. Radial nerve *Wrist drop deformity is characterized by an inability to actively extend the wrist and fingers, resulting in the wrist being flaccidly held in a flexed position

A therapist is working in a home health setting with many patients undergoing cancer treatments. Therefore, he must be aware of the side effects of cancer treatments. Which type of cancer treatment is LEAST LIKELY to cause nausea, vomiting, and/or anorexia? A. Immunotherapy B. Chemotherapy C. Radiation D. Hormonal therapy

C. Radiation

A physical therapist determines that a patient needs a spinal orthosis to control lower back motion in order to treat stenosis. Which of the following spinal orthoses would the physical therapist MOST LIKELY recommend to the patient? A. Thoracolumbosacral orthosis B. Flexible lumbosacral orthosis C. Rigid lumbosacral orthosis D. Minerva orthosis

C. Rigid lumbosacral orthosis *Lumbosacral orthoses (LSOs) control lower back motion or lumbosacral motion. Rigid SOs are used for postoperative protection, stenosis, low back pain, and spondylolisthesis. *A flexible LSO is indicated for low back pain, compression of abdominal incisions, or for sacroiliac support during pregnancy. *A thoracolumbosacral orthosis controls thoracic and lumbosacral motions, which would be too restrictive for the patient in this example. *A Minerva orthosis is a cervical orthosis, and would be inappropriate for this patient.

Therapists completing wound care for patients might use foams to assist with the treatment of full- and partial-thickness wounds as well as wounds with slough and necrotic tissue. What is the best way to describe foams? A. Adhesive wafers that form a gelatinous mass over the wound B. Semipermeable membranes that are always hydrophobic, with varying adhesive and absorptive properties C. Semipermeable membranes that are either hydrophobic or hydrophilic, with varying adhesive and absorptive properties D. Water based gels contained within impregnated gauze

C. Semipermeable membranes that are either hydrophobic or hydrophilic, with varying adhesive and absorptive properties *Hydrocolloids are adhesive wafers that form a gelatinous mass over the wound. Hydrogels are water-based gels contained within impregnated gauze.

A patient is in her last trimester of pregnancy complains of a burning sensation in the stomach rising up to her chest and frequent vomiting is diagnosed with GERD. What advice would you prefer to give to her to reduce her symptoms? A. Supine lying for 10 minutes before meals B .Side lying on the right side C. Side lying on the left side D. Supine lying with a large towel roll under the left side.

C. Side lying on the left side *When L sidling, the lower esophageal sphincter is placed superior to the stomach anatomically and can help to control acid refluxes and prevent regurgitation *Supine is before meals in GERD--> more than 5 min after eating can increase pressure on the Inferior vena cava

A patient reports muscle pains of excess severity in relation to their current activity level and the interventions provided. Which of the following medications would MOST likely be associated with this finding? A. Amlodipine (Norvasc) B. Albuterol (Ventolin) C. Simvastatin (Zocor) D. Methotrexate (Rheumatrex)

C. Simvastatin (Zocor) *Amlodipine (Norvasc) is a calcium channel blocker agent used to treat hypertension, arrhythmias, and angina. Side effects associated with calcium channel blockers include dizziness, hypotension, headache, and peripheral edema. *Albuterol (Ventolin) is a short-acting beta-adrenergic (SABA) bronchodilator used to treat bronchospasm. SABA bronchodilators are typically used as a rescue medication to relieve shortness of breath associated with asthma and COPD. Common side effects associated with albuterol (Ventolin) include nervousness, restlessness, and trembling. Myalgia is not an adverse effect associated with SABA bronchodilators. *Simvastatin (Zocor) is an antihyperlipidemia agent that inhibits cholesterol synthesis, breaks down low-density lipoproteins, lowers triglyceride levels, and increases high-density lipoprotein levels. In addition to myalgia, other side effects associated with statins include headache, gastrointestinal distress, and rash. *Methotrexate (Rheumatrex) is a disease-modifying antirheumatic drug (DMARD) used to slow or halt the progression of rheumatic disease, preferably during early treatment. Common side effects of DMARDs include gastrointestinal distress, toxicity, liver dysfunction, mouth ulcers, and hair loss. Myalgia is not an adverse effect associated with methotrexate (Rheumatrex).

A physical therapist writes in a patient's therapy note that they have a left leg laceration. What is the most accurate way to describe this skin trauma? A. Skin that is scraped away due to an injury B. Skin that is torn causing a wound with clear, discrete edges C. Skin that is torn, causing a jagged wound D. Skin that has a bluish discoloration due to the extravasation of blood

C. Skin that is torn, causing a jagged wound

Which of the following is LEAST likely to show an 'empty' end-feel? A. Psychogenic disorder B. Acute Inflammation C. Soft tissue edema D. Scaphoid fracture

C. Soft tissue edema *A soft end feel occurs sooner or later in a ROM than usual or firm or hard *Mushy tissue stretch = tight muscles

A woman presents to a women's health therapy clinic with stress urinary incontinence. This condition involves the unexpected and rapid release of urine from the bladder. What is a potential cause of stress urinary incontinence? A. Hyperreflexia B. Hypersensitive bladder C. Sphincter weakness D. Neurogenic bladder

C. Sphincter weakness *Sphincter weakness = stress urinary incontinence. Other causes associated with stress urinary incontinence include intra-abdominal pressure and weakness of pelvic floor musculature. *Hyperreflexia and hypersensitive bladders = urge urinary incontinence. *A neurogenic bladder may cause overflow incontinence.

A physical therapist is performing a neurological examination to determine a patient's level of consciousness. The patient can only be aroused from sleep using painful stimuli, responds slowly, and returns to an unresponsive state once the stimuli have been removed. What level of consciousness is this? A. Obtundation B. Minimally conscious state (MCS) C. Stupor D. Coma

C. Stupor *A minimally conscious state (MCS) is a level of consciousness characterized by severely altered awareness with minimal but definite evidence of self or environmental awareness. *Obtundation is a level of consciousness described as a confused state in which the patient can open their eyes and look at the examiner, but they respond slowly and are confused.

You are conducting a PT evaluation for a pt that has experienced an electrical burn. The burn wound appears charred, and you see that the epidermis, dermis, subcutaneous tissues, and muscles have been destroyed. How should this wound be classified? A. Superificial partial-thickness burn (second degree) B. Epidermal burn (first-degree) C. Subdermal burn (fourth degree) D. Deep partial-thickness burn (fourth-degree)

C. Subdermal burn (fourth degree)

A pt suffering from neck pain for the past 3 months visits an in-patient care. After a detailed history, pt mentions that the pain radiates to the last 3 fingers of the R extremity especially after sitting for an hour continuously. You decide to apply intermittent traction. Which position is MOST suitable for this patient? A. Supine position with neck slightly extended B. Supine position next neural C. Supine position with neck flexion D. Intermittent traction is contra-indicated in this patient

C. Supine position with neck flexion

A flat foot deformity will present with the following abnormal positions at the foot EXCEPT? A. Forefoot abduction B. Lateral drift of calcaneus C. Talar dorsiflexion D. Dorsiflexed metatarsals

C. Talar dorsiflexion

A physical therapist is evaluating a patient with complaints of pain and stiffness at the base of their thumb and difficulty grasping objects. The physical therapist decides to perform Finkelstein's test and gets a positive indication of pain. How is Finkelstein's test performed? A. The patient's wrist is hyperflexed with the thumb abducted in full MCP and IP extension, and resistance is applied against the examiner's index finger. B. The patient makes a fist with the thumb flexed within the confines of the fingers; the examiner passively moves the wrist into ulnar deviation. C. The examiner passively pulls the wrist and thumb into ulnar deviation and applies longitudinal traction. D. The examiner places pressure on the distal pole of the scaphoid while radially deviating and slightly flexing the patient's hand.

C. The examiner passively pulls the wrist and thumb into ulnar deviation and applies longitudinal traction.

A physiotherapist wants to check a patient's transverse ligament and performs the modified sharp purser test. Which of the following is false about this test? A. It tells if the transverse ligament is intact and the atlantoaxial joint is normally stable. B. Excessive posterior translation indicates that the test is positive. C. The inability to palpate C2 movement compared to C1 is the positive test. D. The C2 spinous process is moved with a firm pincer grip.

C. The inability to palpate C2 movement compared to C1 is the positive test. *C2 movements are not palpated in conjunction with the C1 movement, it is a positive test for the alar ligament. Whereas the modified sharp purser test is used to check the integrity of the transverse ligament and the atlantoaxial stability by moving the C2 with a firm pincer grip.

Hypoadrenalism (Addison's disease)

results from failure of the adrenal cortex to produce adenocortical hormones due to atrophy or TB related destruction. Sx: fatigue, anorexia, weight loss, nausea & vomiting. Aldosterone deficiency: Regulates absorption of Na - without it, hypotension. Cortisol deficiency - Altered blood glucose levels

A physical therapist is working with a patient in an outpatient setting when the patient describes that they have been consistently dehydrated after sleeping, despite drinking water throughout the day. In addition to referring the patient to a specialty doctor, the therapist advises the patient to focus on breathing through their nose more. What is the MOST likely reason that the therapist has made this recommendation? A. Nose breathing increases thirst B. Mouth breathing uses more energy C. The nose humidifies air D. Mouth breathing decreases thirst

C. The nose humidifies air *The nose is an upper airway structure that is responsible for filtering, humidifying, and warming air. The nose is an entry point for air, and it allows air to enter into the respiratory system. The other entry point is the mouth. Because mouth breathing would result in less humidity of inspired air, it would be advisable for the patient to breathe through their nose more to improve hydration. In most cases, nose breathing decreases thirst and mouth breathing increases thirst.

During an assessment of pain, the therapist should ask questions such as "Describe your pain and symptoms from the time you wake up through the time that you go to bed" in order to allow the patient to describe their pain. The pattern of pain can assist healthcare professionals to recognize the type of pain that the patient is experiencing and design the treatment accordingly. A. A MSK pattern of pain is typically characterized by adjectives such as aching, heavy, cramping, dull and sore B. A neurogenic pattern of pain is typically characterized by adjectives such as burning, hot, searing, sharp, and shooting C. The patient's description is best indicates a vascular pattern of pain. Other terms that typically characterized a vascular pattern include pulsing and beating D. An emotional pattern of pain is typically characterized by adjectives such as miserable, piercing, unbearable, nauseating, and agonizing.

C. The patient's description is best indicates a vascular pattern of pain. Other terms that typically characterized a vascular pattern include pulsing and beating

A physical therapist is preparing for an evaluation with a patient in a skilled nursing facility. During the chart review, the therapist notes a condition related to the patient's sebaceous glands. During the physical exam, where will the physical therapist be LEAST likely to observe this condition? A. The patient's posterior thigh B. The patient's axilla C. The patient's palms D. The patient's forehead

C. The patient's palms

Physical therapists are responsible for controlling infections by following appropriate sterilization and disinfection quidelines. Which of the following statements is TRUE of chemical disinfectants? A. They are not always microbicidal B. They effectively eliminate spores C. They are the most commonly used types of disinfectants D. They must be approved by the CDC

C. They are the most commonly used types of disinfectants *Chemical disinfectants, like chlorine, hydrogen peroxide, and formaldehyde, are the most commonly used types of disinfectants. *Chemical disinfectants do not effectively eliminate spores. In order to achieve this, the object must be sterilized, not disinfected. *Chemical disinfectants are microbicidal, and their usage must be approved by the Environmental Protection Agency (EPA).

A patient has suffered a tibial fracture and is now in a hard cast and has been ordered for non-weight bearing (NWB) status. He is using Lofstrand crutches as an ambulatory aid. Which of the following statements is NOT TRUE of Lofstrand crutches? A. They provide a way to relieve specific weight-bearing on the lower extremities. B. They provide less stability than some other types of crutches. C. They pose a risk of nerve damage with overuse. D. They free the patient's hands for use.

C. They pose a risk of nerve damage with overuse.

A physical therapist treats a patient post total knee arthroplasty in an outpatient clinic. The patient is able to complete a full revolution on a stationary bike, however, the therapist would like the patient to improve their knee flexion range of motion in order to eliminate the use of compensatory strategies on the bike. Which patient would be the BEST candidate for this intervention based on the therapist's stated objective? A. One week post-operative, 85 degrees passive knee flexion B. Two weeks post-operative, 95 degrees passive knee flexion C. Three weeks post-operative, 105 degrees passive knee flexion D. Four weeks post-operative, 125 degrees passive knee flexion

C. Three weeks post-operative, 105 degrees passive knee flexion *Approximately 110 degrees of knee flexion is necessary to comfortably complete full revolutions without the need for compensatory strategies.

Type A nerve fibers are large, fast-conducting nerve fibers. Which of the following is associated with A-beta fibers? A. Pain B. Proprioception C. Touch D. Motor to muscle spindles

C. Touch *A fibers are larger-diameter, myelinated, and fast conducting. There are four typesof A fibers, including: 1. Alpha fibers, which conduct information about proprioception and somatic motor control 2. Beta fibers, which conduct information about touch and pressure 3. Gamma fibers, which send motor signals to muscle spindles 4. Delta fibers, which conduct information about fast/sharp/ localized pain, temperature, and crude touch *B fibers are small, myelinated, preganglionic autonomic fibers. They conduct less quickly than A fibers.

Which of the following is an inflammatory bowel disease? A. Diverticulosis B. IBS C. Ulcerative colitis D. Peptic ulcers

C. Ulcerative colitis *Inflammatory bowel disease includes Ulcerative colitis and Crohn's disease that are characterized by inflammation of the bowel that has exacerbation and remission periods.

A physical therapist determines through examination that a patient exhibits Poor (2/5) strength of the adductor pollicis and a positive Froment's sign. The therapist should MOST expect a palsy involving which of the following nerves? A. Median B. Anterior interosseous C. Ulnar D. Posterior interosseous

C. Ulnar *When assessing for Froment's sign, the patient is asked to hold a piece of paper between their thumb and index finger while the therapist attempts to pull the paper away from the patient. A positive Froment's sign is indicated by the patient flexing the distal phalanx of the thumb due to weakness or paralysis of the adductor pollicis muscle. The adductor pollicis is innervated by the deep motor branch of the ulnar nerve. *The ulnar nerve innervates the flexor carpi ulnaris and the medial half (i.e., digits 4-5) of the flexor digitorum profundus. The deep (motor) branch of the ulnar nerve innervates the hypothenar muscles, the interossel muscles, the two medial lumbricals, the adductor pollicis muscle, and the deep head of the flexor pollicis brevis. Weakness of the adductor pollicis and a positive Froment's sign are typical signs of an ulnar nerve palsy.

A physical therapist is providing gait training to a 30-year-old female patient in the hospital. During the session, the patient complains of muscle cramps and spasms. Upon returning to the room and reviewing the patient's most recent test results, the therapist notes a calcium serum level of 6.8 mg/dL. Of the following, what is the MOST likely explanation for these findings? A. Vitamin D intoxication B. Hyperthyroidism C. Vitamin D deficiency D. Hyperparathyroidism

C. Vitamin D deficiency

A patient who experienced a stroke is now in the third stage of rehabilitation. What happens during this stage? A. Control and coordination near normal B. Voluntary control in isolated joint movements C. Voluntary movement possible, but only in synergies D. Emergence of spasticity

C. Voluntary movement possible, but only in synergies *The stages of sequential recovery from stroke (CVA) are as follows: -Stage 1: initial flaccidity; no voluntary movement -Stage 2: emergence of spasticity, hyperreflexia, synergies -Stage 3: voluntary movement possible, but only in synergies; spasticity strong -Stage 4: voluntary control in isolated joint movements emerging; corresponding decline of spasticity and synergies -Stage 5: increasing voluntary control out of synergy; coordination deficits present -Stage 6: control and coordination near normal

A pt met with a road traffic accident has been admitted to in an in-patient setting. X-ray reveals the presence of a fracture of neck of humerus. Which impairment will be most likely to be present in this patient? A. Winging of medial border of scapula B. Inability to initiate shoulder abduction C. Weakness during external rotation of shoulder D. Weakness during internal rotation of shoulder

C. Weakness during external rotation of shoulder *Medial border winging = serratus anterior weakness = long thoracic nerve palsy

When treating a patient with lymphedema, which of the following is INAPPROPRIATE for the therapist to recommend? A. Meticulous skin care, including the application of moisturizer B. Getting more sun to increase vitamin D C. Weight lifting D. Certain massage techniques

C. Weight lifting *High-exertion exercise, like weight lifting, may damage the balance of the lymph system and interstitial pressure, contributing to an increase in swelling in the limb.

Which of the following options is a common symptom of acute diabetes mellitus? A. Peptic ulcers B. Excessive sweating C. Weight loss D. Atherosclerosis

C. Weight loss

When using superficial thermotherapy, physical therapists can use a convection heat transmission modality. Which of the following is an example of a convective modality? A. Hot packs B. Infrared lamp C. Whirlpool D. Paraffin

C. Whirlpool *Whirlpools (hydrotherapy) are convective modalities. Convection is the transfer of heat using the movement of air or fluid from a warmer area to a cooler area. An infrared lamp is an example of radiation heat transmission. Hot packs and paraffin are examples of conductive heat transmission.

If a therapist wants to perform sensory testing on a patient, which of the following should the therapist do to check graphesthesia? A. Ask the patient to identify an object with eyes closed. B. Ask the patient to identify two different types of touch sensations. C. Write an M on the patient's palm and ask the patient to identify the letter with eyes closed. D. Press the patient's superciliary arch and ask if he feels pain.

C. Write an M on the patient's palm and ask the patient to identify the letter with eyes closed.

What is ambulatory care?

Care that does not require an overnight stay in a facility -Outpatient Care

What is Cervical facet impingement ("facet syndrome"')?

Characterized by inflammation and pain in one or more of the facet joints of the cervical spine. The facet (zygapophyseal) joints are the true synovial intervertebral joints with a joint capsule that can become impinged. Due to the natural coupling motion in the spine, painful and limited rotation and lateral flexion of the neck to the same side is typically seen in cervical facet impingement.

What is the most appropriate scenario to refer a patient with chest pain on the left side for 1 year to a physician or emergency room?

Chest pain radiating to the left shoulder and jaw Increase in chest pain after stopping rehab exercises Upper back feeling abnormally cool and sweaty on touch than usual

What is serous exudate?

Clear or light color fluid with a thin, watery consistency. This particular type of exudate is normal during the inflammatory and proliferative phases of healing.

Cardiac tamponade

Compression of the heart as the result of buildup of blood or other fluid in the pericardial sac, leading to decreased cardiac output.

Tetralogy of Fallot (TOF)

Congenital malformation involving four distinct heart defects -Ventricular septal defect (VSD) -Pulmonary Stenosis -Right ventricular hypertrophy -Aorta overriding the ventricular septal defect *Usually diagnosed during infancy, but may not be detected until later in life *Signs = cyanosis, SOB, rapid breathing, fainting, clubbing fingers and toes, poor weight gain, fatigue, irritability, and heart murmur

Blocked practice

Consistent practice of a single task -the patient would perform the same single task repeatedly to improve

A patient presents to physical therapy for treatment of a non-operative osteoporotic fracture. Which of the following structures would have been LEAST likely affected? A. Distal radius B. Vertebral body C. Proximal femur D Tibial plateau

D Tibial plateau *Vertebral compression fractures are the most common type of fracture in patients with osteoporosis.

What is the classification of the burn if the wound is white, wet, and waxy in texture? The wound also blanches and has a slow capillary refill rate. A. 1st degree burn (only epidermis) B. 2nd degree (only epidermis) C. Full-thickness burn D. 2nd degree (epidermis and dermis damaged)

D. 2nd degree (epidermis and dermis damaged) *2nd degree burn is blanch able on slight touches and involves the epidermis, dermis, sweat glands, hair follicles, and nerve endings

Hypoglycemia is a precaution for patients with diabetes. In particular, those with diabetes are at risk for hypoglycemia during exercise. What is a safe blood glucose range for diabetic patients to maintain during exercise programs? A. 60 mg/dL B. 90 mg/dL C. 75 mg/dL D. 70 mg/dL

D. 70 mg/dL

Hospital staff and therapists often use endotracheal suctioning to clear a patient's airways when they are having trouble breathing. What patient would be the most suitable candidate for this treatment? A. An infant who cannot cough on command B. A 50-year-old patient with a spinal cord injury that resulted in weakened abdominal muscle strength C. A 75-year-old patient with COP who has difficulty clearing his secretions by coughing D. A patient with an endotrache al tube who is in a coma state after a motor vehicle accident

D. A patient with an endotrache al tube who is in a coma state after a motor vehicle accident

Cor pulmonale

right ventricular hypertrophy and heart failure due to pulmonary hypertension

Hospital staff and therapists often use endotracheal suctioning to clear a patient's airways when they are having trouble breathing. What patient would be the most suitable candidate for this treatment? A. An infant who cannot cough on command B. A 50-year-old patient with a spinal cord injury that resulted in weakened abdominal muscle strength C. A 75-year-old patient with COPD who has difficulty clearing his secretions by coughing D. A patient with an endotracheal tube who is in a coma state after a motor vehicle accident

D. A patient with an endotracheal tube who is in a coma state after a motor vehicle accident *Endotracheal suctioning is an airway clearance technique that is used on patients who have not been able to clear their airways in any other manner. This technique is performed by manually feeding a catheter through the patient's airway and suctioning for approximately 10 to 15 seconds. *Huffing is an airway clearance technique that is used on patients who have collapsible airways. This is appropriate for the patient with COPD. *An assisted cough is an airway clearance technique that is used on patients who do not have the necessary abdominal muscles to perform an effective cough. This technique is appropriate for the patient with the spinal cord injury. *Tracheal stimulation is an airway clearance technique that is used on patients who do not have the ability to cough on command. This is appropriate for the infant patient.

A physical therapist is working with a patient who has upper motor neuron syndrome, which causes them to experience spasticity. In particular, this patient has shoulder spasticity. What shoulder action is most likely impacted by this symptom? A. Flexion B. Flexion, adduction C. Adduction, internal rotation, extension D. Adduction, internal rotation, depression

D. Adduction, internal rotation, depression *If a patient with upper motor neuron syndrome exhibits hip spasticity, the actions most likely affected are hip adduction, internal rotation, and extension. *If a patient with upper motor neuron syndrome exhibits elbow spasticity, the action most likely affected is flexion. *If a patient with upper motor neuron syndrome exhibits wrist spasticity, the actions most likely affected are flexion and adduction.

A patient with a partial-thickness wound on the sacrum is wearing a hydrocolloid dressing, which is used for wounds of this severity along with those that have mild exudate. What is the best way to define hydrocolloid dressings? A. Semipermeable membranes that may be hydrophilic or hydrophobic B. Water- or glycerine-based gels that are available in solid sheets, amorphous gels, or impregnated gauze C. Adhesive, clear dressings with semipermeable membranes D. Adhesive wafers that form a gelatinous mass over the wound

D. Adhesive wafers that form a gelatinous mass over the wound

A physical therapist works with a patient six weeks following total hip arthroplasty (posterolateral approach). The patient's medical history includes Graves' disease. Which intervention should the therapist avoid when treating the patient? A. Supine resisted hip external rotation B. Standing hip abduction with an ankle weight C. Stair training D. Ambulation in a warm therapy pool

D. Ambulation in a warm therapy pool *Ambulation in a pool or on land would be an appropriate intervention for this patient. However, the warm therapy pool would make this intervention inappropriate since patients with Graves' disease have an accelerated metabolic rate and are often intolerant to warm environments.

Name the hormone that modulates the rate of gastric emptying and is secreted by the pancreatic beta cells. A. Somatostatin B. Insulin C. Glucagon D. Amylin

D. Amylin *Amylin is a pancreatic hormone released by the beta cells of the pancreas that delays gastric emptying and inhibits glucagon secretion. *Glucagon is secreted by alpha cells and increases blood glucose levels. *Somatostatin is released by delta cells and reduces gastric motility along with suppressing insulin and glucagon production.

A 78-year-old patient was just diagnosed with overflow urinary incontinence, which causes continual leakage due to a chronically distended bladder and urinary retention. Which of the following may cause this overflow urinary incontinence? A. Impaired cognition B. Laxity of pelvic floor musculature C. Detrusor muscle instability D. Anatomical obstruction, like from an enlarged prostate

D. Anatomical obstruction, like from an enlarged prostate *Laxity of pelvic floor musculature is a cause associated with stress urinary incontinence. *Detrusor muscle instability is a cause associated with urge urinary incontinence. *Impaired cognition is a cause associated with functional urinary incontinence.

A patient presents with Poor Plus (2+/5) strength of the right lower extremity and Good Plus (4+/5) strength of the right upper extremity. Sensory loss is also noted to be greater in the right lower extremity than the right upper extremity and the patient is demonstrating abulic aphasia. Occlusion of which of the following arteries is the MOST likely cause of the patient's impairments? A. Middle cerebral B. Vertebral-basilar C. Posterior cerebral D. Anterior cerebral

D. Anterior cerebral *The middle cerebral artery is the most common site of a cerebrovascular accident. Occlusion of the middle cerebral artery will typically produce contralateral weakness and sensory loss of the face and upper extremity with lesser involvement in the lower extremity. Dominant hemisphere impairment includes global, Wernicke's or Broca's aphasia. *There are a wide variety of clinical symptoms and syndromes based on the complex vascularity of the vertebral-basilar artery system. Severe impairment can cause locked-in syndrome, coma or vegetative state. Wallenberg syndrome secondary to a lateral medullary infarct presents with a variety of symptoms including facial pain, temperature impairment, ipsilateral ataxia, and vertigo. *Two of the most significant impairments with posterior cerebral artery occlusion are thalamic pain syndrome and cortical blindness. Thalamic pain presents with abnormal sensation of pain, temperature, touch, and proprioception. Cortical blindness is the loss of vision due to damage to the visual portion of the occipital cortex. *Occlusion of the anterior cerebral artery will typically produce contralateral lower extremity motor and sensory involvement, whereas bilateral occlusion will typically produce paraplegia. Other findings include incontinence, abulic aphasia, frontal lobe symptoms such as personality changes, and potential akinetic mutism (i.e., conscious unresponsiveness).

When a patient experiences an arterial ulcer, it is most often found over a bony prominence on the lower leg. What is not a known cause of arterial ulcers? A. Atheroembolism B. Arteriosclerosis C. Chronic arterial insufficiency D. Atherosclerotic peripheral artery disease

D. Atherosclerotic peripheral artery disease *Atherosclerotic peripheral artery disease is associated with causing diabetic ulcers. *Diabetes is associated with arterial disease and peripheral neuropathy. *Arterial ulcers can be caused by atheroembolism, arteriosclerosis, and chronic arterial insufficiency. *Arterial ulcers generally cause severe pain and skin changes, including thin and shiny skin.

A patient who has cerebral palsy exhibits slow, involuntary, continuous writhing movements of the upper and lower extremities. Which of the following motor impairments is MOST representative of this type of motor disturbance? A. Spasticity B. Ataxia C. Hypotonia D. Athetosis

D. Athetosis *Spasticity refers to an increased resistance to passive stretch. Spasticity is commonly observed with patients diagnosed with cerebral palsy due to upper motor neuron damage. *Ataxia is a generalized term used to describe motor impairments of cerebellar origin. It is characterized by the inability to perform coordinated movement and may affect gait, posture, and patterns of movements. *Hypotonia refers to decreased or absent tone where resistance to passive movement is decreased, stretch reflexes are diminished, and limbs are easily moved. Hypotonicity in children is often associated with motor delays. *Athetosis refers to involuntary movements characterized as slow, irregular, and twisting. Peripheral movements occur without central stability. This type of motor disturbance makes it extremely difficult to maintain a static body position.

A part of CDT treatment for patients with lymphedema, a therapist implements compression with bandages. What statement regarding CDT and the use of bandages for lymphedema is not correct? A. Bandages account for 50% of the improvement of symptoms. B. Bandages have low resting pressure. C. These treatments should only be applied by certified specialists. D. Bandages have low working pressure.

D. Bandages have low working pressure

You are working with a patient who has a known history of hypotension. Given the patient's history, which of the following sites should you be the MOST concerned about to examine their pulse safely? A. Femoral artery B. Radial artery C. Brachial artery D. Carotid artery

D. Carotid artery

A 51-year old male presents with gradual onset pain and weakness over the anterior and lateral aspect of right shoulder, occasionally radiating to the upper aspect of arm especially when sleeping on his right side and when he gets up from his sleep. Pain is relieved particularly when he places his arm slightly overhead and finds this position comfortable. What could be the possible diagnosis in this patient? A. Subacromial bursitis B. Rotator cuff tendinitis C. AC joint sprain D. Cervical radiculopathy

D. Cervical radiculopathy *A C5-C6 radiculopathy can present with anterior shoulder pain and upper arm pain with weakness

A 42 year old female is diagnosed with fibromyalgia complains of severe pain in her scapula, neck, elbow whenever she has to clean the house, cook for a long time or play with her kid. Which of the following physiological response is likely to be responsible her pain? A. Increase in her systolic blood pressure B. Decrease in Heart Rate C. Reduced muscle activity D. Decrease in blood supply

D. Decrease in blood supply *In fibromyalgia, during exercise there is a decrease of circulation of the muscles and brain, but an increase in muscle activity

What is the BEST description of hypokalemia? A. Decreased pH (elevated lactic acid) B. Decreased potassium levels in blood and lymph C. Decreased concentration of calcium D. Decreased concentration of potassium ions

D. Decreased concentration of potassium ions *Hypokalemia occurs when an individual exhibits a decreased concentration of potassium ions in the blood. *Hypokalemia can cause arrhythmias and ventricular fibrillation. *Hypocalcemia is the decreased concentration of calcium, and depresses heart actions. *Chemoreceptors are sensitive to decreased pH (elevated lactic acid); this causes an increase in heart rate.

A child with cerebral palsy is evaluated in physical therapy following a selective dorsal rhizotomy procedure. Which outcomes associated with this surgical procedure are MOST beneficial to improve the patient's overall functional status? A. Decreasing sensation and preserving hypertonicity B. Increasing strength and preserving complete sensation C. Increasing sensation and preserving hypotonicity D. Decreasing spasticity and preserving strength

D. Decreasing spasticity and preserving strength *A selective dorsal rhizotomy is a neurosurgical procedure that may be indicated for patients with spasticity when more conservative treatment has failed. This procedure has relatively few operative and postoperative complications and can offer significant functional gains with some patients. *During a selective dorsal rhizotomy procedure, the majority of lower extremity sensory nerve rootlets are cut to reduce afferent information to the reflex arc. The procedure aims to create a proper balance between the elimination of spasticity and preserving adequate strength within the muscles. This balance, along with intensive physical therapy, will allow the patient to improve strength and overall function.

A physical therapist tests a small area of skin for hypersensitivity prior to using a cold immersion bath. The patient begins to demonstrate evidence of cold intolerance within 60 seconds after cold application. Which of the following responses should the therapist take? A. Limit cold exposure to ten minutes or less B. Select an alternate cryotherapeutic agent C. Continue with the cold immersion bath D. Discontinue cold application and document the findings

D. Discontinue cold application and document the findings

A patient two days post arthroscopy of the knee completes a quadriceps setting exercise while lying supine on a mat table. During the exercise, the patient reports they are experiencing severe pain. Which of the following actions is the MOST appropriate for the physical therapist to take? A. Have the patient perform the exercise in sidelying B. Have the patient flex the knee prior to initiating the exercise C. Place a pillow under the ankle D. Discontinue the exercise

D. Discontinue the exercise *Quad setting exercise = isometrics

A pt refuses PT services after being transported to the gym. The PT explains the potential consequences of refusing treatment, however, the pt does not reconsider. What is the MOST appropriate initial therapist action? A. Treat the patient B. Convince the patient to have therapy C. Contact the referring physician D. Document the incident in the medical record

D. Document the incident in the medical record

Which of the following is false regarding acute bacterial prostatitis? A. Feeling of malaise B. Discharge from urethra C. Symptoms of fever D. ED

D. ED *Acute bacterial prostatitis can cause fever and chills, malaise, and urethral discharge *ED is commonly caused by acute nonbacterial inflammatory prostatitis

Estrogen is a female sex hormone produced in the ovaries. Which of the following statements about estrogen is TRUE? A. Estrogen decreases high-density lipoproteins. B. Estrogen is produced in the ovaries, under the control of the posterior pituitary gland. C. Estrogen decreases thyroid production. D. Estrogen decreases the rate of bone resorption.

D. Estrogen decreases the rate of bone resorption.

You are a physical therapist conducting an evaluation for a patient with left shoulder pain in an outpatient facility. During the assessment, you observe that the patient is lacking full shoulder elevation on their left side compared to their right. Which of the following actions is MOST likely to be limited? A. Scapular adduction B. Rotation of the clavicle at the acromioclavicular joint C. Internal rotation of the humerus D. External rotation of the humerus

D. External rotation of the humerus *External rotation of the humerus is a biomechanical requirement in order to achieve full shoulder elevation. Because this patient is lacking full shoulder elevation, this is the most likely limited action in this scenario. The requirements are as follows: -Scapular stabilization -Inferior glide of the humerus -External rotation of the humerus -Rotation of the clavicle at the sternoclavicular joint -Scapular abduction and lateral rotation of the acromioclavicular joint -Straightening of thoracic kyphosis

A physical therapist evaluates a woman who is 30 weeks pregnant. She reports posterior pelvic pain that radiates to the posterior thigh and buttocks. Based on these symptoms, what is the most appropriate intervention for this patient? A. Elevation of the extremities B. Teach Kegel exercises C. Teach protection of the abdominal musculature; avoid abdominal exercises D. External stabilization with a sacroiliac support belt

D. External stabilization with a sacroiliac support belt

A physical therapist wearing sterile protective clothing establishes a sterile field prior to changing a dressing on a wound. Which area of the protective clothing would NOT be considered sterile even before coming in contact with a non-sterile object? A. Gloves B. Sleeves of the gown C. Front of the gown above waist level D. Front of the gown below waist level

D. Front of the gown below waist level *4 Rules of Asepsis that PT follows: 1. Know which items are sterile 2. Know when the item is not sterile 3. Separate sterile items from non-sterile items 4. If a sterile item becomes contaminated, the situation must be remedied immediately

Wound care has been ordered for a 55-year-old female patient recently admitted to the hospital for multiple lower extremity wounds and poor wound healing. During the assessment, the physical therapist notes an exudative wound on the patient's right medial ankle with infection, necrosis, and tunneling. Of the following, which dressing will be MOST appropriate for the patient's wound? A. Transparent film B. Hydrogel C. Foam D. Gauze

D. Gauze *Gauze dressings are non-adhesive absorptive dressings that are composed of synthetic fiber or cotton. Gauze dressings are used for exudative wounds, wounds with tunneling, and clean wounds that need protection. It is permissible to use gauze dressings in various manners, including dry, wet, moist, or impregnated with antiseptics. *Hydrogels are water-based gels contained within impregnated gauze, which are used with partial and full-thickness wounds or burns. *Foams are hydrophilic or hydrophobic semipermeable membranes with varying adhesive and absorptive properties, which are used for partial and full-thickness wounds with minimal to moderate exudate. *Transparent film is a clear, adhesive, semipermeable membrane dressing that is used with stage I and Il pressure ulcers, autolytic debridement, and skin donor sites.

What sign/symptom is not indicative of hyperglycemia? A. Dulled senses and confusion B. Decreased appetite C. Increased thirst D. Headache

D. Headache *symptoms of hyperglycemia include weakness, increased thirst, dry mouth, frequent scant urination, decreased appetite, nausea/vomiting, abdominal tenderness, dulled senses, confusion, diminished reflexes, paresthesia, flushed skin, signs of dehydration, deep and rapid respiration, rapid/weak pulse, fruity order of the breath, and hyperglycemic coma.

If a patient has a cherry-red appearance of his palms, what is most like the problem? A. Peripheral arterial disease B. Carotenemia C. Aneurysm of internal carotid artery D. Hepatic problems

D. Hepatic problems *A patient with hepatic problems often has palmer erythema. *This condition is also called Liver palm because it is associated with liver problems.

A physical therapist has been working with a 40-year-old female patient in an outpatient clinic for three months following AC reconstruction surgery. Over the last several weeks, the therapist has noticed personality changes and signs of memory loss in the patient. Concerned, the physical therapist decides to contact the patient's physician and recommends medical evaluation. Which of the following is the MOST likely reason for the therapist's concern? A. Surgical infection B. Hypoparathyroidism C. Dehydration D. Hyperparathyroidism

D. Hyperparathyroidism

A physical therapist designs an exercise program for a patient that minimizes the use of isometric work. Which medical conduction would MOST necessitate the need for this type of program? A. Anemia B. Diabetes mellitus C. Hemophilia D. Hypertension

D. Hypertension

Males with erectile dysfunction are unable to obtain and maintain an erection. This condition may be caused by neurogenic disorders, medications, and vascular changes. What hormonal cause may contribute to the development of erectile dysfunction? A. Parkinson's disease B. Diabetes mellitus C. Antiandrogen use D. Hypothyroidism

D. Hypothyroidism

Shock is the failure of the circulatory system to perfuse vital organs. Which of the following types of shock is MOST LIKELY to be caused by severe external or internal bleeding? A. Progressive B. Anaphylactic C. Cardiogenic D. Hypovolemic

D. Hypovolemic *Hypovolemic shock can be caused by hemorrhage, vomiting, or diarrhea, any of which can decrease the volume of healthy fluids in the body. Loss of body fluids (which may lead to hypovolemic shock) also occurs with dehydration, Addison's disease, burns, pancreatitis, or peritonitis. *Anaphylactic shock is caused by an allergic reaction, usually to a drug, insect bite, or food. *Cardiogenic shock is caused by damage to or other pathology of the heart. *Progressive shock is associated with restlessness and anxiety.

A physical therapist treats a patient with osteoporosis for a generalized strengthening program. Based on this diagnosis, which of the following medications would MOST likely be included in the patient's medication list? A. Esomeprazole (Nexium) B. Methotrexate (Rheumatrex) C. Metformin (Riomet) D. Ibandronate (Boniva)

D. Ibandronate (Boniva) *Esomeprazole (Nexium) is a proton pump inhibitor agent that is used in the treatment of dyspepsia or gastroesophageal reflux disease. Proton pump inhibitor agents inhibit the H+/K+-ATPase enzyme, blocking secretions of acid from gastric cells into the stomach. These agents prevent erosive esophagitis and may also possess antibacterial effects against H. pylori. *Methotrexate (Rheumatrex) is an antimetabolite medication that is used in the treatment of rheumatoid arthritis, severe psoriasis, and certain cancers. This medication can stop or at least slow the growth of cancer cells and helps to suppress the immune system. *Metformin (Riomet) is a medication used to control blood sugar levels in patients with type 2 diabetes mellitus. This medication decreases glucose production in the liver and increases the body's sensitivity to insulin. *Ibandronate (Boniva) is a bisphosphonate medication that is commonly used to treat patients with osteoporosis. Other common bisphosphonate medications include alendronate (Fosamax), risedronate (Actonel), and zoledronic acid (Reclast)

Prostatitis occurs when the prostate gland is infected and inflamed. There are three different types of prostatitis: acute bacterial, chronic, and nonbacterial. Which of the following options is a symptom related to nonbacterial inflammatory prostatitis? A. Malaise B. Urethral discharge C. Fever and chills D. Impotence

D. Impotence

During a treatment session, a patient makes a culturally insensitive remark that the physical therapist feels is offensive. Which of the following actions is the MOST appropriate for the therapist to take? A. Document the incident in the medical record B. Transfer the patient to another therapist's schedule C. Recommend discharging the patient from physical therapy D. Inform the patient that the remark was offensive and continue with treatment

D. Inform the patient that the remark was offensive and continue with treatment

Which of the following is true of the inverse stretch reflex? A. It allows muscle tension to come under the control of descending pathways. B. It increases stretch sensitivity. C. It increases the rate of firing from spindle afferents. D. It provides a stretch protection reflex.

D. It provides a stretch protection reflex. *Gamma reflex loops increase stretch sensitivity, increase the rate of firing from spindle afferents, and allow muscle tension to come under the control of descending pathways (reticulospinal, vestibulospinal, and others).

What statement is false regarding a gamma reflex loop? A. It allows muscle tension to come under the control of descending pathways. B. It increases the rate of firing from spindle afferents. C. It increases stretch sensitivity. D. It provides a stretch-protection reflex.

D. It provides a stretch-protection reflex. *Gamma reflex loops increase stretch sensitivity

Which of the following options is an example of a primary immunodeficiency disorder? A. Severe combined immunodeficiency disease B. Chronic diabetes C. Cirrhosis D. Leukemia

D. Leukemia *Immunodeficiency diseases are characterized by depressed or absent immune responses. Primary immunodeficiency disorders result from a defect in T cells, B cells, or lymphoid tissues (ex: Severe combined immunodeficiency disease (SCID)) *Secondary immunodeficiency disorders are caused by an underlying pathology or treatment, rather than a congenital defect (ex: Chronic diabetes, cirrhosis, and leukemia)

A PT observes that a patient has a limitation in PF ROM on the L during ambulation activities. This mobility restriction would be most apparent during which phase of gait? A. Terminal Swing B. IC C. Midstance D. Loading response

D. Loading response *Between IC and beginning of swing phase

A pt elevated on a tilt table to 60 degrees suddenly to demonstrate signs and symptoms of orthostatic hypotension. What is the MOST appropriate PT action? A. Lower the tilt table 10 degrees and monitor the patient's vital signs B. Lower the tilt table 20 degrees and monitor the patient's vital signs C. Lower the tilt table 40 degrees and monitor the patient's vital signs D. Lower the tilt table completely and monitor the patient's vital signs

D. Lower the tilt table completely and monitor the patient's vital signs *Hypotension = 20 mmHg or greater decrease in systolic blood pressure, dizziness, and nausea

Which of the following diagnostic procedures is the primary method of examination for tumors and vascular abnormalities? A. Carotid/vertebral artery doppler ultrasound B. CT C. Functional MRI D. MRI

D. MRI *Examination for tumors and vascular abnormalities

A physical therapist treats a patient referred to an outpatient rehabilitation facility. During the session the therapist identifies weakness, hypertonia, and hyperreflexia. What is the MOST likely diagnosis? A. Carpal tunnel syndrome B. Poliomyelitis C. Bell's palsy D. Multiple sclerosis

D. Multiple sclerosis

A physical therapist works with a patient on proprioceptive and balance activities due to a history of recent falls. The patient informs the therapist that they have recently been diagnosed with optic neuritis. Which of the following medical conditions should the therapist MOST suspect might be the cause of the patient's clinical presentation? A. Parkinson's disease B. Guillain-Barre syndrome C. Myasthenia gravis D. Multiple sclerosis

D. Multiple sclerosis *Because the optic nerve is an extension of the cerebral cortex, it is vulnerable to the effects of demyelination, such as occurs with multiple sclerosis. *Guillain-Barre syndrome (GBS) include distal symmetrical motor weakness and mild distal sensory impairments. *Myasthenia gravis is muscle weakness within the skeletal muscles, with other neurologic findings being normal *The initial symptoms of multiple sclerosis (MS) can include visual problems, sensory changes, clumsiness, weakness, ataxia, balance dysfunction, and fatigue.

A patient has a diagnosis of rheumatoid arthritis (RA). Which of the following statements about RA is TRUE? A. Weight gain is one systemic feature of RA. B. Onset is always gradual. C. RA is characterized by unilateral and asymmetrical joint involvement. D. NSAIDs can be used to treat RA patients.

D. NSAIDs can be used to treat RA patients. *RA is characterized by bilateral and symmetrical joint involvement; weight loss, fever, and extreme fatigue are all systemic features of this disease. The onset may be gradual or abrupt.

In order to maximize the effects of therapy for a patient diagnosed with Parkinson's disease, the physical therapist should coordinate which of the following levodopa dosing schedules with a home therapy session? A. Thirty minutes following the lunch dose B. One hour following the lunch dose C. Thirty minutes following the breakfast dose D. One hour following the breakfast dose

D. One hour following the breakfast dose

The physical therapist designs interventions to focus on long-term goals of increasing functional activities including cross legged sitting on the floor, transitions from the floor to standing, and improved mobility within the school building as part of a child's Individualized Educational Plan. Which of the following timeframes is the MOST appropriate to attain these goals? A. One month B. Four months C. Six months D. One year

D. One year *An Individualized Educational Plan (IEP) is designed for any school-aged child that requires therapy services. Long-term goals are based on a one-year plan. These services are provided through federal legislation through the Individuals with Disabilities Education Act (IDEA). *Federal legislation mandates the review of an IEP on a one-year basis. Goals relate to improving a child's educational experience. IDEA also provides for children 0-5 years of age through early intervention programs.

A physical therapist discusses common cognitive and behavioral changes associated with stroke with family members of a patient with right hemisphere damage and resultant left hemiplegia. Which term does NOT accurately describe the typical patient presentation? A. Poor judgment B. Impulsive C. Quick D. Overly cautious

D. Overly cautious

A patient is referred to physical therapy with a diagnosis of left shoulder impingement. During the examination, the physical therapist begins to suspect a systemic cause for the patient's pain. Which of the following symptoms would BEST support this hypothesis? A. Pain has been present for years B. Pain is alleviated with the use of cryotherapy C. Pain is aggravated by cervical rotation D. Pain remains unrelieved with rest

D. Pain remains unrelieved with rest *Pain that has a systemic cause is difficult to reproduce. Symptoms are usually unrelieved by rest or a change in position.

A therapist observes a patient who has right limb rigidity and bradykinesia after undergoing knee surgery. The therapist sends this patient to a neurologist for follow-up and the patient returns to therapy with a diagnosis of a neurological condition. What is the most realistic diagnosis for this patient? A. Cerebrovascular accident B. Myasthenia gravis C. Multiple sclerosis D. Parkinson's disease

D. Parkinson's disease

A physical therapist is treating a patient two weeks post rotator cuff repair. Which of the following interventions is the MOST appropriate? A. Light progressive shoulder exercises with resistive tubing B. Grade Ill sustained joint mobilizations C. Active range of motion up to 90 degrees of abduction D. Passive range of motion up to 10 degrees of external rotation

D. Passive range of motion up to 10 degrees of external rotation

What aspect of the body is not associated with the lymphatic system? A. Bone Marrow B. Mucosa-associated lymphoid tissue (MALT) C. Thymus D. Plasma

D. Plasma *Plasma is not a component of the lymphatic system. Plasma is found in the blood, circulating through the venous system. The bone marrow, mucosa-associated lymphoid tissue (MALT), and thymus are all components of the lymphatic system. Other components include the lymphatic vessels, lymph fluid, lymph nodes (located throughout the body), spleen, and tonsils.

While working in the ICU, you saw a patient's chart which indicates that the patient underwent surgery for removing one lung. What is this procedure called? A. Wedge resection B. Segmental resection C. Lobectomy D. Pneumonectomy

D. Pneumonectomy

You are preparing to see a patient in an outpatient facility with a known history of diabetes for an exercise session. Prior to the session, the patient checks their blood glucose and reports 68 mg/dL. Based on this information, what is the BEST action to take? A. Seek immediate medical treatment B. Continue with exercise as planned C. Continue with exercise, but provide 15g of carbohydrates every hour D. Provide sugar to the patient and do not exercise

D. Provide sugar to the patient and do not exercise *Exercise uses glucose. When working with patients who have a risk of becoming hypoglycemic, physical therapists should monitor glucose levels before and after exercise. These patients should not be instructed to exercise if their blood glucose is less than 70 mg/dL. Clinics should have carbohydrate snacks available when patients with diabetes are exercising. *Early signs of hypoglycemia include shakiness, sweating, tachycardia, feeling faint, dizziness, fatigue, and walking instability. *In the absence of hypoglycemia, carbohydrates should be provided during exercise (15g per hour). Immediate medical attention is indicated when hyperglycemia is present.

During examination of a patient diagnosed with ALS, you notice an exaggerated jaw jerk and Gag reflex, both upper extremities are graded 2 as per Modified Ashworth Scale. Suddenly, the patient started crying when you are revising functional goals saying "how unfortunate he is for suffering like this". The patient's relatives inform you that this has been a very frequent occurrence. What is most likely to be present here? A. IX nerve palsy B. Bulbar palsy C. V nerve palsy D. Pseudo-bulbar palsy

D. Pseudo-bulbar palsy *Patient crying being a frequent occurrence indicates emotional liability which is only seen in Pseudo-bulbar palsy

A patient sustained an injury in the parietal lobe. Which of the following is the function of the parietal lobe of the brain? A. Sensing auditory stimulus B. Motor area C. Associated motor area D. Sensory fibers for touch, pain, and temperature from opposite side of the body.

D. Sensory fibers for touch, pain, and temperature from opposite side of the body. *Parietal lobe = sensory fibers for touch, pain, and temperature from the opposite side of the body -Receives information from the sensory system of the body and integrates the sensory info to the motor area

There are both primary and secondary immunodeficiency disorders. What is an example of a primary immunodeficiency disorder? A. Cirrhosis B. Chronic diabetes C. Leukemia D. Severe combined immunodeficiency disease

D. Severe combined immunodeficiency disease *Primary immunodeficiency disorders result from a defect in T cells, B cells, or lymphoid tissues. *Secondary immunodeficiency disorders are caused by an underlying pathology or treatment, rather than a congenital defect.

A physical therapist treats a patient with a medical history that includes Klinefelter syndrome. What is the exact etiology of this genetic condition? A. Autosomal dominant disorder B. Autosomal trisomy disorder C. Partial deletion disorder D. Sex chromosome aneuploidy disorder

D. Sex chromosome aneuploidy disorder *Aneuploidy is the presence of an abnormal number of chromosomes in a cell. Klinefelter syndrome is a genetic condition that results when a male is born with at least one extra copy of the X sex chromosome. *Klinefelter syndrome is a common genetic condition affecting males, and often is not diagnosed until adulthood. *Symptoms include gynecomastia, small testes, reduced libido, and quiet personality.

A physical therapist is working with a 38- year-old female patient admitted to the hospital following a traumatic brain injury. While transfer training, the patient demonstrates higher than normal muscle weakness. Upon returning to the patient's room, the physical therapist notes new test results indicating potassium serum levels of 6.5 mEq/L. Of the following, what is the MOST likely explanation for this? A. Diarrhea B. Renal tubular disease C. Alkalosis D. Sickle cell anemia

D. Sickle cell anemia *The normal potassium serum level is 3.5 to 5.5 mEq/L. If an individual's potassium serum level is greater than 5.5 mEq/L, hyperkalemia occurs. Sickle cell anemia is a cause associated with hyperkalemia. *Other causes associated with hyperkalemia include kidney disease, metabolic acidosis, and diabetic ketoacidosis. *Renal tubular disease, alkalosis, and diarrhea are causes associated with hypokalemia or a deficiency in potassium.

A physical therapist follows disinfection guidelines by utilizing only sterilized equipment during sessions. Sometimes, smaller equipment is cleaned using an autoclave. What is the mechanism through which an autoclave sterilizes medical instruments and other objects? A. Chlorine compound B. Dry heat C. lonizing radiation D. Steam

D. Steam *An autoclave is a chamber that uses heat and water pressure to produce steam that sterilizes instruments. It is not recommended for heat-sensitive objects. *Dry heat sterilizes instruments by using prolonged, high-heat exposure in an oven. lonizing radiation sterilizes plastics, sutures, and some medications. Chlorine and chlorine compounds, such as bleach, are used to disinfect instruments.

A patient with knee pain since 4 weeks visits an outpatient facility. During examination the patient's active flexion range is full while extension is 135 to 18* in sitting, with a normal "firm' endfeel when checked by the therapist. The patient does not have any tightness of Hamstrings. Which of the following intervention would be most appropriate in this patient? A. Posterior glide to the tibia B. Anterior glide to the tibia C. Strengthen Gracilis D. Stretch sartorius

D. Stretch sartorius *Sartorius = ER of femur + IR of tibia especially during end extension

You are a physical therapist working in an inpatient hospital and performing your chart review of a patient before their session. You learn from their chart that they have injuries to the lumbar plexus nerve roots. Which nerve roots does the lumbar plexus arise from? A. L1 through L3 B. T12 through L2 C. L4 through S3 D. T12 through L4

D. T12 through L4 *The cervical plexus arises from nerve roots C1 through C4. *The brachial plexus arises from nerve roots C5 through T1 *The lumbar plexus arises from nerve roots T12 through L4. *The sacral plexus arises from nerve roots L4 through S3.

A 28 years old mother visits diagnosed with RA visits an in-patient care with frequent bouts of fatigue and joint pain. What advice would highly benefit the mother's fatigue? A. It is ideal to carry out strenuous tasks first followed with easier ones B. Sitting on low stool so that it is easier to get up C. Use soft furniture to lessen your energy expenditure D. Tend your baby on your lap instead of lifting it

D. Tend your baby on your lap instead of lifting it

Constipation is a common gastrointestinal disorder that is caused by a reduction in normal stool eliminations. If a person suffers from constipation, which of the following symptoms are they MOST LIKELY to experience? A. Lower back pain B. Upper abdominal pain C. Scapular pain D. Tenderness in the anterior hip, groin, or thigh regions

D. Tenderness in the anterior hip, groin, or thigh regions

Patients who have fewer episodes of eliminating normal stool are considered to have constipation. What symptoms would someone with constipation most likely have? A. Lower back pain B. Scapular pain C. Upper abdominal pain D. Tenderness in the anterior hip, groin, or thigh regions

D. Tenderness in the anterior hip, groin, or thigh regions *Visceral pain from the stomach, pancreas, pleura, and spleen can refer to the upper abdomen. *Visceral pain from the gallbladder and pancreas can refer to the scapular region. *The gall bladder and heart can refer pain to the lower back.

Thompson's test is performed to look for which of the following? A. For anterior talofibular ligament B. For flat foot C. For distal tibiofibular syndesmosis D. The integrity of the Achilles' tendon

D. The integrity of the Achilles' tendon

Of the following options, what is the BEST description of stroke volume? A. The volume of blood that is emitted per minute B. The volume of blood that is emitted from the left ventricle per minute C. The volume of blood that is emitted from the right ventricle per minute D. The volume of blood that is emitted during each myocardial contraction

D. The volume of blood that is emitted during each myocardial contraction *The stroke volume is the volume of blood that is emitted during each myocardial contraction. This is measured by contractions, not by minutes. The normal stroke volume range is between 55 and 100 mL/beat. *The volume of blood that is emitted from the left or right ventricle per minute describes cardiac output.

What is the degree of burn when the epidermis, dermis, and subcutaneous fats are destroyed by the injury and there are ischemic changes without pain? A. Second degree burn injury B. First degree burn injury C. Fourth degree burn injury D. Third degree burn injury

D. Third degree burn injury = Full thickness burn *Sometimes muscles are also involved and appears white to ischemia

A patient presents to physical therapy for treatment of a non-operative osteoporotic fracture. Which of the following structures would have been LEAST likely affected? A. Distal radius B. Vertebral body C. Proximal femur D. Tibial plateau

D. Tibial plateau

A patient presents to PT for treatment of a non-operative osteoporotic fracture. Which of the following structures would have been LEAST likely affected? A. Distal radius B. Vertebral body C. Proximal femur D. Tibial plateau

D. Tibial plateau *Impaired bone formation due to declining osteoblast function

A physical therapist examines a patient who has been experiencing intermittent buttock and lateral thigh pain for two weeks. Presently, the patient rates the pain as a "3" on a 0-10 scale, however, indicates that the pain is a "6" or a "7" during activity or at night. Which of the following conditions is MOST consistent with this clinical presentation? A. Sacroilitis B. lliolumbar syndrome C. Piriformis syndrome D. Trochanteric bursitis

D. Trochanteric bursitis *Piriformis syndrome = Piriformis muscle irritates the sciatic nerve causing pain in the buttocks *Trochanteric bursitis = inflammation of the bursa of the greater trochanter. Lateral thigh pain that increases with activity or prolonged rest

A physical therapist examines a patient who has been experiencing intermittent buttock and lateral thigh pain for two weeks. Presently, the patient rates the pain as a "3" on a 0-10 scale, however, indicates that the pain is a "6" or a "7" during activity or at night. Which of the following conditions is MOST consistent with this clinical presentation? A. Sacroillitis B. lliolumbar syndrome C. Piriformis syndrome D. Trochanteric bursitis

D. Trochanteric bursitis *Sacroiliitis refers to inflammation of one or both of the sacroiliac joints. The patient is typically tender to palpation directly over the sacroiliac joint. *lliolumbar syndrome, also known as iliac crest pain syndrome, is caused by inflammation or a tear of the iliolumbar ligament. The condition often leads to referred pain in the pelvis or groin. The patient is often tender to palpation over the iliac crest and pain tends to be exacerbated with sidebending. *Piriformis syndrome refers to a condition in which the piriformis muscle irritates the sciatic nerve causing pain in the buttocks and referred pain along the course of the sciatic nerve. The primary patient complaint is buttock pain that is made worse by sitting, stair climbing or squatting. *Trochanteric bursitis refers to inflammation of the trochanteric bursa, which is a pad-like sac that protects the soft tissue structures that cross the posterior portion of the greater trochanter. The patient is often extremely sensitive to palpation over the bursa and may experience lateral thigh pain that is exacerbated by activity or periods of prolonged rest.

Which of the following disorders may be classified as a type of inflammatory bowel disease? A. Peptic ulcer disease B. Irritable bowel syndrome (IBS) C. Diverticular disease D. Ulcerative colitis (UC)

D. Ulcerative colitis (UC)

A patient's job requires them to move boxes weighing 35 pounds from a transport cart to an elevated conveyor belt. The patient can complete the activity, however, is unable to prevent hyperextension of the spine. Which of the following actions is the MOST appropriate for the therapist to implement? A. Initiate a pelvic stabilization program B. Design an abdominal strengthening program C. Review proper body mechanics D. Use an elevated platform when placing boxes on the belt

D. Use an elevated platform when placing boxes on the belt *In order to eliminate hyperextension of the spine it may be necessary to modify the workstation. The most reasonable modification would be to utilize an elevated platform in order to minimize the height of the conveyor belt. In many instances, it is possible to modify a work site without utilizing large amounts of resources (e.g., time, money).

A physical therapist progresses a patient involved in a phase I cardiac rehabilitation program through an established exercise protocol. The protocol indicates the patient should be performing activities requiring 3-4 METs. Which of the following activities is the MOST appropriate to achieve this goal? A. Level walking at 1 mph B. Jogging at 5 mph C. Cycling at 10 mph D. Walking on a treadmill at 3 mph

D. Walking on a treadmill at 3 mph *Level walking at 1 mph is approximately 1-2 METs *Jogging at 5 mph is approximately 7-8 METs. *Cycling at 10 mph is approximately 5-6 METs. *Walking on a treadmill at 3 mph is approximately 3-4 METs.

Skin conditions are treated differently depending on whether they are viral or non-viral. What is known as a viral skin condition? A. Cellulitis B. Impetigo C. Tinea Pedis D. Warts

D. Warts *Warts are a common, benign infection caused by HPVs

Which of the following choices is the most ideal way to implement phase Il CDT compression techniques? A. With high pressure B. To manage the symptoms of patients with stage Il lymphedema C. Compression pumps should only be applied in phase I of CDT, not phase Il D. With low pressure

D. With low pressure *Compression pumps should be used with low pressure, as high pressure may damage the lymph nodes. This technique is only appropriate for patients with stage 1 lymphedema, as it may exacerbate inflammation in stage 2 or stage 3 patients. It should not be applied during phase l of healing.

Flat Back Posture

Decrease in the normal inward curve of the lower back, with the pelvis in posterior tilt

Sway Back Posture

Decreased anterior lumbar curve and increased posterior thoracic curve from neutral

What is probability sampling?

Each member of the population must have the same probability fo being selected for the sample --> since the sample should be free of bias and representative of the population

What is the Performance-Oriented Mobility Assessment (POMA) used for?

Examines balance and walking -Max Score = 28 -19-24 = Moderate risk for falls -Lower Score = 19 = High risk for falls

Hyperoxia

Excess oxygen in tissues and organs

Causes of short leg include what?

Flat foot/pes planus Coxa Vara Genu valgum Poor standing posture Anteversion of femur Anteriorly rotated innominate bone

What motions are most likely affected with an upper motor neuron syndrome exhibited in the elbow spasticity?

Flexion

How would you stretch the hip capsule?

Flexion Abduction ER or FABER

What motions are most likely affected with an upper motor neuron syndrome exhibited in the wrist spasticity?

Flexion Adduction

What are possible side effects of NSAIDs?

Fluid retention Gastrointestinal irritation Renal or liver problems Prolonged bleeding

Fluoroscopy is what?

Fluoroscopy is a radiographic test used to observe diaphragmatic excursion. This test uses an X-ray beam that allows for continuous observation.

Round Back Posture

Forward head w/ cervical spine hyperextended Scapula protracted Increase in thoracic kyphosis Hip flexed Knees hyperextended

What is tertiary care?

Highly specialized diagnostic, therapeutic, and rehab services that require specific tools or staff (beyond what a community hospital can do) e.g. open heart surgery, organ transplants, chemo, extremely premature infants

What motions are most likely affected with an upper motor neuron syndrome exhibited in the hip adduction spasticity?

Hip adduction Internal Rotation Extension

Airplane splint

Holds arm in abduction with the elbow in approximately 90 degrees of flexion --> protecting against a shoulder adduction contracture Used by pt with burns affecting the shoulder and axillary regions

What is swan neck deformity?

Hyperextension of PIP and flexion of DIP Dorsal displacement of the lateral bands of extensor digitorum

What is associated with hyperkalemia?

Hyperkalemia = excessive potassium in the blood Sickle Cell Kidney Disease Metabolic acidosis Diabetic ketoacidosis

Hyperoxemia

Hyperoxemia refers to an increased oxygen concentration in the blood measured by arterial oxygen partial pressure (Pa02) values. A Pa02 greater than 120 mm Hg constitutes hyperoxemia.

What are some potentials that can cause urge urinary incontinence?

Hypersensitive bladder Detrusor muscle instability Hyperreflexia

What is associated with hypokalemia?

Hypokalemia = deficiency in potassium in the blood Renal tubular disease Alkalosis Diarrhea

Hypoemia

Hypoxemia refers to a decreased oxygen concentration in the blood measured by arterial oxygen partial pressure (Pa02) values. A Pa02 less than 80 mm Hg constitutes hypoxemia.

What is the Tinetti Performance Oriented Mobility Assessment?

Identify if there is an increased risk for falling. 5-15 minutes -Tool measures balance and gait using 2 to 3 point ordinal scale -Higher Risk = Low Total Score

Venous stasis ulcer

Inadequate venous return and eventual tissue damage and ulceration -Shallow and irregular in shape -Moderate to heavy exudate -Dark purple or rusty color -Around the ankle

When are rashes typical?

Inflammation Vasomotor disturbances Skin disease Mediation

What is gout?

Inflammatory condition characterized by acute pain due to deposits of rate crystals in the joint *Most common in the 1st metatarsophalangeal joint

What does visceral pain from the pancreas can refer to?

L lower back

Corticospinal Tract

Lateral Corticospinal -Skilled distal limb movements (forearm and hand) Anterior Corticospinal -Axial and proximal limb muscles The largest descending pathway where 80% of the fibers decussate and descend on the opposite side; 20% continue to descend ipsilaterally. The corticospinal tract carries information from the motor cortex directly to the spinal cord

What can a short leg cause?

Lateral pelvic tilt (pelvic drop) and side bending of trunk away from short side

What does visceral pain from the pleura and spleen can refer to?

Lower L abdominal quadrant

Valgus Stress Test

MCL and PCL *Full extension = MCL and posteromedial capsule

Distributed practice

Method of practice where a patient's rest time between trials is equal to or greater than the practice time itself

Moon-shaped face

Moon-shaped face refers to a rounded appearance of the face due to fat deposits on the side of the face. This condition is often associated with hyperadrenalism due to an overproduction of cortisol (Cushing's syndrome) or an overproduction of aldosterone (Conn's syndrome). Symptoms associated with hyperadrenalism include moon-shaped face, fat accumulation often behind the shoulders known as a buffalo hump, fatigue, weakness, HTP, and weight gain

What are the first symptoms of heat-related illness?

Muscle cramping in the legs or abdomen *Heat-related = heat exhaustion and heat stroke

Balance grades are as follows:

Normal: The patient can maintain steady balance without handhold support (static). They accept maximal challenge and can easily shift their weight at full range in all directions (dynamic). Good: The patient can maintain balance without handhold support, exhibiting limited postural sway (static). They accept moderate challenge and are able to maintain balance while picking an object up from the floor (dynamic). Fair: The patient can maintain balance with handhold support, and may need occasional minimal assistance (static). They accept minimal challenge and are able to maintain balance while turning their head/trunk (dynamic). Poor: The patient requires handhold support and moderate to maximal assistance (static). They cannot accept challenge or move without loss of balance (dynamic). Absent: The patient is unable to maintain balance.

Canalithiasis

Occurs when otoconia are moving within the semicircular canal, causing vertigo and nystagmus that resolves within 60 seconds

A neurogenic bladder is associated with what?

Overflow urine incontinence

Medulla oblongata

Part of the brainstem that controls vital life-sustaining functions such as heartbeat, breathing, blood pressure, and digestion.

What is acute rehabilitation?

Patient generally stays an average of 15 days and receives therapy daily for a minimum of 3 hours

What is subacute care?

Patient receives treatment in a hospital or skilled nursing facility

What is acute care?

Practice environment in which the patient receives treatment for a short-term illness in a hospital for a few days

Constant practice

Practice involving a constrained set of materials and skills

Variable practice

Practice of a given task under differing conditions -Ex: Patient practices on level surfaces or uneven surfaces

To mangage dyspnea, what do you have to do?

Pursed lip breathing is most effective

What are some different drainages?

Purulent = thick milky drainage, that can by gray, green, or yellow Serous = thin, clear, watery serum Sanguineous = contains blood and is prevalent among deeper wounds

What is the Functional Independence Measure (FIM) score?

Records the assistance during: -Self care -Sphincter control -Transfers -Locomotion -Communication -Social cognition 1 = Total Assistance (less than 25%) 2 = Maximum Assistance (25% or more) 3 = Moderate Assistance (50% or more) 4 = Minimal Assistance (75% or more) 5 = Supervision (100% supervision) 6 = Modified Independence (use of devices/equipment) 7 = Complete Independence (timely and safety)

Tectospinal Tract

Responsible for visual information related to spatial awareness. The tract ends at the cervical spine and controls the musculature of the neck as well as head position

Systematic sampling

select some starting point and then select every kth element in the population

Long sitting test

SIJ dysfunction due to functional leg length discrepancy. Have patient come from Supine to long sitting. Observe change in medial malleolar positions

What does visceral pain from the liver, diaphragm, or pericardium refer to?

Shoulder

The stages of sequential recovery from stroke (CVA) are what?

Stage 1: initial flaccidity; no voluntary movement Stage2: emergence of spasticity, hyperreflexia, synergies Stage 3: voluntary movement possible, but only in synergies; spasticity strong Stage 4: voluntary control in isolated joint movements emerging; corresponding decline of spasticity and synergies Stage 5: increasing voluntary control out of synergy; coordination deficits present Stage 6: control and coordination near normal

Pressure Ulcer Stages

Stage I: -Skin is fully intact, -Nonblanchable -May have a temperature change (cool or warm) -May have sensation (itching or pain). Stage II: -Partial-thickness skin loss involving the epidermis and/or dermis skin layers and presents as a blister, abrasion, or shallow crater. Stage III: -Full-thickness skin loss that involves subcutaneous tissue damage or necrosis, and presents as a deep crater. Stage IV: -Full-thickness skin loss that involves tissue necrosis and damage to bone, muscle, or other supporting structures.

Weak pelvic floor musculature may cause what?

Stress urinary incontinence

Galant Reflex

Stroking along one side of the spine while newborn is in ventral suspension (face down) causes lateral flexion of lower body toward stimulated side

Connective tissue massage

Stroking technique that uses the pads of the middle and ring fingers. It is used to treat numerous arterial and venous disorders

What is polymyalgia rheumatica (PMR)?

Systemic inflammatory condition that is experienced primarily in older adults

What is slough?

Tan, yellow, or green scab-like material.

Urge incontinence

the loss of urine in response to a sudden, urgent need to void; the person cannot get to a toilet in time

Functional incontinence

the person has bladder control but cannot use the toilet in time due to a functional limitation

Motions in the transverse plane occur around a _____________ axis

vertical axis

4 stages of lymphedema

• Stage 0 (Latent) - At risk; lymphatic vessels sustained damage, which isn't yet apparent = asymptomatic - Transport capacity still sufficient • Stage 1 (Spontaneously reversible) - Pitting stage; normal in AM but increases size throughout the day -Resolves with elevation -Stemmer's sign = negative • Stage 2 (Spontaneously irreversible) - Spongy consistency & non-pitting, beginning of hardening of limbs with increasing size -Elevation does not reduce swelling -Stemmer's sign = positive • Stage 3 (Lymphostatic elephantiasis) - Irreversible swelling, hard/fibrotic tissue, unresponsive - Possible changes to skin color - Changes may limit mobility


संबंधित स्टडी सेट्स

Chapter 16 listening quiz answers

View Set

Med Surg. Chapter 45 Digestive and Gastrointestinal Treatment Modalities

View Set